高校数学の質問スレPART346

このエントリーをはてなブックマークに追加
1132人目の素数さん
前スレ
高校数学の質問スレPART345
http://uni.2ch.net/test/read.cgi/math/1356018339/

【質問者必読!】
まず>>1-3をよく読んでね

数学@2ch掲示板用 掲示板での数学記号の書き方例と一般的な記号の使用例
http://mathmathmath.dotera.net/

・まずは教科書、参考書、web検索などで調べるようにしましょう。(特に基本的な公式など)
・問題の写し間違いには気をつけましょう。
・長い分母分子を含む分数はきちんと括弧でくくりましょう。
  (× x+1/x+2 ;  ○((x+1)/(x+2)) )
・丸文字、顔文字、その他は環境やブラウザによりうまく表示できない場合があります。
 どうしても画像を貼る場合はPCから直接見られるところに見やすい画像を貼ってください。
 ピクトはPCから見られないことがあるので避けてください。
・質問者は名前を騙られたくない場合、トリップを付けましょう。 (トリップの付け方は 名前(N)に 俺!#oretrip ←適当なトリ)
・質問者は回答者がわかるように問題を書くようにしましょう。でないと放置されることがあります。
  (変に省略するより全文書いた方がいい、また説明なく習慣的でない記号を使わないように)
・質問者は何が分からないのか、どこまで考えたのかを明記しましょう。それがない場合、放置されることがあります。
  (特に、自分でやってみたのにあわないので教えてほしい、みたいなときは必ず書くように)
・970くらいになったら次スレを立ててください。
2132人目の素数さん:2013/01/22(火) 17:25:41.33
主な公式と記載例

(a±b)^2=a^2±2ab+b^2
(a±b)^3=a^3±3a^2b+3ab^2±b^3
a^3±b^3=(a±b)(a^2干ab+b^2)

√a*√b=√(ab)、√a/√b=√(a/b)、 √(a^2b)=a√b [a > 0、b > 0]
√((a+b)±2√(ab))=√a±√b [a > b > 0]

ax^2+bx+c=a(x-α)(x-β)=0 [a≠0、α+β=-b/a、αβ=c/a]
(α,β)=(-b±√(b^2-4ac))/2a  [2次方程式の解の公式]

a/sin(A)=b/sin(B)=c/sin(C)=2R [正弦定理]
a^2=b^2+c^2-2bccos(A)      [余弦定理]

sin(a±b)=sin(a)cos(b)±cos(a)sin(b)  [加法定理]
cos(a±b)=cos(a)cos(b)干sin(a)sin(b)

log_{a}(xy)=log_{a}(x)+log_{a}(y)
log_{a}(x/y)=log_{a}(x)-log_{a}(y)
log_{a}(x^n)=n(log_{a}(x))
log_{a}(x)=(log_{b}(x))/(log_{b}(a))  [底の変換定理]

f'(x)=lim_[h→0] (f(x+h)-f(x))/h  [微分の定義]
(f±g)'=f'±g'、(fg)'=f'g+fg'、(f/g)'=(f'g-fg')/(g^2) [和差積商の微分]
3132人目の素数さん:2013/01/22(火) 17:26:14.15
基本的な記号の使い方は以下を参照してください。その他については>>1のサイトで。
■ 足し算/引き算/掛け算/割り算(加減乗除)
 a+b → a 足す b   (足し算)     a-b → a 引く b    (引き算)
 a*b → a 掛ける b  (掛け算)     a/b → a 割る b    (割り算)
■ 累乗 ^
 a^b     a の b乗
 a^(b+1)  a の b+1乗
 a^b + 1  (a の b乗) 足す 1
■ 括弧の使用
 a/(b + c) と a/b + c
 a/(b*c)  と a/b*c
 はそれぞれ、違う意味です。括弧を多用して、キチンと区別をつけてください。
■ 数列
 a[n] or a_(n)     → 数列aの第n項目
 a[n+1] = a[n] + 3  → 等差数列の一例
 Σ[k=1,n]a_(k)     → 数列の和
■ 積分 ( "∫"は「せきぶん」「いんてぐらる」「きごう」「すうがく」などで変換せよ(環境によって異なる).唐ヘ高校では使わない)
 ∫[0,1] x^2 dx = (x^3)/3|_[x=0,1]
 ∫[0,x] sin(t) dt
■ 三角関数
 (sin(x))^2 + (cos(x))^2 = 1
 cos(2x) = (cos(x))^2 - (sin(x))^2
■ ベクトル
 AB↑ a↑
 ベクトル:V=[V[1],V[2],...], |V>, V↑, vector(V)
 (混同しない場合はスカラーと同じ記号でいい.通常は縦ベクトルとして扱う.)
■行列
 (全成分表示):M=[[M[1,1],M[2,1],...],[M[1,2],M[2,2],...],...], I=[[1,0,0,...],[0,1,0,...],...]
 (行(または列ごと)に表示する. 例)M=[[1,-1],[3,2]])
■順列・組合せ
 P[n,k]=nPk, C[n.k]=nCk, H[n,k]=nHk
4あのこうちやんは始皇帝だった:2013/01/22(火) 19:16:47.46
 テメ〜、いいかげんにしねえと、ブッ殺すぞ!

 60代の、無職の、知的障害の、女性恐怖症の、頭デッカチの虚弱児・ひ弱の、ゴミ・クズ・カス・無能・虫けらのクソガキ!

 死ね!!!!!!!!!!!!!!!!!!!!!!
5132人目の素数さん:2013/01/22(火) 19:24:06.68
          __ノ)-'´ ̄ ̄`ー- 、_
        , '´  _. -‐'''"二ニニ=-`ヽ、
      /   /:::::; -‐''"        `ーノ
     /   /:::::/           \
     /    /::::::/          | | |  |
     |   |:::::/ /     |  | | | |  |
      |   |::/ / / |  | ||  | | ,ハ .| ,ハ|
      |   |/ / / /| ,ハノ| /|ノレ,ニ|ル' 
     |   |  | / / レ',二、レ′ ,ィイ|゙/   
.     |   \ ∠イ  ,イイ|    ,`-' |      
     |     l^,人|  ` `-'     ゝ  |        このスレは馬と鹿と豚さんばかりね。
      |      ` -'\       ー'  人            
    |        /(l     __/  ヽ、          
     |       (:::::`‐-、__  |::::`、     ヒニニヽ、         
    |      / `‐-、::::::::::`‐-、::::\   /,ニニ、\            
   |      |::::::::::::::::::|` -、:::::::,ヘ ̄|'、  ヒニ二、 \
.   |      /::::::::::::::::::|::::::::\/:::O`、::\   | '、   \
   |      /:::::::::::::::::::/:::::::::::::::::::::::::::::'、::::\ノ  ヽ、  |
  |      |:::::/:::::::::/:::::::::::::::::::::::::::::::::::'、',::::'、  /:\__/‐、
  |      |/:::::::::::/::::::::::::::::::::::::::::::::::O::| '、::| く::::::::::::: ̄|
   |     /_..-'´ ̄`ー-、:::::::::::::::::::::::::::::::::::|/:/`‐'::\;;;;;;;_|
   |    |/::::::::::::::::::::::\:::::::::::::::::::::::::::::|::/::::|::::/:::::::::::/
    |   /:::::::::::::::::::::::::::::::::|:::::::::::::::::::::O::|::|::::::|:::::::::::::::/
6132人目の素数さん:2013/01/22(火) 19:50:40.49
たまに大問1つまるまる確率の漸化式とか期待値の問題ってあるじゃん
ああいうの出されると何からはじめればいいかわかんなくなるんだけど
なんかフローチャートとかコツとかありますか?
7132人目の素数さん:2013/01/22(火) 19:56:06.37
文節一つ一つを順番に漫画化する
未知数が出てきたら適当に文字を当てまくりながら進む
書き終えたら関係式を立てられそうなところは立ててみる
あとはパズル
8132人目の素数さん:2013/01/22(火) 20:46:01.20
>>4-5
他にすることない人生なの?
9132人目の素数さん:2013/01/22(火) 21:41:20.63
ODの求め方が分かりません
10132人目の素数さん:2013/01/22(火) 21:54:46.30
余弦定理
11132人目の素数さん:2013/01/22(火) 21:55:37.85
?
12132人目の素数さん:2013/01/22(火) 22:05:59.32
          __ノ)-'´ ̄ ̄`ー- 、_
        , '´  _. -‐'''"二ニニ=-`ヽ、
      /   /:::::; -‐''"        `ーノ
     /   /:::::/           \
     /    /::::::/          | | |  |
     |   |:::::/ /     |  | | | |  |
      |   |::/ / / |  | ||  | | ,ハ .| ,ハ|
      |   |/ / / /| ,ハノ| /|ノレ,ニ|ル' 
     |   |  | / / レ',二、レ′ ,ィイ|゙/   
.     |   \ ∠イ  ,イイ|    ,`-' |      
     |     l^,人|  ` `-'     ゝ  |        このスレは馬と鹿と豚さんばかりね。
      |      ` -'\       ー'  人            
    |        /(l     __/  ヽ、          
     |       (:::::`‐-、__  |::::`、     ヒニニヽ、         
    |      / `‐-、::::::::::`‐-、::::\   /,ニニ、\            
   |      |::::::::::::::::::|` -、:::::::,ヘ ̄|'、  ヒニ二、 \
.   |      /::::::::::::::::::|::::::::\/:::O`、::\   | '、   \
   |      /:::::::::::::::::::/:::::::::::::::::::::::::::::'、::::\ノ  ヽ、  |
  |      |:::::/:::::::::/:::::::::::::::::::::::::::::::::::'、',::::'、  /:\__/‐、
  |      |/:::::::::::/::::::::::::::::::::::::::::::::::O::| '、::| く::::::::::::: ̄|
   |     /_..-'´ ̄`ー-、:::::::::::::::::::::::::::::::::::|/:/`‐'::\;;;;;;;_|
   |    |/::::::::::::::::::::::\:::::::::::::::::::::::::::::|::/::::|::::/:::::::::::/
    |   /:::::::::::::::::::::::::::::::::|:::::::::::::::::::::O::|::|::::::|:::::::::::::::/
13前スレ952:2013/01/22(火) 22:24:35.21
前スレ961さま

丁寧なレスどうもありがとうございます。初等幾何で十分に分かりやすいです。

この問題で、座標とかベクトルで解こうとするのはセンスなさすぎということでしょうかね。。。
14132人目の素数さん:2013/01/22(火) 22:26:04.59
          __ノ)-'´ ̄ ̄`ー- 、_
        , '´  _. -‐'''"二ニニ=-`ヽ、
      /   /:::::; -‐''"        `ーノ
     /   /:::::/           \
     /    /::::::/          | | |  |
     |   |:::::/ /     |  | | | |  |
      |   |::/ / / |  | ||  | | ,ハ .| ,ハ|
      |   |/ / / /| ,ハノ| /|ノレ,ニ|ル' 
     |   |  | / / レ',二、レ′ ,ィイ|゙/   
.     |   \ ∠イ  ,イイ|    ,`-' |      
     |     l^,人|  ` `-'     ゝ  |        このスレは馬と鹿と豚さんばかりね。
      |      ` -'\       ー'  人            
    |        /(l     __/  ヽ、          
     |       (:::::`‐-、__  |::::`、     ヒニニヽ、         
    |      / `‐-、::::::::::`‐-、::::\   /,ニニ、\            
   |      |::::::::::::::::::|` -、:::::::,ヘ ̄|'、  ヒニ二、 \
.   |      /::::::::::::::::::|::::::::\/:::O`、::\   | '、   \
   |      /:::::::::::::::::::/:::::::::::::::::::::::::::::'、::::\ノ  ヽ、  |
  |      |:::::/:::::::::/:::::::::::::::::::::::::::::::::::'、',::::'、  /:\__/‐、
  |      |/:::::::::::/::::::::::::::::::::::::::::::::::O::| '、::| く::::::::::::: ̄|
   |     /_..-'´ ̄`ー-、:::::::::::::::::::::::::::::::::::|/:/`‐'::\;;;;;;;_|
   |    |/::::::::::::::::::::::\:::::::::::::::::::::::::::::|::/::::|::::/:::::::::::/
    |   /:::::::::::::::::::::::::::::::::|:::::::::::::::::::::O::|::|::::::|:::::::::::::::/
15132人目の素数さん:2013/01/22(火) 22:46:24.51
>>13
座標は軌跡求めたり直線や円、点を求めたりするにはいいけどあの手の証明にはまるきり使えないと思っていい
途中でちょっと交点の座標とか直線の式求めるだけでめちゃくちゃ煩雑になること多い
ベクトルはまだ使い道は広いかな
平面幾何(+三角比)という方法は原始的だがなかなか使える
16132人目の素数さん:2013/01/23(水) 02:11:34.68
極値を持つ三次関数f(x)について、極大値をとるxの値をα、極小をとるxの値をβとしたとき、
変曲点(α+β)/2に対して点対称なグラフになっているため、極大と同じ値をとるx座標は、βにβー(α+β)/2を足した(3/2)β-α/2であると教わりました
しかしなぜ点対称だったら、変曲点から極小までの距離と、極小から「極大と同じy座標をとるx座標」までの距離が等しいのかがいまいち理解できません
よろしくお願いします
17132人目の素数さん:2013/01/23(水) 02:36:56.92
>>16
『点対称
⇒変曲点から極小までの距離と、極小から「極大と同じy座標をとるx座標」までの距離が等しい』
はあなたの指摘通りおかしい。一般には成り立たない

だが詭弁とはいえ三次関数においては
f(α)=f(γ),α≠γであるγ=(3/2)β-α/2を
覚えやすい方便ではある

そういう教え方をする教師であると割りきって
さっさと勉強を進めるのがお互いにとって吉だと思う
18132人目の素数さん:2013/01/23(水) 04:04:55.74
I(s)=1/(Rs+Ls^2+1/c)

振動系になると思うのですが、どやって逆ラプラス変換すればいいのでしょうか?
1916:2013/01/23(水) 04:05:33.34
ありがとうございます
ではなぜf(α)=f(γ),α≠γであるγ=(3/2)β-α/2が成り立つのかについて考えたのですが、代入して計算する以外に証明法はありませんか?
20132人目の素数さん:2013/01/23(水) 04:24:35.85
f(x)=x^3-3x で確かめるだけで良い
21132人目の素数さん:2013/01/23(水) 14:09:48.20
「x=aで極値」という条件を「f'(a)=0」として使うとき吟味が必要。
と教材に書いているのですが何故必要なのか教えてください
22132人目の素数さん:2013/01/23(水) 14:28:16.97
山の頂上は平らであるが
連峰の頂上がひとつであるとは限らない
23132人目の素数さん:2013/01/23(水) 14:40:35.57
>>22
スッキリしました
ありがとうございます
24132人目の素数さん:2013/01/23(水) 14:50:37.75
逆の場合ならば、
y = x^3 は、 f'(a)=0 (a=0) はあるが、極値はない

一般的には
> f'(a)=0 ならば x=aで極値
とは限らない
25132人目の素数さん:2013/01/23(水) 15:02:45.26
>>23>>22で理解できたとはとても思えない
26132人目の素数さん:2013/01/23(水) 15:16:04.95
>>21
関数f(x)=|x| はx=0で極小値をとる。
しかし、x=0では微分可能でないから f'(0)は意味がない。
27132人目の素数さん:2013/01/23(水) 15:20:04.19
どのように考えたらこのような図になるのてしょうか?
http://www.imgur.com/OmF8tNG.jpeg
28132人目の素数さん:2013/01/23(水) 15:43:02.24
y≧0の場合とy<0の場合に分けて考える。
29132人目の素数さん:2013/01/23(水) 16:08:26.37
>>28
なるほど!
さんくすです
30132人目の素数さん:2013/01/23(水) 18:40:27.55
∫(0 π) sinx dx



計算過程がわかりまへん
31132人目の素数さん:2013/01/23(水) 19:17:07.19
そりゃわからんわな
32132人目の素数さん:2013/01/23(水) 20:03:35.41
          __ノ)-'´ ̄ ̄`ー- 、_
        , '´  _. -‐'''"二ニニ=-`ヽ、
      /   /:::::; -‐''"        `ーノ
     /   /:::::/           \
     /    /::::::/          | | |  |
     |   |:::::/ /     |  | | | |  |
      |   |::/ / / |  | ||  | | ,ハ .| ,ハ|
      |   |/ / / /| ,ハノ| /|ノレ,ニ|ル' 
     |   |  | / / レ',二、レ′ ,ィイ|゙/   
.     |   \ ∠イ  ,イイ|    ,`-' |      
     |     l^,人|  ` `-'     ゝ  |        このスレは馬と鹿と豚さんばかりね。
      |      ` -'\       ー'  人            
    |        /(l     __/  ヽ、          
     |       (:::::`‐-、__  |::::`、     ヒニニヽ、         
    |      / `‐-、::::::::::`‐-、::::\   /,ニニ、\            
   |      |::::::::::::::::::|` -、:::::::,ヘ ̄|'、  ヒニ二、 \
.   |      /::::::::::::::::::|::::::::\/:::O`、::\   | '、   \
   |      /:::::::::::::::::::/:::::::::::::::::::::::::::::'、::::\ノ  ヽ、  |
  |      |:::::/:::::::::/:::::::::::::::::::::::::::::::::::'、',::::'、  /:\__/‐、
  |      |/:::::::::::/::::::::::::::::::::::::::::::::::O::| '、::| く::::::::::::: ̄|
   |     /_..-'´ ̄`ー-、:::::::::::::::::::::::::::::::::::|/:/`‐'::\;;;;;;;_|
   |    |/::::::::::::::::::::::\:::::::::::::::::::::::::::::|::/::::|::::/:::::::::::/
    |   /:::::::::::::::::::::::::::::::::|:::::::::::::::::::::O::|::|::::::|:::::::::::::::/
33132人目の素数さん:2013/01/23(水) 20:28:59.80
>>30
どーしてこーなった
34132人目の素数さん:2013/01/23(水) 21:00:34.69
sin(x+π/4)は偶関数なのか奇関数なのか分かりません
奇関数?
35132人目の素数さん:2013/01/23(水) 21:09:05.32
>>34
どっちでもない
36132人目の素数さん:2013/01/23(水) 21:55:39.56
そんなぁ
37132人目の素数さん:2013/01/23(水) 22:11:34.94
http://beebee2see.appspot.com/i/azuY9PPcBww.jpg

(2)で何でそうなるのか分かりません。
(1,2)の像Aがl:x+2y=5???
38132人目の素数さん:2013/01/23(水) 22:15:20.85
ちゃんと読め
(1, 2)の像Aが、直線L の上にある って書いてあるんだぞ。
39132人目の素数さん:2013/01/23(水) 22:35:29.96
ぁ、そうですね
その下の条件と今の条件でなぜ、lとmが一致するんですか?
40132人目の素数さん:2013/01/23(水) 22:51:28.21
>>39
作図してみ
41132人目の素数さん:2013/01/23(水) 22:52:25.97
>>39
直線は、それが通る点(1点)と方向ベクトルが決まれば、一意的に確定するから。
42132人目の素数さん:2013/01/23(水) 23:02:39.65
数列{a_n}について、すべての番号nでa_(n+1)-a_n=n^2となっている時のa_nをnで表せ
教えてください
両辺にΣをつけるのかと考えましたがa_(n+1)-a_1=…とa_1がでてきて解けませんでした><
43132人目の素数さん:2013/01/23(水) 23:09:49.71
階差数列
44132人目の素数さん:2013/01/23(水) 23:11:24.48
a_1ないとわかんなくね?
45132人目の素数さん:2013/01/23(水) 23:20:25.33
ルアーだろ
46132人目の素数さん:2013/01/23(水) 23:21:44.33
不定積分
∫1/x^2(x+1)dx
答えが手元になく、部分分数への変換もうまくいきません

お願いします
47132人目の素数さん:2013/01/23(水) 23:22:03.64
積分定数Cと同じで、a_1は任意定数
48132人目の素数さん:2013/01/23(水) 23:24:19.82
>>47
そのまま置いとけばいいんですね有難うございます
49132人目の素数さん:2013/01/23(水) 23:26:59.07
>>46
1/{x^2(x+1)}=1/x^2-1/{x(x+1)}
=1/x^2-1/x+1/(x+1)
50132人目の素数さん:2013/01/23(水) 23:42:06.54
>>49
ありがたまきん
51132人目の素数さん:2013/01/24(木) 01:16:17.84
          __ノ)-'´ ̄ ̄`ー- 、_
        , '´  _. -‐'''"二ニニ=-`ヽ、
      /   /:::::; -‐''"        `ーノ
     /   /:::::/           \
     /    /::::::/          | | |  |
     |   |:::::/ /     |  | | | |  |
      |   |::/ / / |  | ||  | | ,ハ .| ,ハ|
      |   |/ / / /| ,ハノ| /|ノレ,ニ|ル' 
     |   |  | / / レ',二、レ′ ,ィイ|゙/   
.     |   \ ∠イ  ,イイ|    ,`-' |      
     |     l^,人|  ` `-'     ゝ  |        このスレは馬と鹿と豚さんばかりね。
      |      ` -'\       ー'  人            
    |        /(l     __/  ヽ、          
     |       (:::::`‐-、__  |::::`、     ヒニニヽ、         
    |      / `‐-、::::::::::`‐-、::::\   /,ニニ、\            
   |      |::::::::::::::::::|` -、:::::::,ヘ ̄|'、  ヒニ二、 \
.   |      /::::::::::::::::::|::::::::\/:::O`、::\   | '、   \
   |      /:::::::::::::::::::/:::::::::::::::::::::::::::::'、::::\ノ  ヽ、  |
  |      |:::::/:::::::::/:::::::::::::::::::::::::::::::::::'、',::::'、  /:\__/‐、
  |      |/:::::::::::/::::::::::::::::::::::::::::::::::O::| '、::| く::::::::::::: ̄|
   |     /_..-'´ ̄`ー-、:::::::::::::::::::::::::::::::::::|/:/`‐'::\;;;;;;;_|
   |    |/::::::::::::::::::::::\:::::::::::::::::::::::::::::|::/::::|::::/:::::::::::/
    |   /:::::::::::::::::::::::::::::::::|:::::::::::::::::::::O::|::|::::::|:::::::::::::::/
5216:2013/01/24(木) 01:41:22.73
>>19ですが、>>20さんのおっしゃるように具体例を考える以外の証明法はないのでしょうか?
53132人目の素数さん:2013/01/24(木) 01:46:39.30
>>52
3次の多項式関数の場合の主張なんだから、それでいいじゃん。
54348:2013/01/24(木) 04:32:50.40
物分かりが悪くてすいません、三次関数の証明では、一例を挙げれば全てに当てはまる、というのは証明として使ってよいのですか?
5516:2013/01/24(木) 04:33:52.06
すいません、>>54>>16のレスです
56132人目の素数さん:2013/01/24(木) 06:40:08.36
>>54
y=a(x^3-3p^2x)  (p>0)とでも置いたらいいんじゃね
極値をもつ3次関数はこれを平行移動したものだ
5717:2013/01/24(木) 07:36:29.14
直接ガリガリ記述していったらえらいことになってきた

a≠0かつf(x)は極値を持つとき
f(x) = ax^3 + bx^2 + cx + d
⇔ (3√3){f(x)-d-2b^3/(27a^2)+bc/3a}/[a{b^2/(3a^2)-c/a}^(3/2)]
=[(√3)(x+b/3a)/{b^2/(3a^2)-c/a}^(1/2)]^3-(3√3)(x+b/3a)/{b^2/(3a^2)-c/a}^(1/2)

なので
g(x)=(3√3){f(x)-d-2b^3/(27a^2)+bc/3a}/[a{b^2/(3a^2)-c/a}^(3/2)]
t=(√3)(x+b/3a)/{b^2/(3a^2)-c/a}^(1/2)
とおくと

y=f(x)は平行移動と軸方向の拡大縮小反転で
y=g(t)=t^3-3tと変形できることがわかる

(±1,g(±1))は極大極小点、(0,g(0))は変曲点で、g(-2)=g(1)=-2, g(0)=0, g(-1)=g(2)=2
x=[t{(b^2-3ac)/a^2}^(1/2)-b/a]/3
58132人目の素数さん:2013/01/24(木) 08:21:43.67
>>57
もっと頑張れ!
59132人目の素数さん:2013/01/24(木) 10:24:42.06
>>16
f'(x)=k(x-α)(x-β)とおいて f(α)=f(η)となるηを求める。
f(x)-f(α)=k(x-α)^2(x-η)となるので両辺微分して
k(x-α)(x-β)=2k(x-α)(x-η)+k(x-α)^2
これはxについての恒等式なのでx=βの時も成り立つ。
またα≠βであるから0=2k(β-η)+k(β-α)
k≠0よりη=β+(β-α)/2=3β/2-α/2
60132人目の素数さん:2013/01/24(木) 14:22:45.04
>>16
一対一なんかで「三次関数の箱詰め」として紹介されてたような気がする。
この辺りの小ネタは「数学ショートプログラム」という本が詳しい。読むといいと思う。
61132人目の素数さん:2013/01/24(木) 14:36:58.39
単位円上を一定の速さkで動く点Pについて
Pの速度ベクトルvと加速度ベクトルaが垂直な事を示したいのですが

v=(x',y') a=(x'',y'')と置いて条件から(x')^2+(y')^2=k^2の両辺を時間で微分して
x'*x''+y'*y''=0となることは分かったのですが

Pの位置を(cost,sint)と置いて
v=(-sint,cost) a=(-cost,-sint)条件から(sint)^2+(cost)^2=1
となり条件が足りなくなってしまいました

前者の解き方では使っていてと後者の解き方では使っていない条件ってどれにあたりますか?
62132人目の素数さん:2013/01/24(木) 14:41:00.11
>>61
(sint)^2+(cost)^2=1
を時間で微分したら、上と同じなんじゃないの。
63132人目の素数さん:2013/01/24(木) 14:44:43.88
>>61
後者はちゃんとv=(-sint,cost)とa=(-cost,-sint)の内積0になってるじゃん
64132人目の素数さん:2013/01/24(木) 14:46:55.15
>>61
>Pの位置を(cost,sint)と置いて
では
>単位円上を一定の速さkで動く
になってない(tが時間だとすれば)。
65132人目の素数さん:2013/01/24(木) 16:11:51.99
          __ノ)-'´ ̄ ̄`ー- 、_
        , '´  _. -‐'''"二ニニ=-`ヽ、
      /   /:::::; -‐''"        `ーノ
     /   /:::::/           \
     /    /::::::/          | | |  |
     |   |:::::/ /     |  | | | |  |
      |   |::/ / / |  | ||  | | ,ハ .| ,ハ|
      |   |/ / / /| ,ハノ| /|ノレ,ニ|ル' 
     |   |  | / / レ',二、レ′ ,ィイ|゙/   
.     |   \ ∠イ  ,イイ|    ,`-' |      
     |     l^,人|  ` `-'     ゝ  |        このスレは馬と鹿と豚さんばかりね。
      |      ` -'\       ー'  人            
    |        /(l     __/  ヽ、          
     |       (:::::`‐-、__  |::::`、     ヒニニヽ、         
    |      / `‐-、::::::::::`‐-、::::\   /,ニニ、\            
   |      |::::::::::::::::::|` -、:::::::,ヘ ̄|'、  ヒニ二、 \
.   |      /::::::::::::::::::|::::::::\/:::O`、::\   | '、   \
   |      /:::::::::::::::::::/:::::::::::::::::::::::::::::'、::::\ノ  ヽ、  |
  |      |:::::/:::::::::/:::::::::::::::::::::::::::::::::::'、',::::'、  /:\__/‐、
  |      |/:::::::::::/::::::::::::::::::::::::::::::::::O::| '、::| く::::::::::::: ̄|
   |     /_..-'´ ̄`ー-、:::::::::::::::::::::::::::::::::::|/:/`‐'::\;;;;;;;_|
   |    |/::::::::::::::::::::::\:::::::::::::::::::::::::::::|::/::::|::::/:::::::::::/
    |   /:::::::::::::::::::::::::::::::::|:::::::::::::::::::::O::|::|::::::|:::::::::::::::/
66132人目の素数さん:2013/01/24(木) 19:24:02.72
>>62>>63>>64
有り難うございます

では問題が円の代わりで楕円x^2/A^2 + y^2/B~2 = 1上をPが動き
後者の解き方で考えた時
Pの位置を(Acost,Bsint)と置いて
v=(-Asint,Bcost) a=(-Acost,-Bsint)
速さが定数の条件から
(Asint)^2+(Bcost)^2=k
これらを使って

v↑*a↑=A^2sint*cost-B^2sintcost=0
を示せないのは何故なんでしょう?もしくは式変形をうまくすれば示すことはできるのでしょうか
67あのこうちやんは始皇帝だった:2013/01/24(木) 19:26:09.89
 テメ〜ら、いいかげんにしねえと、ブッ殺すぞ!

 20代と30代の、ニート・無職の、知的障害の、女性恐怖症の、頭デッカチの虚弱児・ひ弱の、ゴミ・クズ・カス・無能・虫けらのクソガキども!

 死ね!!!!!!!!!!!!!!!!!!!!!!
68132人目の素数さん:2013/01/24(木) 19:29:03.31
          __ノ)-'´ ̄ ̄`ー- 、_
        , '´  _. -‐'''"二ニニ=-`ヽ、
      /   /:::::; -‐''"        `ーノ
     /   /:::::/           \
     /    /::::::/          | | |  |
     |   |:::::/ /     |  | | | |  |
      |   |::/ / / |  | ||  | | ,ハ .| ,ハ|
      |   |/ / / /| ,ハノ| /|ノレ,ニ|ル' 
     |   |  | / / レ',二、レ′ ,ィイ|゙/   
.     |   \ ∠イ  ,イイ|    ,`-' |      
     |     l^,人|  ` `-'     ゝ  |        このスレは馬と鹿と豚さんばかりね。
      |      ` -'\       ー'  人            
    |        /(l     __/  ヽ、          
     |       (:::::`‐-、__  |::::`、     ヒニニヽ、         
    |      / `‐-、::::::::::`‐-、::::\   /,ニニ、\            
   |      |::::::::::::::::::|` -、:::::::,ヘ ̄|'、  ヒニ二、 \
.   |      /::::::::::::::::::|::::::::\/:::O`、::\   | '、   \
   |      /:::::::::::::::::::/:::::::::::::::::::::::::::::'、::::\ノ  ヽ、  |
  |      |:::::/:::::::::/:::::::::::::::::::::::::::::::::::'、',::::'、  /:\__/‐、
  |      |/:::::::::::/::::::::::::::::::::::::::::::::::O::| '、::| く::::::::::::: ̄|
   |     /_..-'´ ̄`ー-、:::::::::::::::::::::::::::::::::::|/:/`‐'::\;;;;;;;_|
   |    |/::::::::::::::::::::::\:::::::::::::::::::::::::::::|::/::::|::::/:::::::::::/
    |   /:::::::::::::::::::::::::::::::::|:::::::::::::::::::::O::|::|::::::|:::::::::::::::/
69132人目の素数さん:2013/01/24(木) 19:34:11.95
何が疑問なのかわからん

そもそもだ円上の等速運動で速度と加速度が垂直なんて一般に言えんだろ
70132人目の素数さん:2013/01/24(木) 19:39:19.04
          __ノ)-'´ ̄ ̄`ー- 、_
        , '´  _. -‐'''"二ニニ=-`ヽ、
      /   /:::::; -‐''"        `ーノ
     /   /:::::/           \
     /    /::::::/          | | |  |
     |   |:::::/ /     |  | | | |  |
      |   |::/ / / |  | ||  | | ,ハ .| ,ハ|
      |   |/ / / /| ,ハノ| /|ノレ,ニ|ル' 
     |   |  | / / レ',二、レ′ ,ィイ|゙/   
.     |   \ ∠イ  ,イイ|    ,`-' |      
     |     l^,人|  ` `-'     ゝ  |        このスレは馬と鹿と豚さんばかりね。
      |      ` -'\       ー'  人            
    |        /(l     __/  ヽ、          
     |       (:::::`‐-、__  |::::`、     ヒニニヽ、         
    |      / `‐-、::::::::::`‐-、::::\   /,ニニ、\            
   |      |::::::::::::::::::|` -、:::::::,ヘ ̄|'、  ヒニ二、 \
.   |      /::::::::::::::::::|::::::::\/:::O`、::\   | '、   \
   |      /:::::::::::::::::::/:::::::::::::::::::::::::::::'、::::\ノ  ヽ、  |
  |      |:::::/:::::::::/:::::::::::::::::::::::::::::::::::'、',::::'、  /:\__/‐、
  |      |/:::::::::::/::::::::::::::::::::::::::::::::::O::| '、::| く::::::::::::: ̄|
   |     /_..-'´ ̄`ー-、:::::::::::::::::::::::::::::::::::|/:/`‐'::\;;;;;;;_|
   |    |/::::::::::::::::::::::\:::::::::::::::::::::::::::::|::/::::|::::/:::::::::::/
    |   /:::::::::::::::::::::::::::::::::|:::::::::::::::::::::O::|::|::::::|:::::::::::::::/
71132人目の素数さん:2013/01/24(木) 19:40:37.58
>>69
(Asint)^2+(Bcost)^2=k @この式を両辺tで微分したら
2Asintcost-2Bcost=0
⇔A^2sint*cost-B^2sintcost=0
を導けるのですが・・


@の式を微分したらできるのに
媒介変数で(Acost,Bsint)とわけてそれぞれ微分したら
うまく導けないのはどうしてなのかと疑問に思いまして
72132人目の素数さん:2013/01/24(木) 19:52:18.70
ますます何がしたいのかわからん

何が仮定で
何を結論したいのか
ちょっと書いてみろ
73132人目の素数さん:2013/01/24(木) 19:54:10.71
>(Asint)^2+(Bcost)^2=k @この式を両辺tで微分したら
>2Asintcost-2Bcost=0
>⇔A^2sint*cost-B^2sintcost=0
>を導けるのですが・・



なにを導いたつもりなのかわからねぇ
74132人目の素数さん:2013/01/24(木) 19:59:07.36
>>69
>そもそもだ円上の等速運動で速度と加速度が垂直なんて一般に言えんだろ

そんなことない
75132人目の素数さん:2013/01/24(木) 20:00:38.74
>>66
だ円の周上を等速運動する点Pの位置は (Acost, Bsint) とはおけない。
76132人目の素数さん:2013/01/24(木) 20:06:18.23
すいません
仮定 楕円x^2/A^2 + y^2/B~2 = 1上 を一定の速さkで動く点Pがある
結論 Pの速度ベクトルvと加速度ベクトルaが垂直


Pの位置を(Acost,Bsint)とおいて

一つ目の解法
速度一定の条件から(Asint)^2+(Bcost)^2=k
2Asintcost-2Bcost=0
⇔A^2sint*cost-B^2sintcost=0
↑これは速度ベクトルと加速度ベクトルの内積になっていて0なので垂直ということをしめせた

2つ目の解法
Pの位置を(Acost,Bsint)これをtで微分して
v=(-Asint,Bcost) a=(-Acost,-Bsint)
速度一定の条件から
(Asint)^2+(Bcost)^2=k

速度ベクトルと加速度ベクトルの内積が0であることをしめしたいができない
何故なのでしょうか?
77132人目の素数さん:2013/01/24(木) 20:08:09.50
>>75
何故ですか?
円でおいてx軸y軸各方向に拡大縮小したとかんがえたのですが
78132人目の素数さん:2013/01/24(木) 20:08:16.72
>>76
> だ円の周上を等速運動する点Pの位置は (Acost, Bsint) とはおけない。
79132人目の素数さん:2013/01/24(木) 20:12:12.75
>>77
時刻tにおけるPの位置を(Acost, Bsint)と置くとその時の速度は
(d/dt(Acost),d/dt(Bsint))=(-Asint,Bcost)
その絶対値は
√(A^2sin^2(t)+B^2cos^2(t))
となりtによって変化する
よって速さは一定ではなく、等速運動ではない
80132人目の素数さん:2013/01/24(木) 20:13:58.38
>>76
>(Asint)^2+(Bcost)^2
t=0のときB^2, t=π/4のとき(A^2+B^2)/2 , t=π/2のときA^2
81132人目の素数さん:2013/01/24(木) 20:15:19.60
>>79
なるほど
いわれてみれば膝を叩きたくなりました
何で自分式弄ってる最中に気が付かなかったんでしょう・・

ではこの場合速度ベクトルを(x’、y’)とおくしかないということなんでしょうか?
82132人目の素数さん:2013/01/24(木) 20:18:13.56
>>77
(cost, sint) は円周上の等速運動をする確かに。
しかし、それをx軸方向とy軸方向で 異 な る 比率 で拡大したら、もはや等速にならんだろ。

(2cost, sint)という点だったら、点(2,0)付近で遅く、点(0,1)付近で速く動きそうに思えんか?
一周する時間が変わらんわけだから、x軸方向に伸びた分、x方向の移動ペースが速くなる。
83132人目の素数さん:2013/01/24(木) 20:21:37.67
そもそも「速さ一定なら速度⊥加速度」てのは軌道の形によらないわけだが(微分可能な曲線であれば)。
84132人目の素数さん:2013/01/24(木) 20:22:53.73
>>82
仰るとおりです

>>83
そうなのですか?
なんだか不思議な性質ですね




みなさん詳しく説明有り難うございました
85132人目の素数さん:2013/01/24(木) 20:31:07.67
2次元なら
v↑=(dx/dt, dy/dt) …(i)
a↑=(d^2x/dt^2, d^2y/dt^2) …(ii)
|v↑|^2=(dx/dt)^2+(dy/dt)^2 = 定数…(iii)
(i),(ii) と (iii)を両辺微分してv↑・a↑=0が得られる

それと>>66が問題なのは角速度を一定にしたことだから、
これををω(t)みたいな形でおいてやればいい
86132人目の素数さん:2013/01/24(木) 20:43:52.04
          __ノ)-'´ ̄ ̄`ー- 、_
        , '´  _. -‐'''"二ニニ=-`ヽ、
      /   /:::::; -‐''"        `ーノ
     /   /:::::/           \
     /    /::::::/          | | |  |
     |   |:::::/ /     |  | | | |  |
      |   |::/ / / |  | ||  | | ,ハ .| ,ハ|
      |   |/ / / /| ,ハノ| /|ノレ,ニ|ル' 
     |   |  | / / レ',二、レ′ ,ィイ|゙/   
.     |   \ ∠イ  ,イイ|    ,`-' |      
     |     l^,人|  ` `-'     ゝ  |        このスレは馬と鹿と豚さんばかりね。
      |      ` -'\       ー'  人            
    |        /(l     __/  ヽ、          
     |       (:::::`‐-、__  |::::`、     ヒニニヽ、         
    |      / `‐-、::::::::::`‐-、::::\   /,ニニ、\            
   |      |::::::::::::::::::|` -、:::::::,ヘ ̄|'、  ヒニ二、 \
.   |      /::::::::::::::::::|::::::::\/:::O`、::\   | '、   \
   |      /:::::::::::::::::::/:::::::::::::::::::::::::::::'、::::\ノ  ヽ、  |
  |      |:::::/:::::::::/:::::::::::::::::::::::::::::::::::'、',::::'、  /:\__/‐、
  |      |/:::::::::::/::::::::::::::::::::::::::::::::::O::| '、::| く::::::::::::: ̄|
   |     /_..-'´ ̄`ー-、:::::::::::::::::::::::::::::::::::|/:/`‐'::\;;;;;;;_|
   |    |/::::::::::::::::::::::\:::::::::::::::::::::::::::::|::/::::|::::/:::::::::::/
    |   /:::::::::::::::::::::::::::::::::|:::::::::::::::::::::O::|::|::::::|:::::::::::::::/
87132人目の素数さん:2013/01/24(木) 21:05:33.56
f(x)=2x^2-6x+1
極値を求めよ。

教えてください
88132人目の素数さん:2013/01/24(木) 21:09:46.97
極値ってなにか分かっていれば絶対解けるはず。
極値がそもそも分かっていなければ解けるはずないから、まずその意味を調べてからだ。
89132人目の素数さん:2013/01/24(木) 21:25:22.74
このスレ見てからEnglish板の中高生質問スレにいったらここのスレの優しさを改めて認識しました
皆様いつも御世話になっております
ありがとうございます
90132人目の素数さん:2013/01/24(木) 23:27:57.58
1+log{1/2}(x-3)(x-6)>log{1/2}(x)を解け

真数条件より、x<3,6<x,x>0,つまり0<x<3
このとき、
1+log{1/2}(x-3)(x-6)>log{1/2}(x)
log{1/2}(1/2)+log{1/2}(x-3)(x-6)>log{1/2}(x)
底1/2<0より、
(1/2)+(x^2-9x+54)<x
2x^2-20x+109<0

としたのですが、2x^2-20x+109<0では実数解がありません。
式変形のどこかで間違っているはずなのですがわからないです。
どこがいけないんでしょうか?
91132人目の素数さん:2013/01/24(木) 23:33:12.54
>>90
ここ >(1/2)+(x^2-9x+54)<x
92132人目の素数さん:2013/01/24(木) 23:34:15.05
> (1/2)+(x^2-9x+54)<x
↑ここ、掛け算だよ
93792:2013/01/24(木) 23:44:17.08
>>91-92
ああ!足し算は掛け算になるんでしたね!
即レスありがとうごさいました!
9416:2013/01/25(金) 00:22:12.44
>>56-60
ありがとうございます!
正直>>57さんの四行目あたりからは書いても何が起こっているのか理解が難しいのですが(理解力が乏しくて申し訳ないです。また考え直します)、>>59さんの方法で納得できました。
皆さんありがとうございました。数学のショートプログラム、という本は興味が湧いたので購入しようと思います
勉強になりましたm(__)m
9557:2013/01/25(金) 00:51:03.19
>>94
いやまあ>>59が一番いいよ、うん
96132人目の素数さん:2013/01/25(金) 01:01:37.98
          __ノ)-'´ ̄ ̄`ー- 、_
        , '´  _. -‐'''"二ニニ=-`ヽ、
      /   /:::::; -‐''"        `ーノ
     /   /:::::/           \
     /    /::::::/          | | |  |
     |   |:::::/ /     |  | | | |  |
      |   |::/ / / |  | ||  | | ,ハ .| ,ハ|
      |   |/ / / /| ,ハノ| /|ノレ,ニ|ル' 
     |   |  | / / レ',二、レ′ ,ィイ|゙/   
.     |   \ ∠イ  ,イイ|    ,`-' |      
     |     l^,人|  ` `-'     ゝ  |        このスレは馬と鹿と豚さんばかりね。
      |      ` -'\       ー'  人            
    |        /(l     __/  ヽ、          
     |       (:::::`‐-、__  |::::`、     ヒニニヽ、         
    |      / `‐-、::::::::::`‐-、::::\   /,ニニ、\            
   |      |::::::::::::::::::|` -、:::::::,ヘ ̄|'、  ヒニ二、 \
.   |      /::::::::::::::::::|::::::::\/:::O`、::\   | '、   \
   |      /:::::::::::::::::::/:::::::::::::::::::::::::::::'、::::\ノ  ヽ、  |
  |      |:::::/:::::::::/:::::::::::::::::::::::::::::::::::'、',::::'、  /:\__/‐、
  |      |/:::::::::::/::::::::::::::::::::::::::::::::::O::| '、::| く::::::::::::: ̄|
   |     /_..-'´ ̄`ー-、:::::::::::::::::::::::::::::::::::|/:/`‐'::\;;;;;;;_|
   |    |/::::::::::::::::::::::\:::::::::::::::::::::::::::::|::/::::|::::/:::::::::::/
    |   /:::::::::::::::::::::::::::::::::|:::::::::::::::::::::O::|::|::::::|:::::::::::::::/
97132人目の素数さん:2013/01/25(金) 08:03:08.09
>>84
加速度の進行方向成分が0じゃなかったら速さが変わっちゃうだろ?
98132人目の素数さん:2013/01/25(金) 09:44:23.14
>>95
>>59でちょっと訂正
f’(x)の係数をk→3kに
結果は変わらんけど
99132人目の素数さん:2013/01/25(金) 10:01:09.93
          __ノ)-'´ ̄ ̄`ー- 、_
        , '´  _. -‐'''"二ニニ=-`ヽ、
      /   /:::::; -‐''"        `ーノ
     /   /:::::/           \
     /    /::::::/          | | |  |
     |   |:::::/ /     |  | | | |  |
      |   |::/ / / |  | ||  | | ,ハ .| ,ハ|
      |   |/ / / /| ,ハノ| /|ノレ,ニ|ル' 
     |   |  | / / レ',二、レ′ ,ィイ|゙/   
.     |   \ ∠イ  ,イイ|    ,`-' |      
     |     l^,人|  ` `-'     ゝ  |        このスレは馬と鹿と豚さんばかりね。
      |      ` -'\       ー'  人            
    |        /(l     __/  ヽ、          
     |       (:::::`‐-、__  |::::`、     ヒニニヽ、         
    |      / `‐-、::::::::::`‐-、::::\   /,ニニ、\            
   |      |::::::::::::::::::|` -、:::::::,ヘ ̄|'、  ヒニ二、 \
.   |      /::::::::::::::::::|::::::::\/:::O`、::\   | '、   \
   |      /:::::::::::::::::::/:::::::::::::::::::::::::::::'、::::\ノ  ヽ、  |
  |      |:::::/:::::::::/:::::::::::::::::::::::::::::::::::'、',::::'、  /:\__/‐、
  |      |/:::::::::::/::::::::::::::::::::::::::::::::::O::| '、::| く::::::::::::: ̄|
   |     /_..-'´ ̄`ー-、:::::::::::::::::::::::::::::::::::|/:/`‐'::\;;;;;;;_|
   |    |/::::::::::::::::::::::\:::::::::::::::::::::::::::::|::/::::|::::/:::::::::::/
    |   /:::::::::::::::::::::::::::::::::|:::::::::::::::::::::O::|::|::::::|:::::::::::::::/
100132人目の素数さん:2013/01/25(金) 11:57:06.97
>>97
なるほとです!
101132人目の素数さん:2013/01/25(金) 11:59:06.48
          __ノ)-'´ ̄ ̄`ー- 、_
        , '´  _. -‐'''"二ニニ=-`ヽ、
      /   /:::::; -‐''"        `ーノ
     /   /:::::/           \
     /    /::::::/          | | |  |
     |   |:::::/ /     |  | | | |  |
      |   |::/ / / |  | ||  | | ,ハ .| ,ハ|
      |   |/ / / /| ,ハノ| /|ノレ,ニ|ル' 
     |   |  | / / レ',二、レ′ ,ィイ|゙/   
.     |   \ ∠イ  ,イイ|    ,`-' |      
     |     l^,人|  ` `-'     ゝ  |        このスレは馬と鹿と豚さんばかりね。
      |      ` -'\       ー'  人            
    |        /(l     __/  ヽ、          
     |       (:::::`‐-、__  |::::`、     ヒニニヽ、         
    |      / `‐-、::::::::::`‐-、::::\   /,ニニ、\            
   |      |::::::::::::::::::|` -、:::::::,ヘ ̄|'、  ヒニ二、 \
.   |      /::::::::::::::::::|::::::::\/:::O`、::\   | '、   \
   |      /:::::::::::::::::::/:::::::::::::::::::::::::::::'、::::\ノ  ヽ、  |
  |      |:::::/:::::::::/:::::::::::::::::::::::::::::::::::'、',::::'、  /:\__/‐、
  |      |/:::::::::::/::::::::::::::::::::::::::::::::::O::| '、::| く::::::::::::: ̄|
   |     /_..-'´ ̄`ー-、:::::::::::::::::::::::::::::::::::|/:/`‐'::\;;;;;;;_|
   |    |/::::::::::::::::::::::\:::::::::::::::::::::::::::::|::/::::|::::/:::::::::::/
    |   /:::::::::::::::::::::::::::::::::|:::::::::::::::::::::O::|::|::::::|:::::::::::::::/
102132人目の素数さん:2013/01/25(金) 18:12:04.41
U_[n]=8(8^(n)-1)/21とするときlim[n→∞]U_[n-1]/8^nを求めよ、で
lim[n→∞]U_[n-1]/8^n=lim[n→∞]U_[n]/8^n
としたら左辺と右辺で値が違うのですが何故ですか
103132人目の素数さん:2013/01/25(金) 18:50:46.79
例えば 16÷8 と 16 は違うだろ。これと同じことだ。
104132人目の素数さん:2013/01/25(金) 19:01:58.55
10コの玉の中にあたりが3つある
この中から同時に4つ取り出すとき、あたりを2コだけひく確率を求めよ

4C2/10C4 だと思ったんですけど違うんでしょうか・・?
105132人目の素数さん:2013/01/25(金) 19:12:38.71
4つ引いて、当たりが2つということは、外れも2つ引くということだから
求める確率は ( C[4,2]×C[6,2] ) / C[10,4] としなくては。
106132人目の素数さん:2013/01/25(金) 19:14:06.30
なるほど ハズレも数えないとダメなんですね!
ありがとうございました
107132人目の素数さん:2013/01/25(金) 19:15:07.46
>>104
ハズレを2個だけ引く確率と考えると6C2/10C4になっちゃうぞ。同じはずなのに。
108132人目の素数さん:2013/01/25(金) 19:17:31.01
>10コの玉の中にあたりが3つある
109105:2013/01/25(金) 19:19:06.14
しまったツラレテだまされた。

>>105
誤 求める確率は ( C[4,2]×C[6,2] ) / C[10,4] としなくては。
性 求める確率は ( C[3,2]×C[7,2] ) / C[10,4] としなくては。
に訂正。
110132人目の素数さん:2013/01/25(金) 20:35:54.87
111132人目の素数さん:2013/01/25(金) 22:08:23.41
落ち着いてください
112132人目の素数さん:2013/01/25(金) 22:51:38.80
rearrangement inequalityを証明なしに使うのはまずいでしょうか
113132人目の素数さん:2013/01/25(金) 23:00:38.41
こんばんは
質問させてください。
大学の入試問題です。

4x^3y -4xy^3 -2x^2 +2y^2 +4xy -1

この式を因数分解し

(?x^2-?x+?)(?y^2+?y-?)

の形にせよ。という問題です。

選択式の問題でして、?には
@:1 A:2 B:3 C:4 D:xy
E:x F:y G:2x H:2y I:0

のいずれか(重複あり)が入ります。


最後に定数項"-1"があるため、因数分をした式の最後の二つに1が入る事は類推できました。
ですが、選択肢に"xy"などがあるため非常に考えにくく、そこからの因数分解がどうしても思いつきません...。

解き方、手順などを教えてください。協力お願いしますm(_ _)m

長文すいません!
114132人目の素数さん:2013/01/25(金) 23:06:03.98
類推ってなんだよw
115132人目の素数さん:2013/01/25(金) 23:10:22.03
          __ノ)-'´ ̄ ̄`ー- 、_
        , '´  _. -‐'''"二ニニ=-`ヽ、
      /   /:::::; -‐''"        `ーノ
     /   /:::::/           \
     /    /::::::/          | | |  |
     |   |:::::/ /     |  | | | |  |
      |   |::/ / / |  | ||  | | ,ハ .| ,ハ|
      |   |/ / / /| ,ハノ| /|ノレ,ニ|ル' 
     |   |  | / / レ',二、レ′ ,ィイ|゙/   
.     |   \ ∠イ  ,イイ|    ,`-' |      
     |     l^,人|  ` `-'     ゝ  |        このスレは馬と鹿と豚さんばかりね。
      |      ` -'\       ー'  人            
    |        /(l     __/  ヽ、          
     |       (:::::`‐-、__  |::::`、     ヒニニヽ、         
    |      / `‐-、::::::::::`‐-、::::\   /,ニニ、\            
   |      |::::::::::::::::::|` -、:::::::,ヘ ̄|'、  ヒニ二、 \
.   |      /::::::::::::::::::|::::::::\/:::O`、::\   | '、   \
   |      /:::::::::::::::::::/:::::::::::::::::::::::::::::'、::::\ノ  ヽ、  |
  |      |:::::/:::::::::/:::::::::::::::::::::::::::::::::::'、',::::'、  /:\__/‐、
  |      |/:::::::::::/::::::::::::::::::::::::::::::::::O::| '、::| く::::::::::::: ̄|
   |     /_..-'´ ̄`ー-、:::::::::::::::::::::::::::::::::::|/:/`‐'::\;;;;;;;_|
   |    |/::::::::::::::::::::::\:::::::::::::::::::::::::::::|::/::::|::::/:::::::::::/
    |   /:::::::::::::::::::::::::::::::::|:::::::::::::::::::::O::|::|::::::|:::::::::::::::/
116132人目の素数さん:2013/01/25(金) 23:13:56.41
>>113
そのまんま未知の係数を文字でおいて展開して係数比較
117132人目の素数さん:2013/01/25(金) 23:16:45.83
>>114
整数係数と仮定すれば推測できるだろ
間違っていれば推測を破棄すればいいだけだし、受験数学のテクニック
118132人目の素数さん:2013/01/25(金) 23:35:25.58
>>116,117
ありがとうございます。

やっぱり文字でおいて解くのが普通でしょうか(´・ω・`)?
未知の係数をそれぞれa~fでおいて計算して、恒等式(?)のようにして計算してみたのですが、
選択肢にxyがあるおかげで何パターンか出来てしまうようで...思うように進まないんですよね...。

うまく整理が出来てないのでしょうか?
119132人目の素数さん:2013/01/25(金) 23:37:00.54
ヒント:マイナス
120132人目の素数さん:2013/01/26(土) 00:04:23.02
>>119

...あ、分かったかもしれないです(´・ω・`)!

x^2y^2の項を消す手順が大間違いでした...
上手く埋まるところから係数を埋めていったらなんとかなりそうです!

すいませんありがとうございましたm(_ _)m
121132人目の素数さん:2013/01/26(土) 02:13:03.86
正規分布描画で質問です。
モデル図作成のため、幾つかの標準偏差(10,15,20,25,30)を持つグラフを描く
方法を探しています。
平均は50で、よくあるテスト点数の0〜100点で良いです。

Excelでは数字を入れてグラフと標準偏差を得ていますが、この逆の方法はありますか?
宜しく願います。
※Excelスレで聞いた方が良ければ移動します。
122132人目の素数さん:2013/01/26(土) 02:27:47.35
>>113
これ面白いな。
最低次の文字で纏める、
対称項に注目する、
方程式の解を利用する、
といったありきたりの方法が通用しない。
123121:2013/01/26(土) 02:37:56.65
すみません、追加質問です。
母集団が1000人で平均が50点の標準偏差20の場合、平均前後で±5%内を示す
偏差値ってどうやって求めるのでしょうか。
※例えば偏差値48〜52とか
ググって数式を探していますが見つかりません。お願いします。
124132人目の素数さん:2013/01/26(土) 02:55:20.18
          __ノ)-'´ ̄ ̄`ー- 、_
        , '´  _. -‐'''"二ニニ=-`ヽ、
      /   /:::::; -‐''"        `ーノ
     /   /:::::/           \
     /    /::::::/          | | |  |
     |   |:::::/ /     |  | | | |  |
      |   |::/ / / |  | ||  | | ,ハ .| ,ハ|
      |   |/ / / /| ,ハノ| /|ノレ,ニ|ル' 
     |   |  | / / レ',二、レ′ ,ィイ|゙/   
.     |   \ ∠イ  ,イイ|    ,`-' |      
     |     l^,人|  ` `-'     ゝ  |        このスレは馬と鹿と豚さんばかりね。
      |      ` -'\       ー'  人            
    |        /(l     __/  ヽ、          
     |       (:::::`‐-、__  |::::`、     ヒニニヽ、         
    |      / `‐-、::::::::::`‐-、::::\   /,ニニ、\            
   |      |::::::::::::::::::|` -、:::::::,ヘ ̄|'、  ヒニ二、 \
.   |      /::::::::::::::::::|::::::::\/:::O`、::\   | '、   \
   |      /:::::::::::::::::::/:::::::::::::::::::::::::::::'、::::\ノ  ヽ、  |
  |      |:::::/:::::::::/:::::::::::::::::::::::::::::::::::'、',::::'、  /:\__/‐、
  |      |/:::::::::::/::::::::::::::::::::::::::::::::::O::| '、::| く::::::::::::: ̄|
   |     /_..-'´ ̄`ー-、:::::::::::::::::::::::::::::::::::|/:/`‐'::\;;;;;;;_|
   |    |/::::::::::::::::::::::\:::::::::::::::::::::::::::::|::/::::|::::/:::::::::::/
    |   /:::::::::::::::::::::::::::::::::|:::::::::::::::::::::O::|::|::::::|:::::::::::::::/
125132人目の素数さん:2013/01/26(土) 03:10:39.12
>>121
> 数字を入れて
何(どういう量)に数字を入れるんだ
126113:2013/01/26(土) 03:19:17.89
>>122
大学入試で使うであろう思いつく解法はだいたい試したのですが駄目でした...
係数を文字でおいて、出てきたx^2y^2を上手く消すように文字に入る数字を調整するという
考え方で解いてみました。(合っているかは分かりませんが...)

あんまり見ない問題なので面白いですよねw
127132人目の素数さん:2013/01/26(土) 03:46:48.50
数学1の問題で質問なんですが、三角形ABCの中心をIとし直線AIと辺BCの交点をDとする。
AB=6 BC=5 CA=3であるときのAI:IDを求める問題なんですが、
解答にBD:DC=AB:AC:=6:3=2:1
となっており いきなり
BD=2/2+1BCとなってるんですが、これの意味がわからないんです
128132人目の素数さん:2013/01/26(土) 03:51:14.39
内心、外心、垂心、重心、(傍心)など三角形の中心は山ほどあるわけだが
どの中心かを書かなきゃ答えが定まるわけねえだろ
129132人目の素数さん:2013/01/26(土) 03:52:38.66
>>127
BD:DC=2:1
130132人目の素数さん:2013/01/26(土) 03:59:57.32
>>128
内心です

>>129
それはわかるんですがなぜ分数がでてくるんですか?
131132人目の素数さん:2013/01/26(土) 04:00:42.22
>>130
図を描けばわかる
132132人目の素数さん:2013/01/26(土) 10:17:12.06
          __ノ)-'´ ̄ ̄`ー- 、_
        , '´  _. -‐'''"二ニニ=-`ヽ、
      /   /:::::; -‐''"        `ーノ
     /   /:::::/           \
     /    /::::::/          | | |  |
     |   |:::::/ /     |  | | | |  |
      |   |::/ / / |  | ||  | | ,ハ .| ,ハ|
      |   |/ / / /| ,ハノ| /|ノレ,ニ|ル' 
     |   |  | / / レ',二、レ′ ,ィイ|゙/   私は只の数ヲタなんかとは付き合わないわ。
.     |   \ ∠イ  ,イイ|    ,`-' |      頭が良くて数学が出来てかっこいい人。それが必要条件よ。
     |     l^,人|  ` `-'     ゝ  |        さらに Ann.of Math に論文書けば十分条件にもなるわよ。
      |      ` -'\       ー'  人          一番嫌いなのは論文数を増やすためにくだらない論文を書いて
    |        /(l     __/  ヽ、           良い論文の出版を遅らせるお馬鹿な人。
     |       (:::::`‐-、__  |::::`、     ヒニニヽ、         あなたの論文が Ann of Math に accept される確率は?
    |      / `‐-、::::::::::`‐-、::::\   /,ニニ、\            それとも最近は Inv. Math. の方が上かしら?
   |      |::::::::::::::::::|` -、:::::::,ヘ ̄|'、  ヒニ二、 \
.   |      /::::::::::::::::::|::::::::\/:::O`、::\   | '、   \
   |      /:::::::::::::::::::/:::::::::::::::::::::::::::::'、::::\ノ  ヽ、  |
  |      |:::::/:::::::::/:::::::::::::::::::::::::::::::::::'、',::::'、  /:\__/‐、
  |      |/:::::::::::/::::::::::::::::::::::::::::::::::O::| '、::| く::::::::::::: ̄|
   |     /_..-'´ ̄`ー-、:::::::::::::::::::::::::::::::::::|/:/`‐'::\;;;;;;;_|
   |    |/::::::::::::::::::::::\:::::::::::::::::::::::::::::|::/::::|::::/:::::::::::/
133132人目の素数さん:2013/01/26(土) 11:34:25.00
(1)なぜこんな場合分けになるんでしょうか?
134132人目の素数さん:2013/01/26(土) 11:35:11.97
135132人目の素数さん:2013/01/26(土) 11:42:58.22
関数v(t)
(dv/dt)+v+2=0の微分方程式を解くとき、最初何をすればいいのでしょうか?
136132人目の素数さん:2013/01/26(土) 11:44:33.27
>>135
変数分離
137132人目の素数さん:2013/01/26(土) 11:48:40.76
>>135
言い方悪かったかも
{1/(v+2)}dv/dt=-1
と変形して両辺tで積分
138132人目の素数さん:2013/01/26(土) 11:51:08.50
          __ノ)-'´ ̄ ̄`ー- 、_
        , '´  _. -‐'''"二ニニ=-`ヽ、
      /   /:::::; -‐''"        `ーノ
     /   /:::::/           \
     /    /::::::/          | | |  |
     |   |:::::/ /     |  | | | |  |
      |   |::/ / / |  | ||  | | ,ハ .| ,ハ|
      |   |/ / / /| ,ハノ| /|ノレ,ニ|ル' 
     |   |  | / / レ',二、レ′ ,ィイ|゙/   
.     |   \ ∠イ  ,イイ|    ,`-' |      
     |     l^,人|  ` `-'     ゝ  |        このスレは馬と鹿と豚さんばかりね。
      |      ` -'\       ー'  人            
    |        /(l     __/  ヽ、          
     |       (:::::`‐-、__  |::::`、     ヒニニヽ、         
    |      / `‐-、::::::::::`‐-、::::\   /,ニニ、\            
   |      |::::::::::::::::::|` -、:::::::,ヘ ̄|'、  ヒニ二、 \
.   |      /::::::::::::::::::|::::::::\/:::O`、::\   | '、   \
   |      /:::::::::::::::::::/:::::::::::::::::::::::::::::'、::::\ノ  ヽ、  |
  |      |:::::/:::::::::/:::::::::::::::::::::::::::::::::::'、',::::'、  /:\__/‐、
  |      |/:::::::::::/::::::::::::::::::::::::::::::::::O::| '、::| く::::::::::::: ̄|
   |     /_..-'´ ̄`ー-、:::::::::::::::::::::::::::::::::::|/:/`‐'::\;;;;;;;_|
   |    |/::::::::::::::::::::::\:::::::::::::::::::::::::::::|::/::::|::::/:::::::::::/
    |   /:::::::::::::::::::::::::::::::::|:::::::::::::::::::::O::|::|::::::|:::::::::::::::/
139132人目の素数さん:2013/01/26(土) 11:56:30.76
>>137
ありがとうございます!
やってみます。
140132人目の素数さん:2013/01/26(土) 12:02:24.74
>>137
log(v+2)=-t+C (C:定数)でいいですか?
初期条件として、v(0)=1がありますがどうすればいいでしょうか?
141132人目の素数さん:2013/01/26(土) 12:14:44.13
log|v+2|=-t+C
⇔v+2=±exp(-t+C)=±exp(C)&bull;exp(-t)
±exp(C)=Aとおいて整理すると
v=Aexp(-t) (Aは任意定数)
あとは初期条件を代入してAを求めておしまい
142132人目の素数さん:2013/01/26(土) 12:15:44.05
2行目訂正
v+2=±exp(-t+C)=±exp(C)×exp(-t)
143132人目の素数さん:2013/01/26(土) 12:16:36.68
間違いだらけで死にたい
log|v+2|=-t+C
⇔v+2=±exp(-t+C)=±exp(C)×exp(-t)
±exp(C)=Aとおいて整理すると
v=Aexp(-t)-2 (Aは任意定数)
あとは初期条件を代入してAを求める
144132人目の素数さん:2013/01/26(土) 12:20:06.46
ありがとうございました。
145132人目の素数さん:2013/01/26(土) 14:13:56.59
http://beebee2see.appspot.com/i/azuYyMHdBww.jpg
http://beebee2see.appspot.com/i/azuYoYTdBww.jpg

1枚目の後半
上の結果からもわかるように、『点(1,1)が動かないとは、y=x+1の方向ベクトル(1,1)が変わらないことを意味します。』すると問題は、y=x+kの点、たとえば(0,k)がどうなるか?だけです。

この『 』部分が分かりません。
その後もよく分かりません。
なぜ、このようなことが言えるのですか?
146132人目の素数さん:2013/01/26(土) 14:45:25.05
          __ノ)-'´ ̄ ̄`ー- 、_
        , '´  _. -‐'''"二ニニ=-`ヽ、
      /   /:::::; -‐''"        `ーノ
     /   /:::::/           \
     /    /::::::/          | | |  |
     |   |:::::/ /     |  | | | |  |
      |   |::/ / / |  | ||  | | ,ハ .| ,ハ|
      |   |/ / / /| ,ハノ| /|ノレ,ニ|ル' 
     |   |  | / / レ',二、レ′ ,ィイ|゙/   
.     |   \ ∠イ  ,イイ|    ,`-' |      
     |     l^,人|  ` `-'     ゝ  |        このスレは馬と鹿と豚さんばかりね。
      |      ` -'\       ー'  人            
    |        /(l     __/  ヽ、          
     |       (:::::`‐-、__  |::::`、     ヒニニヽ、         
    |      / `‐-、::::::::::`‐-、::::\   /,ニニ、\            
   |      |::::::::::::::::::|` -、:::::::,ヘ ̄|'、  ヒニ二、 \
.   |      /::::::::::::::::::|::::::::\/:::O`、::\   | '、   \
   |      /:::::::::::::::::::/:::::::::::::::::::::::::::::'、::::\ノ  ヽ、  |
  |      |:::::/:::::::::/:::::::::::::::::::::::::::::::::::'、',::::'、  /:\__/‐、
  |      |/:::::::::::/::::::::::::::::::::::::::::::::::O::| '、::| く::::::::::::: ̄|
   |     /_..-'´ ̄`ー-、:::::::::::::::::::::::::::::::::::|/:/`‐'::\;;;;;;;_|
   |    |/::::::::::::::::::::::\:::::::::::::::::::::::::::::|::/::::|::::/:::::::::::/
    |   /:::::::::::::::::::::::::::::::::|:::::::::::::::::::::O::|::|::::::|:::::::::::::::/
147132人目の素数さん:2013/01/26(土) 15:29:26.56
>>145
(1,1)を変換しても(1,1)のままってことは、
方向ベクトルが(1,1)の直線を変換しても方向ベクトルは(1,1)のままってこと。
148132人目の素数さん:2013/01/26(土) 15:30:18.94
>>145
方向ベクトル(1,1)の始点を原点に取るとその終点が(1,1)であり
その点(1,1)が一次変換fの不動点であることから『』がいえる

もっと簡単に一次変換fの不動点pがあれば
成分をpの座標と同一とする方向ベクトルp↑も一次変換fで不動であり
一次変換fにおける固有値1の固有ベクトルと表現してもいい

・方向ベクトルという言葉の定義
・一次変換における基底と固有ベクトル
このへんの復習が必要ではないかと思う次第
149132人目の素数さん:2013/01/26(土) 16:44:14.68
細い円柱(底面積S, 長さ l , 密度ρ)の棒が、軸を鉛直に保って水面へ鉛直下向きに速さVで水中に突入するとき
どこまで水中に潜るかを考えたいのですが、水面から棒の底面までの距離をy(鉛直下向きを正に)として、
運動方程式は Mを棒の質量 (= ρSl)として

 y≦l のとき My'' = Mg - (Sy)g 
 y≧l のとき My'' = Mg - (Sl)g

でいいでしょうか。右辺は重力-浮力を考えたつもりです。
また、いいとしてら、これより水中のどこまで潜るかを解くにはどうすればいいでしょうか。
150132人目の素数さん:2013/01/26(土) 16:52:54.82
>>149
>右辺は重力-浮力を考えたつもり
次元が合ってない

>水中のどこまで潜るかを解くには
エネルギー保存
151132人目の素数さん:2013/01/26(土) 18:37:46.67
密度1より大きかったらどこまでも沈むだろダボ
152132人目の素数さん:2013/01/26(土) 19:27:35.20
物理までカバーできるとかなんて有能なのこのスレ
153132人目の素数さん:2013/01/26(土) 19:39:57.70
-4/π(log1/√2-log1)
ココから先が出来ません
154132人目の素数さん:2013/01/26(土) 19:56:35.88
対数の公式くらい調べなさい
155132人目の素数さん:2013/01/26(土) 20:15:33.71
サイコロを3回振り、
一回目の目をA、二回目の目をB、三回目の目をCとした
(A、B、C)の組を作る

組を次々に生成する

n回目に初めて
【それまで出たいずれかの組と全く同じ組が生成される確率】
はどうなるか
156132人目の素数さん:2013/01/26(土) 21:07:01.48
>>151
別に密度が1以上とか仮定されてないだろハゲ
157132人目の素数さん:2013/01/26(土) 21:11:46.27
158132人目の素数さん:2013/01/26(土) 21:20:55.07
>>150
たぶんSygには水の密度1[kg/m^3]がかけられているんだろう
159132人目の素数さん:2013/01/26(土) 21:26:47.29
>>149
y≦lのとき 単振動
y≧lのとき 等加速度運動
160132人目の素数さん:2013/01/26(土) 21:43:33.47
>>158
水はそんなに軽くない。
161132人目の素数さん:2013/01/26(土) 21:44:57.15
ワロタ
162132人目の素数さん:2013/01/26(土) 21:47:29.61
17年振りだなぁ。
163132人目の素数さん:2013/01/26(土) 21:50:49.91
>>159
バーカ
164132人目の素数さん:2013/01/26(土) 21:58:37.85
東大の主張通りなら液体ヘリウムの一形態が2kg/m^3らしいな
http://www.s.u-tokyo.ac.jp/ja/press/2012/44.html
あともうちょっとだ
165132人目の素数さん:2013/01/26(土) 22:01:00.67
受験に精通してないと回答できないとは思うが
バウムクーヘンのy軸周りの回転体の公式とか∫(α~β)(2/3)πr^3sinθdθの
x軸周りの回転体の公式ってx軸またいでたら使えないの?
166132人目の素数さん:2013/01/26(土) 22:19:40.92
1/3π+π(1−1/a)-1/3aπ
=4/3π(1−1/a)

途中式お願い
167132人目の素数さん:2013/01/26(土) 22:23:47.15
168132人目の素数さん:2013/01/26(土) 22:33:14.39
えこれ普通にカッコから出して足してでまたくくればいいんじゃないの
中1でも感のいいやつは分かるレベルだろこれ
169132人目の素数さん:2013/01/26(土) 22:46:49.45
>>162
やあ
170132人目の素数さん:2013/01/26(土) 23:00:01.15
>>155
誕生日のパラドックスと同じ感覚でときゃいい
171132人目の素数さん:2013/01/26(土) 23:03:24.53
このスレで物理質問すること自体間違ってるけど>>159は何がおかしいんだ?
172132人目の素数さん:2013/01/26(土) 23:06:59.25
>>171合ってるでしょ
173132人目の素数さん:2013/01/26(土) 23:15:38.02
いや間違ってるだろ
174132人目の素数さん:2013/01/26(土) 23:17:06.12
>>155
(A,B,C)の組をn回作るってことだよな?

(A,B,C)の組は 6*6*6=216通りあり、全て同様に確からしい。
n回目に初めてそれまで出たいずれかの組と全く同じ組が生成される確率は
(n-1)回目まで互いに異なる目が出て、
n回目でその(n-1)通りの目のうちの1つが出る確率だから
P[216,n-1]/216^(n-1) * ((n-1)/216)
175132人目の素数さん:2013/01/26(土) 23:18:19.91
>>173
どこが?
176132人目の素数さん:2013/01/26(土) 23:20:58.50
http://i.imgur.com/WrL0B6H.jpg
右の図は正三角形ABCを
頂点Aが辺BC上の点E
に重なるように折り曲げた
図です。
BE=3.ED=7.DB=8
のときのEFを求めよ。
これ解けませんか?
177132人目の素数さん:2013/01/26(土) 23:26:19.33
解けないことを証明するのは難しいんじゃね・
178132人目の素数さん:2013/01/26(土) 23:37:38.23
確率の問題を教えてください。

A、B2人がじゃんけんをする。2人とも6回に3回はグー、2回はチョキ、1回はパーを出すとき、Aが勝つ確率を求めよ。

2人とも同じ数の手を出すのならば1/2だとおもうの ですが、違うようです。
179132人目の素数さん:2013/01/26(土) 23:41:12.26
授業が遅くなれば遅いですし
180132人目の素数さん:2013/01/26(土) 23:41:15.57
>>178
あいこ
181132人目の素数さん:2013/01/26(土) 23:42:46.76
あなたがベイズ確率の信奉者であっても
勝つ負けるそしてあいこがあるから
そのそれぞれは1/3になるはずだ
182178:2013/01/26(土) 23:55:21.05
わかりました!
183132人目の素数さん:2013/01/27(日) 00:00:25.98
何をわかったんだ
披露してみろ
184178:2013/01/27(日) 00:05:49.89
>>183
あいこを考えなきゃいけないから、1からあいこの確率を引く。これはAの勝率だから2で割る。あいこの確率は出す手の確率の2乗。
あってますか?
185132人目の素数さん:2013/01/27(日) 00:42:09.77
http://www.imgur.com/0hUdeL3.jpeg
よって、log_{3}{(a(n+1)/a(n)}=…
の変形の仕方がわかりません。
186132人目の素数さん:2013/01/27(日) 00:43:45.94
185ですすみません改めて見たらわかりました。ごめんなさい。
187132人目の素数さん:2013/01/27(日) 01:09:34.13
>>176
AD=ED=7よりAB=AD+DB=7+8=15であるから
正三角形ABCの一辺の長さが15。
よってEF=xとおけば、AF=EF=xゆえ、CF=AC-AF=15-xである。
すなわち、三角形CFEにおいて、
CE=CB-EB=15-3=12、EF=x、CF=15-a、∠C=60°
であるから、余弦定理により
a^2=12^2+(15-a)^2-2・12・(15-a)cos(60°)
これより、EF=a=? である。
188132人目の素数さん:2013/01/27(日) 01:10:41.47
>>187

xはaと読み替えてくれ。
189132人目の素数さん:2013/01/27(日) 01:24:42.45
お願いします。

sinθ+cosθ=1のときsin2θ、cos4θ
2倍角の公式にあてはめるんでしょうか?

よろしくお願いします。
190132人目の素数さん:2013/01/27(日) 01:26:53.57
>>189
その通り
191132人目の素数さん:2013/01/27(日) 02:08:21.60
絶対値を含む方程式の場合わけの意味がわからないんですが、、
どなたか詳しく教えてくれませんか?
192132人目の素数さん:2013/01/27(日) 02:15:21.86
>>191
|x|+x=1を満たすxを求めて見て下さい。
193132人目の素数さん:2013/01/27(日) 03:06:20.15
>>192
すいません、基本を理解してないんでさっぱり、、、
教科書、例題みてもよくわかりません
194132人目の素数さん:2013/01/27(日) 03:17:12.95
なぜ場合分けするのか考えろ
195132人目の素数さん:2013/01/27(日) 03:38:37.98
>>191
絶対値がどんなものか分かってるのか?
196132人目の素数さん:2013/01/27(日) 04:01:45.23
lim[x→∞]((x^3-x^2)^(1/3)-x)^2

極限値の求め方が分かりません
197132人目の素数さん:2013/01/27(日) 05:06:32.17
a^3-b^3=(a-b)(a^2+ab+b^2)
a-b=(a^3-b^3)/(a^2+ab+b^2)
を使い分子の有理化を行う
198132人目の素数さん:2013/01/27(日) 05:51:57.39
>>197
できました
ありがとうございます
199132人目の素数さん:2013/01/27(日) 08:02:31.06
>>171
微分方程式の意味を勘違いしてるだろ
>>159の微分方程式は
 棒が水中に完全に没しないあいだは第一の式だが
 その後、もしも水中に完全に没することになったら第二の式が発動する
 さらにその後、浮かび上がってまた水面上に顔を出すことがあったりすると第一の式が発動する
ってことだろ。171の答じゃ何の意味もないじゃん。
200132人目の素数さん:2013/01/27(日) 11:11:09.20
何分かり切ったことを得意気に言ってるんだ
それぞれの微分方程式を解くと単振動(の一部)と等加速度運動を表すということだろ
201132人目の素数さん:2013/01/27(日) 11:25:45.78
>>193
まず、数直線というものを調べてみたらいい。

それが分ったとして、
実数 x の数直線上での位置(点)をPとおくとき、
原点Oに対して線分OPの長さを実数xの絶対値という。
長さだから、実数xの絶対値は常に負でない数(正の数 または 0)になる。
たとえば、 3 の絶対値は 3、-2 の絶対値は 2、 0の絶対値は 0 だ。
このことを記号を使って書けば、
x≧0の時 |x|=x、 x<0の時 |x|=-x となる。

>>192 を解けば、
x≧0の範囲で|x|+x=1 を満たすxはあるか?
x<0の範囲で|x|+x=1 を満たすxはあるか?
の2点を調べることになる。
もし x≧0 の範囲で解があるならば、
その解を a とおくと |a|=a であるからa+a=1を満たさなければならない。
よってa=1/2となるが、実際x=1/2を|x|+xに代入すると |1/2|+1/2=1となり確かに解になっている。

x<0 の範囲で解があるならば、
その解を b とおくと、 |b|=-b であるから -b+b=1 を満たさなければならない。
しかし、この場合 左辺:-b+b の値は 0 であるから、-b+b=1 を満たさない。
よって x<0 の場合は解はない。

以上から、求める x の値は 1/2 である。 
202132人目の素数さん:2013/01/27(日) 11:48:12.26
>>200
それを書いてなんか意味あんの?意図的無駄レスのつもり?
203132人目の素数さん:2013/01/27(日) 11:55:02.08
204132人目の素数さん:2013/01/27(日) 11:56:48.59
まあ2ch自体無駄なモンだし
205132人目の素数さん:2013/01/27(日) 12:32:26.42
>>149
水の粘性抵抗はどうするんだ?
206132人目の素数さん:2013/01/27(日) 12:45:44.98
粘性についてはプリンキピアに詳しく書いてあります
ヒマな人は是非とも読みましょう
207132人目の素数さん:2013/01/27(日) 13:05:02.69
ゲームの事?
208132人目の素数さん:2013/01/27(日) 13:08:06.49
浮力によって物体の速度が変化するので抗力も変化する
よって等加速度運動はしない
209132人目の素数さん:2013/01/27(日) 13:09:06.01
浮力と重力の合力だったな
210132人目の素数さん:2013/01/27(日) 13:30:28.28
問題文に無い水の抗力持ち出してくるバカがいるとはな
211132人目の素数さん:2013/01/27(日) 13:32:45.56
>>170>>174
ありがとうございました。
216通りもあるのに18回目にはかぶる確率が50%超えるんですね。
なんだか不思議です。
212132人目の素数さん:2013/01/27(日) 13:35:08.90
213132人目の素数さん:2013/01/27(日) 13:39:59.32
アホが一人居るだけみたいだし、物理の話題はそろそろ止めようぜ
214132人目の素数さん:2013/01/27(日) 14:48:52.75
問題文にはgも書かれていないが?
都合のいいように解釈して馬鹿呼ばわりとは情けない
それに水による抗力は気体による抗力よりはるかに大きく決して無視出来るものではないのだが
数学だけで物理学には精通していないとみた
215132人目の素数さん:2013/01/27(日) 14:52:01.48
>>149の要求は、149の微分方程式でyのとり得る値を求めたいってんだから
物理知らなくても微分方程式解いてやればいいんじゃないの?
216132人目の素数さん:2013/01/27(日) 15:30:12.31
関数f(x)がf(x)=∫[0,π/2]g(t)*sin(x-t)dtと定義されているとき
f(x)にx=tを代入することはできますか?
217132人目の素数さん:2013/01/27(日) 15:33:27.30
ヴァカじゃねーの
そんなアホ計算したら
ロケットが爆散だ
218132人目の素数さん:2013/01/27(日) 17:13:07.37
          __ノ)-'´ ̄ ̄`ー- 、_
        , '´  _. -‐'''"二ニニ=-`ヽ、
      /   /:::::; -‐''"        `ーノ
     /   /:::::/           \
     /    /::::::/          | | |  |
     |   |:::::/ /     |  | | | |  |
      |   |::/ / / |  | ||  | | ,ハ .| ,ハ|
      |   |/ / / /| ,ハノ| /|ノレ,ニ|ル' 
     |   |  | / / レ',二、レ′ ,ィイ|゙/   私は只の数ヲタなんかとは付き合わないわ。
.     |   \ ∠イ  ,イイ|    ,`-' |      頭が良くて数学が出来てかっこいい人。それが必要条件よ。
     |     l^,人|  ` `-'     ゝ  |        さらに Ann.of Math に論文書けば十分条件にもなるわよ。
      |      ` -'\       ー'  人          一番嫌いなのは論文数を増やすためにくだらない論文を書いて
    |        /(l     __/  ヽ、           良い論文の出版を遅らせるお馬鹿な人。
     |       (:::::`‐-、__  |::::`、     ヒニニヽ、         あなたの論文が Ann of Math に accept される確率は?
    |      / `‐-、::::::::::`‐-、::::\   /,ニニ、\            それとも最近は Inv. Math. の方が上かしら?
   |      |::::::::::::::::::|` -、:::::::,ヘ ̄|'、  ヒニ二、 \
.   |      /::::::::::::::::::|::::::::\/:::O`、::\   | '、   \
   |      /:::::::::::::::::::/:::::::::::::::::::::::::::::'、::::\ノ  ヽ、  |
  |      |:::::/:::::::::/:::::::::::::::::::::::::::::::::::'、',::::'、  /:\__/‐、
  |      |/:::::::::::/::::::::::::::::::::::::::::::::::O::| '、::| く::::::::::::: ̄|
   |     /_..-'´ ̄`ー-、:::::::::::::::::::::::::::::::::::|/:/`‐'::\;;;;;;;_|
   |    |/::::::::::::::::::::::\:::::::::::::::::::::::::::::|::/::::|::::/:::::::::::/
    |   /:::::::::::::::::::::::::::::::::|:::::::::::::::::::::O::|::|::::::|:::::::::::::::/
219132人目の素数さん:2013/01/27(日) 19:57:10.38
確率の!の計算方法と意味を教えてください。
教科書見ても書いてなかったので知りたいです
220132人目の素数さん:2013/01/27(日) 19:58:49.13
ttp://ja.wikipedia.org/wiki/%E9%9A%8E%E4%B9%97
あとお前は中学生あたりから入念にやり直した方がいい
221132人目の素数さん:2013/01/27(日) 20:16:19.62
4!=4*3*2*1
222132人目の素数さん:2013/01/27(日) 20:36:15.08
すいません!
確率とか結構苦手でして、、
なら質問なんですけど、赤玉ニ 青玉ニ 白玉三
を合わせて七個の玉を横一列に並べる
ただし同じ色の玉は区別しないものとする

白玉どうしが隣合わない並べ方は何通りあるか

という問題でなんで4!/2!2!×5C3となるんですか?
223132人目の素数さん:2013/01/27(日) 20:43:24.58
それは確率の問題じゃない
224132人目の素数さん:2013/01/27(日) 21:01:05.60
バカはどうせ解けないんだから全パターン書け
バカでもできるし運が良ければ解けるかもしれない
解き方なんか覚えても無駄
225132人目の素数さん:2013/01/27(日) 21:01:22.18
x≧-6の範囲のすべてのxに対して不等式 2ax≦6x+1
が成り立つための必要十分条件は□≦a≦□
という問題なのですが
答えは(35)/(12)≦a≦3になるらしいのですが
xの係数が正のときと負のときに場合分けして
正(a>3)のときa≧(35)/(12)
負(a<3)のときa≦(35)/(12)となり答えになりません
お願いします
226132人目の素数さん:2013/01/27(日) 21:09:39.64
洗剤A平均860g偏差20
洗剤B平均863g偏差25
洗剤ABをそれぞれ1箱ランダムにとるとき、Bの方が重い確率
教えてください
227132人目の素数さん:2013/01/27(日) 21:13:29.62
思い軽い同じの三択であり
すなわち1/3
228132人目の素数さん:2013/01/27(日) 21:18:03.17
説明してくれませんか?
質問スレなんでお願いします!
229132人目の素数さん:2013/01/27(日) 21:19:09.90
質問スレなんでって何?
230132人目の素数さん:2013/01/27(日) 21:27:24.11
大雑把で良いので解法の方針を教えて下さい。

【問】行列([a,b][1,2])で表される1次変換をfとする。y=x-3をlとするとき、l上の各点はfによって、l上に移る。このときのa,bを求めよ。また、直線3x+2y+1=0上の点Pによる像をQとする。点Pが直線3x+2y+1=0全体を動くとき、点Qの描く図形の方程式を求めよ。
231132人目の素数さん:2013/01/27(日) 21:28:52.11
統計の値がどうであろうと
重さの場合ならば 重い・軽い・同じ のどれかなので
その三択になる
従って1/3
232132人目の素数さん:2013/01/27(日) 21:44:45.44
>>222
白玉を○、赤玉を●、青玉を▲で表すことにすると、白玉どうしが隣り合わない並べ方ってのは例えば

○●○▲ ● ▲○

 ●○▲○▲ ●○

みたいな感じ。
で、これで気付いてくれるといいんだが、つまりこういう並び方は全部


 ● ▲ ▲ ●

みたいに、青玉と赤玉が並んでいて、その隙間に三つ白玉をおいているみなせる。
隙間の部分を□であえて書くと

□●□▲□▲□●□

この□に白玉を三つ入れるから5C3通り

それに最初の赤玉と青玉の並べ方4!/2!2!をかければOK
233132人目の素数さん:2013/01/27(日) 21:53:02.64
>>226
普通に考えてBが重くなる確率の方が高くなるよな
だが、それ以上のことは分からん。分かるやつが出てくるの待ってくれ
234132人目の素数さん:2013/01/27(日) 21:59:11.16
>>225
係数が0のときを忘れている。
35/12はいちいち分子分母に括弧をつける必要はないぞ。
235132人目の素数さん:2013/01/27(日) 22:01:17.98
>>225
a>3ではそもそも成り立たないぞ
a≦3のときの範囲も間違えてる
計算過程で符号間違ってないか?
236132人目の素数さん:2013/01/27(日) 22:04:05.20
>>226
条件不足で求まらないんじゃね?
237132人目の素数さん:2013/01/27(日) 22:06:47.41
>>234-235
ありがとうございます。
そもそも問題の意味を理解していませんでした。
やり直してみたら解決しました。
238132人目の素数さん:2013/01/27(日) 22:14:20.34
          __ノ)-'´ ̄ ̄`ー- 、_
        , '´  _. -‐'''"二ニニ=-`ヽ、
      /   /:::::; -‐''"        `ーノ
     /   /:::::/           \
     /    /::::::/          | | |  |
     |   |:::::/ /     |  | | | |  |
      |   |::/ / / |  | ||  | | ,ハ .| ,ハ|
      |   |/ / / /| ,ハノ| /|ノレ,ニ|ル' 
     |   |  | / / レ',二、レ′ ,ィイ|゙/   私は只の数ヲタなんかとは付き合わないわ。
.     |   \ ∠イ  ,イイ|    ,`-' |      頭が良くて数学が出来てかっこいい人。それが必要条件よ。
     |     l^,人|  ` `-'     ゝ  |        さらに Ann.of Math に論文書けば十分条件にもなるわよ。
      |      ` -'\       ー'  人          一番嫌いなのは論文数を増やすためにくだらない論文を書いて
    |        /(l     __/  ヽ、           良い論文の出版を遅らせるお馬鹿な人。
     |       (:::::`‐-、__  |::::`、     ヒニニヽ、         あなたの論文が Ann of Math に accept される確率は?
    |      / `‐-、::::::::::`‐-、::::\   /,ニニ、\            それとも最近は Inv. Math. の方が上かしら?
   |      |::::::::::::::::::|` -、:::::::,ヘ ̄|'、  ヒニ二、 \
.   |      /::::::::::::::::::|::::::::\/:::O`、::\   | '、   \
   |      /:::::::::::::::::::/:::::::::::::::::::::::::::::'、::::\ノ  ヽ、  |
  |      |:::::/:::::::::/:::::::::::::::::::::::::::::::::::'、',::::'、  /:\__/‐、
  |      |/:::::::::::/::::::::::::::::::::::::::::::::::O::| '、::| く::::::::::::: ̄|
   |     /_..-'´ ̄`ー-、:::::::::::::::::::::::::::::::::::|/:/`‐'::\;;;;;;;_|
   |    |/::::::::::::::::::::::\:::::::::::::::::::::::::::::|::/::::|::::/:::::::::::/
    |   /:::::::::::::::::::::::::::::::::|:::::::::::::::::::::O::|::|::::::|:::::::::::::::/
239132人目の素数さん:2013/01/27(日) 22:33:50.29
>>232
なるほど!
ありがとうごさいます!
240132人目の素数さん:2013/01/27(日) 22:37:31.49
>>232
すいません、追記なんですが
青玉と赤玉の式がなぜ4!/2!2!となるんですか?
wikiよんでもいまいちわからなかったので、、
241132人目の素数さん:2013/01/27(日) 22:52:18.11
>>230
自己解決
スレ汚し失礼。
242132人目の素数さん:2013/01/27(日) 22:58:53.41
>>240
教科書か参考書を読め。
243132人目の素数さん:2013/01/27(日) 22:59:55.50
>>240
●×▲△
●△▲×
×△●▲
△×●▲
みたいに△×●▲のような異なるものを四つ並べるなら並べ方は4!通り


ここで×が△に変わったのを考えて、△△●▲の並べ方を考えると

●△▲△
●△▲△
△△●▲
△△●▲

のようにもとの4!通りの並べ方で△と×が入れ替わっただけの並べ方は同じものになるから
2で割って4!/2通りになる
さらに●が▲に変わった△△▲▲の並べ方も同様に●と▲が入れ替わっただけのものはおなじになるので
さらに2で割って4!/2・2通りになる
244132人目の素数さん:2013/01/27(日) 23:00:45.83
ABCD-EFGHで各辺が2センチの立方体があります。
辺FGの中点をB、辺PHの中点をQ、辺EQの延長で辺GHに当たる点をRとしたとき

EQ:QR
ERの長さ、
三角形DERの面積を教えてください!!
245132人目の素数さん:2013/01/27(日) 23:12:57.86
問題文を考えるのもめんどくせぇ
だから図に書いてうp

あと
> 辺FGの中点をB
この空間は何次元なんだ
246132人目の素数さん:2013/01/27(日) 23:14:45.50
クソワロタww
247132人目の素数さん:2013/01/27(日) 23:15:04.34
>>244
最初の比もわからんのか?
248132人目の素数さん:2013/01/27(日) 23:15:39.78
中点はPですすいません

画像の載せ方がわかりません(´・_・`)
249132人目の素数さん:2013/01/27(日) 23:21:48.40
面積(2/3)√17になっちゃった。どこか計算間違えてる気分。
250132人目の素数さん:2013/01/27(日) 23:55:51.78
>>243
なるほど!
ありがとうございます!
251132人目の素数さん:2013/01/28(月) 07:21:58.40
参考書の問題
2x^2-16x-24

参考書の答え
=2(x^2-8x+12)
=2(x-6)(x-2)

ここたすき掛けだとこうなると思うんだけど
=(2x-12)(x-2)

前者が(a)(b)(c)、後者が(ab)(c)ということでどちらも正解でよい?
252132人目の素数さん:2013/01/28(月) 07:35:37.93
2x と 12 が 「2」という共通因数をもつわけだから
因数分解せよ という問題なら
 2(x-6)(x-2) の方が “より正しい” という感じがするがな。
253132人目の素数さん:2013/01/28(月) 07:51:52.82
>>252
なるほどなあ
thx
254132人目の素数さん:2013/01/28(月) 10:21:22.51
          __ノ)-'´ ̄ ̄`ー- 、_
        , '´  _. -‐'''"二ニニ=-`ヽ、
      /   /:::::; -‐''"        `ーノ
     /   /:::::/           \
     /    /::::::/          | | |  |
     |   |:::::/ /     |  | | | |  |
      |   |::/ / / |  | ||  | | ,ハ .| ,ハ|
      |   |/ / / /| ,ハノ| /|ノレ,ニ|ル' 
     |   |  | / / レ',二、レ′ ,ィイ|゙/   私は只の数ヲタなんかとは付き合わないわ。
.     |   \ ∠イ  ,イイ|    ,`-' |      頭が良くて数学が出来てかっこいい人。それが必要条件よ。
     |     l^,人|  ` `-'     ゝ  |        さらに Ann.of Math に論文書けば十分条件にもなるわよ。
      |      ` -'\       ー'  人          一番嫌いなのは論文数を増やすためにくだらない論文を書いて
    |        /(l     __/  ヽ、           良い論文の出版を遅らせるお馬鹿な人。
     |       (:::::`‐-、__  |::::`、     ヒニニヽ、         あなたの論文が Ann of Math に accept される確率は?
    |      / `‐-、::::::::::`‐-、::::\   /,ニニ、\            それとも最近は Inv. Math. の方が上かしら?
   |      |::::::::::::::::::|` -、:::::::,ヘ ̄|'、  ヒニ二、 \
.   |      /::::::::::::::::::|::::::::\/:::O`、::\   | '、   \
   |      /:::::::::::::::::::/:::::::::::::::::::::::::::::'、::::\ノ  ヽ、  |
  |      |:::::/:::::::::/:::::::::::::::::::::::::::::::::::'、',::::'、  /:\__/‐、
  |      |/:::::::::::/::::::::::::::::::::::::::::::::::O::| '、::| く::::::::::::: ̄|
   |     /_..-'´ ̄`ー-、:::::::::::::::::::::::::::::::::::|/:/`‐'::\;;;;;;;_|
   |    |/::::::::::::::::::::::\:::::::::::::::::::::::::::::|::/::::|::::/:::::::::::/
    |   /:::::::::::::::::::::::::::::::::|:::::::::::::::::::::O::|::|::::::|:::::::::::::::/
255132人目の素数さん:2013/01/28(月) 11:36:39.22
すいません数3の問題なんですが、問題文が∫(上1下0)1/√1+x^2dx で
たぶん最後のほうで-1/2log(3-2√2)というのが出てきたのですが、
ここから先が解けないのでおしえてくだしあ
256132人目の素数さん:2013/01/28(月) 12:32:41.25
式くらいまともに書けバカ
257132人目の素数さん:2013/01/28(月) 12:58:34.25
>>255
∫[0,1] dx/sqrt(1+x^2)
=∫[0,π/4] dt/cos(t)  (x:=tan(t), dx=dt/cos^2(t) )
=∫[0,π/4] cos(t)dt/{1-sin^2(t)}
∫[0,1/sqrt(2)] du/(1-u^2)  (u:=sin(t), du=cos(t)dt )
=1/2∫[0,1/sqrt(2)] {1/(1+u) + 1/(1-u)}du
=1/2[ln|1+u| - ln|1-u|] |_[u=0,1/sqrt(2)]
=1/2ln|{sqrt(2)+1}/{sqrt(2)-1}|
=1/2ln|sqrt(2)+1|^2
=ln|sqrt(2)+1|
=ln{sqrt(2)+1}

最後に分母を有理化する際、分子を展開しないでln(x^n)=n*ln(x)を適用

数値だけ出されても意味不明なので次からは途中式も書くように
それと式を自己流の書き方で書かないように
258132人目の素数さん:2013/01/28(月) 13:08:49.31
頻出形の積分だ
原始関数を覚えておこう
259132人目の素数さん:2013/01/28(月) 14:50:07.90
          __ノ)-'´ ̄ ̄`ー- 、_
        , '´  _. -‐'''"二ニニ=-`ヽ、
      /   /:::::; -‐''"        `ーノ
     /   /:::::/           \
     /    /::::::/          | | |  |
     |   |:::::/ /     |  | | | |  |
      |   |::/ / / |  | ||  | | ,ハ .| ,ハ|
      |   |/ / / /| ,ハノ| /|ノレ,ニ|ル' 
     |   |  | / / レ',二、レ′ ,ィイ|゙/   私は只の数ヲタなんかとは付き合わないわ。
.     |   \ ∠イ  ,イイ|    ,`-' |      頭が良くて数学が出来てかっこいい人。それが必要条件よ。
     |     l^,人|  ` `-'     ゝ  |        さらに Ann.of Math に論文書けば十分条件にもなるわよ。
      |      ` -'\       ー'  人          一番嫌いなのは論文数を増やすためにくだらない論文を書いて
    |        /(l     __/  ヽ、           良い論文の出版を遅らせるお馬鹿な人。
     |       (:::::`‐-、__  |::::`、     ヒニニヽ、         あなたの論文が Ann of Math に accept される確率は?
    |      / `‐-、::::::::::`‐-、::::\   /,ニニ、\            それとも最近は Inv. Math. の方が上かしら?
   |      |::::::::::::::::::|` -、:::::::,ヘ ̄|'、  ヒニ二、 \
.   |      /::::::::::::::::::|::::::::\/:::O`、::\   | '、   \
   |      /:::::::::::::::::::/:::::::::::::::::::::::::::::'、::::\ノ  ヽ、  |
  |      |:::::/:::::::::/:::::::::::::::::::::::::::::::::::'、',::::'、  /:\__/‐、
  |      |/:::::::::::/::::::::::::::::::::::::::::::::::O::| '、::| く::::::::::::: ̄|
   |     /_..-'´ ̄`ー-、:::::::::::::::::::::::::::::::::::|/:/`‐'::\;;;;;;;_|
   |    |/::::::::::::::::::::::\:::::::::::::::::::::::::::::|::/::::|::::/:::::::::::/
    |   /:::::::::::::::::::::::::::::::::|:::::::::::::::::::::O::|::|::::::|:::::::::::::::/
260132人目の素数さん:2013/01/28(月) 15:30:29.07
|√(2a^2-1)*sin(t+a)-√3a|の絶対値を外すときに、前提としてa>1,-1≦sin(t+a)≦1があるのですが、
解答では
(√3a)^2-{√(2a^2-1)}^2=a^2+1>0としていますが、
sin(t+a)はいれなくていいのでしょうか?
261132人目の素数さん:2013/01/28(月) 15:44:39.72
>>260
sinが掛かったらもっと小さくなるだろ
262132人目の素数さん:2013/01/28(月) 15:59:31.61
>>261
どういうことか、説明してくれないか。
263132人目の素数さん:2013/01/28(月) 16:30:45.84
>>260
いろいろと端折りすぎていてよくわからない。
>>1
> ・質問者は回答者がわかるように問題を書くようにしましょう。でないと放置されることがあります。
>   (変に省略するより全文書いた方がいい、また説明なく習慣的でない記号を使わないように)
264132人目の素数さん:2013/01/28(月) 16:54:30.86
>>263
すみませんでした。
http://beebee2see.appspot.com/i/azuYhpvfBww.jpg
http://beebee2see.appspot.com/i/azuY1b7cBww.jpg

この問題で、この解答です。
265132人目の素数さん:2013/01/28(月) 17:17:55.27
(√3)a>√(2a^2+1)
-√(2a^2+1)≦sin(t+a)*√(2a^2+1)≦√(2a^2+1)
より
(√3)a-sin(t+a)*√(2a^2+1)≧(√3)a-√(2a^2+1)>0.
266132人目の素数さん:2013/01/28(月) 17:31:15.87
「log2x=5/12」を「log(1/2)=-5/12」に直す時
記述ではどういう答え方がいいでしょうか
267132人目の素数さん:2013/01/28(月) 17:36:47.85
2xを1/2、また5/12を-5/12に書き換えることにより、以下の等式を得る。
268132人目の素数さん:2013/01/28(月) 18:13:49.10
直せるのかそれ
269132人目の素数さん:2013/01/28(月) 18:16:24.55
>>266
「log2x=5/12」を「log(1/2)x=-5/12」
x抜けてましたね。
270132人目の素数さん:2013/01/28(月) 18:27:36.43
艇の変換硬式より

で、式変形でいんでねぇべか?
271132人目の素数さん:2013/01/28(月) 18:44:19.34
          __ノ)-'´ ̄ ̄`ー- 、_
        , '´  _. -‐'''"二ニニ=-`ヽ、
      /   /:::::; -‐''"        `ーノ
     /   /:::::/           \
     /    /::::::/          | | |  |
     |   |:::::/ /     |  | | | |  |
      |   |::/ / / |  | ||  | | ,ハ .| ,ハ|
      |   |/ / / /| ,ハノ| /|ノレ,ニ|ル' 
     |   |  | / / レ',二、レ′ ,ィイ|゙/   私は只の数ヲタなんかとは付き合わないわ。
.     |   \ ∠イ  ,イイ|    ,`-' |      頭が良くて数学が出来てかっこいい人。それが必要条件よ。
     |     l^,人|  ` `-'     ゝ  |        さらに Ann.of Math に論文書けば十分条件にもなるわよ。
      |      ` -'\       ー'  人          一番嫌いなのは論文数を増やすためにくだらない論文を書いて
    |        /(l     __/  ヽ、           良い論文の出版を遅らせるお馬鹿な人。
     |       (:::::`‐-、__  |::::`、     ヒニニヽ、         あなたの論文が Ann of Math に accept される確率は?
    |      / `‐-、::::::::::`‐-、::::\   /,ニニ、\            それとも最近は Inv. Math. の方が上かしら?
   |      |::::::::::::::::::|` -、:::::::,ヘ ̄|'、  ヒニ二、 \
.   |      /::::::::::::::::::|::::::::\/:::O`、::\   | '、   \
   |      /:::::::::::::::::::/:::::::::::::::::::::::::::::'、::::\ノ  ヽ、  |
  |      |:::::/:::::::::/:::::::::::::::::::::::::::::::::::'、',::::'、  /:\__/‐、
  |      |/:::::::::::/::::::::::::::::::::::::::::::::::O::| '、::| く::::::::::::: ̄|
   |     /_..-'´ ̄`ー-、:::::::::::::::::::::::::::::::::::|/:/`‐'::\;;;;;;;_|
   |    |/::::::::::::::::::::::\:::::::::::::::::::::::::::::|::/::::|::::/:::::::::::/
    |   /:::::::::::::::::::::::::::::::::|:::::::::::::::::::::O::|::|::::::|:::::::::::::::/
272132人目の素数さん:2013/01/28(月) 18:53:32.59
-log2x=-5/12
log(2x)^(-1)=-5/12
273132人目の素数さん:2013/01/28(月) 18:55:50.58
>>269
xってどこにあるん?
274132人目の素数さん:2013/01/28(月) 21:11:44.49
行列の問題で解法の一部に
『(12-9a^2)A-(36a^2+16+k)E=0
このとき、a≠0より、A≠kE(kは任意の実数)となる (※A=([2,-9a][a,2]),Eは単位行列,aは0でない実数) 』とあるのですが、
なぜa≠0だとA≠kEだと判定できるのでしょうか?
問題の前後が無く、分かり難いようなら打ち出します。どなたかお願いします。
275132人目の素数さん:2013/01/28(月) 21:16:01.19
>>274
>A=([2,-9a][a,2])
276132人目の素数さん:2013/01/28(月) 21:16:20.08
Aのaが0なら
Aは単位行列のスカラ倍になります。
277132人目の素数さん:2013/01/28(月) 21:24:02.82
>>275
>>276
早速ご回答ありがとうございました。理解…というか、気付かされ(?)ました!
278132人目の素数さん:2013/01/28(月) 22:07:30.47
ワロタwwwwwww



あの、質問なんですが
S(θ)=(θ-sinθ)/2θ^2の最大値を求める問題で(0<θ<2π)
S(θ)の分母は符号変化を担わないので分子のθ-sinθをf(θ)とおき、符号変化を調べる。(これはS(θ)の符号変化と同じ)
f'(θ)=1-cosθより
f'(θ)=0とすると、θ=0,2π
これでf'(θ)のグラフを書くと、θ=πで最大値とるような関数になります。
0<θ<2πではf'(θ)は常にプラスです。
ここからf(θ)のグラフを再生しようとすると、常に単調増加で最大値など分かりません。
なぜでしょうか?
279132人目の素数さん:2013/01/28(月) 22:24:30.80
>>274
その解法の一部というのは、なんか変だね。
280132人目の素数さん:2013/01/28(月) 22:24:59.02
f(θ)が単調増加でもS(θ)が単調増加するわけではないだろう
分母の増加にも注意しなければいけない
281132人目の素数さん:2013/01/28(月) 22:25:09.31
>>278
S(θ)の最大値を求めるときに分母無視していいと思ったの?
分母がつねに正だから?
282132人目の素数さん:2013/01/28(月) 22:28:58.88
>>278
f(θ)は単調増加でも、分母も単調増加だから
S(θ)の増減がどうなるかはまだ何も言っていないな。
283132人目の素数さん:2013/01/28(月) 23:12:16.98
>>280-282
解答ではS(θ)を一度微分して、S'(θ)として
S'(θ)の分母>0より、S'(θ)の分子の符号とS'(θ)の符号は一致する。
S'(θ)の分子をf(θ)とおき、.....と進めていたのですが、

これとの違いはなんですか?
S(θ)のグラフの概形を描くにはS'(θ)の符号変化を追えばいいので、S(θ)のグラフが書けるのは分かりますが、S(θ)の符号とS(θ)の分子の符号は一致するので...ってあれ?

よく分かりません。
284132人目の素数さん:2013/01/28(月) 23:15:48.21
>>283
>>278ではS(θ)の分子をf(θ)とおいているようだが。
285132人目の素数さん:2013/01/28(月) 23:16:45.38
>>284
自分はそうしました。
286132人目の素数さん:2013/01/28(月) 23:17:07.28

ああ、勘違いした。>>283の冒頭3行はどこかにある解答を書き写しただけなんだな。
287132人目の素数さん:2013/01/28(月) 23:25:00.91
>>283
類題: -1<x<1 のとき S(x)=1/(1+x^2) の最大値を求めてみ。
>>278 に倣って S(x)の分母は符号変化を担わないので分子の1をf(x)とおき…
でうまくいくかい?
288132人目の素数さん:2013/01/28(月) 23:37:44.09
四角形ABCDは平行四辺形でないとし、辺AB.BC,CD,DAの中点をそれぞれP,Q,R,Sとする。
線分ACの中点Mと線分BDの中点Nを結ぶ直線は点Kを通ることを示せ。

MN↑=AN↑-AM↑=1/2AB↑+1/2AD↑-1/2AC↑
AK↑=1/2AP↑+1/2AR↑=1/4AB↑+1/4AD↑+1/4AC↑
MN↑=○AK↑より、としたいのですが合いません
289132人目の素数さん:2013/01/28(月) 23:38:34.98
>>287
ですよね、分かるんです。
でもなぜ解答ではうまくいくんですかね?
多分分からないのはそこです。
290132人目の素数さん:2013/01/28(月) 23:45:31.98
>>288
点Kってなんぞ?
291132人目の素数さん:2013/01/28(月) 23:48:43.42
>>290
すみません書き忘れましたKはPRの中点です
292132人目の素数さん:2013/01/28(月) 23:51:56.23
>>288
合うわけないだろ。MN↑=○AK↑にならないんだから。
293132人目の素数さん:2013/01/28(月) 23:54:16.11
>>288
それ、ベクトルでやらなきゃダメなのか?
PNRMが平行四辺形になるから、その対角線は中点で交わるよ。
294132人目の素数さん:2013/01/28(月) 23:54:34.49
>>292
あ、MN↑=○MK↑にしなきゃですね、ありがとうございます
295132人目の素数さん:2013/01/29(火) 00:04:48.11
S={(x,y)|x^2+3y^2=1}が有界集合であることを示せ
また特異点を持たないことを示せ

この問題の解き方を教えてほしいです
296132人目の素数さん:2013/01/29(火) 00:31:15.31
楕円の周か
297132人目の素数さん:2013/01/29(火) 00:32:27.30
>>295
定義に従って、てかどこの高校数学?
298132人目の素数さん:2013/01/29(火) 00:50:15.23
質問です
曲線y=logxとx、y軸およびy=10で囲まれる領域をy軸の周りに回転して得られる器に毎秒vの水をいれる
(1)水の深さがy=hのときの水の体積を求めよ
→2πe^(2h)
(2)水の深さがhのときの水面の上昇速度を求めよ

この二番が分かりません。一番よりv=2πe^(2h)からh=1/2logv-1/2{log(2π)}を求めて、それを時間で微分しようと思ったのですが、vを微分したdv/dtが出てきてそれが分からないので困っています
よろしくお願いします
299132人目の素数さん:2013/01/29(火) 00:56:00.84
>>298
>毎秒vの水をいれる
このvと
>一番よりv=2πe^(2h)からh=1/2logv-1/2{log(2π)}を求めて
このvは別ものでは?
300132人目の素数さん:2013/01/29(火) 00:58:07.09
>>298
(1)の答をvじゃなくて大文字のVで書くぞ。
h = (1/2)logV - (1/2)log(2pi)

tで微分してdh/dt を求めようとすると右辺に dV/dt が登場するが、それは当然 v だろ。
だって注水速度(つまり水の体積の増加する速度)が毎秒一定vなんだから。
301298:2013/01/29(火) 01:04:05.76
>>299-300
なるほど! 体積を混同していました
ありがとうございました!
302132人目の素数さん:2013/01/29(火) 01:06:01.51
>>283
>S(θ)の分母は符号変化を担わないので分子のθ-sinθをf(θ)とおき、符号変化を調べる。(これはS(θ)の符号変化と同じ)

>S(θ)のグラフの概形を描くにはS'(θ)の符号変化を追えばいい
お前が追ってるのS(θ)の符号じゃん
303132人目の素数さん:2013/01/29(火) 01:19:26.70
>>298
これって(1)あってんの?
304132人目の素数さん:2013/01/29(火) 06:08:14.83
>>302
導関数の符号変化を調べることでもとの関数の単調増加とか単調減少が分かるってことですよね?

自分がやってる、S(θ)の符号変化を調べるってのはいったい何をしてるってことなんですか?
305132人目の素数さん:2013/01/29(火) 07:56:40.27
>>303
あってないよね。
(pi/2)( e^(2h) - 1) か。
306132人目の素数さん:2013/01/29(火) 08:05:51.99
S(θ)の符号追いかけたってx軸の上下どちらにあるかが分かるだけだろ
307132人目の素数さん:2013/01/29(火) 08:29:03.20
>>304
S(θ)が正か負かはたまた0か
それだけ
308132人目の素数さん:2013/01/29(火) 11:00:36.59
>>305
俺もそうだと思うんだけどどうなんだろ
paix^2を0からhまでyで積分すればいいだけだよな
309132人目の素数さん:2013/01/29(火) 14:57:39.78
>>306-307
S'(θ)の符号変化を追うことは、S(θ)の増減を追うことになりますよね?
310132人目の素数さん:2013/01/29(火) 15:02:40.40
a(r^10-1)/(r-1)=3・・・(1)
a(r^30-1)/(r-1)=21・・・(2)
(2)/(1)より
(r^10)^2+(r^10)-6=0

この2次式までの途中式お願いします
311132人目の素数さん:2013/01/29(火) 15:12:00.48
>>310
(2)/(1)って書いてあるじゃん。
312132人目の素数さん:2013/01/29(火) 15:25:02.93
オイラー関数のφ(n)はことわりなしに記述して大丈夫なのでしょうか?
313132人目の素数さん:2013/01/29(火) 15:27:29.27
(2)/(1) と書いてはいるが
まさにそこ、方程式どうしの割り算のやり方がワカランのだろ

=で結ばれた方程式二つをどうやって割り算するか
そもそも割り算していいのか
そこがボヤけてるはずだ
314132人目の素数さん:2013/01/29(火) 15:35:23.36
>>313
仰る通りその割り算の部分が分かりません
言葉足らずですみません
315132人目の素数さん:2013/01/29(火) 15:35:35.81
>>310
(i)/(ii) ⇔ (iの左辺)/(iiの左辺)=(iの右辺)/(iiの右辺)
あとはr^10=αとでもおけば見通しが良くなる
316132人目の素数さん:2013/01/29(火) 15:38:11.24
>>314
目標は a を消去すること。やり方はいろいろある。
317132人目の素数さん:2013/01/29(火) 15:47:15.64
ありがとうございました。
318315:2013/01/29(火) 15:47:44.01
>>310
こういうことだ

r^10=αとおくと
a(α−1)/(r−1)=3 ------@
a(α^3−1)/(r−1)=21 ----A

A÷@ ⇔ (α^3−1)/(α−1)=21/3
ここで(α^3−1) ⇔ (α−1)(α^2+α+1)を利用すると
A÷@ ⇔ (α^2+α+1)=7 ⇔ α^2+α−6=0 ----B

Bにα=r^10を代入して
(r^10)^2+r^10−6=0
319132人目の素数さん:2013/01/29(火) 16:01:14.32
>S(θ)の分母は符号変化を担わないので分子のθ-sinθをf(θ)とおき、符号変化を調べる。(これはS(θ)の符号変化と同じ)

S(θ)の符号変化を追いかけて何がしたいの?
S(θ)がどこでx軸と交わるかなんてこの問題を解くのには全く必要ないから
知りたいのはS(θ)の増減
それを知るにはS(θ)の導関数の符号変化を調べるの
S(θ)の符号変化とその分子の符号変化の仕方が同じだからと言って
導関数の符号変化の仕方とS(θ)の分子の導関数の符号変化の仕方が同じだなんていう根拠はどこにもないわけ
320132人目の素数さん:2013/01/29(火) 16:12:10.24
          __ノ)-'´ ̄ ̄`ー- 、_
        , '´  _. -‐'''"二ニニ=-`ヽ、
      /   /:::::; -‐''"        `ーノ
     /   /:::::/           \
     /    /::::::/          | | |  |
     |   |:::::/ /     |  | | | |  |
      |   |::/ / / |  | ||  | | ,ハ .| ,ハ|
      |   |/ / / /| ,ハノ| /|ノレ,ニ|ル' 
     |   |  | / / レ',二、レ′ ,ィイ|゙/   私は只の数ヲタなんかとは付き合わないわ。
.     |   \ ∠イ  ,イイ|    ,`-' |      頭が良くて数学が出来てかっこいい人。それが必要条件よ。
     |     l^,人|  ` `-'     ゝ  |        さらに Ann.of Math に論文書けば十分条件にもなるわよ。
      |      ` -'\       ー'  人          一番嫌いなのは論文数を増やすためにくだらない論文を書いて
    |        /(l     __/  ヽ、           良い論文の出版を遅らせるお馬鹿な人。
     |       (:::::`‐-、__  |::::`、     ヒニニヽ、         あなたの論文が Ann of Math に accept される確率は?
    |      / `‐-、::::::::::`‐-、::::\   /,ニニ、\            それとも最近は Inv. Math. の方が上かしら?
   |      |::::::::::::::::::|` -、:::::::,ヘ ̄|'、  ヒニ二、 \
.   |      /::::::::::::::::::|::::::::\/:::O`、::\   | '、   \
   |      /:::::::::::::::::::/:::::::::::::::::::::::::::::'、::::\ノ  ヽ、  |
  |      |:::::/:::::::::/:::::::::::::::::::::::::::::::::::'、',::::'、  /:\__/‐、
  |      |/:::::::::::/::::::::::::::::::::::::::::::::::O::| '、::| く::::::::::::: ̄|
   |     /_..-'´ ̄`ー-、:::::::::::::::::::::::::::::::::::|/:/`‐'::\;;;;;;;_|
   |    |/::::::::::::::::::::::\:::::::::::::::::::::::::::::|::/::::|::::/:::::::::::/
    |   /:::::::::::::::::::::::::::::::::|:::::::::::::::::::::O::|::|::::::|:::::::::::::::/
321132人目の素数さん:2013/01/29(火) 16:16:44.85
>>319
理解しました。
ありがとうございました。
322132人目の素数さん:2013/01/29(火) 16:19:56.45
>>318
ありがとうございます
3乗を分解する所が思いつかなかったので
これでスッキリしました
323132人目の素数さん:2013/01/29(火) 19:40:37.15
数列の和についてです。
n+(n+1)+(n+2)+…+2n
Σで表すとΣ(n+k)※k=0からnまで
これは数列に表れるnは定数ということですか?
それとk=0からなのは初項に1がないからですか?
教えてください
324132人目の素数さん:2013/01/29(火) 19:56:12.64
初歩的な質問です。
『直観でわかる微分積分』にありました。(66〜70ページ付近)

ベキ乗関数の微分・積分の左右方向の連続性について。グラフと図式で
描かれているのですが、ベキ乗関数の微分・積分では「微分した結果」と
「積分した結果」のそれぞれが連続的に並び、グラフの形が順に横方向に
1つずつずれていく。
ただし、y=xの0乗=1の微分とy=x分の1の積分だけは連続性から外れる。
y=xの0乗=1の微分はx分の1になると思ったら、意に反してゼロが現れた。
y=x分の1の積分は1になると思ったら、logxなどというものが現れた。
とあります。

この全体の流れが理解できる比較的簡単といいますか、わかりやすい本などありましたら、
教えていただけませんか?書店で探しましたが、どれがいいかわかりませんでした。
325132人目の素数さん:2013/01/29(火) 20:05:57.16
>>324
高校の教科書
おおざっぱな証明なら載ってる
326132人目の素数さん:2013/01/29(火) 20:13:05.59
          __ノ)-'´ ̄ ̄`ー- 、_
        , '´  _. -‐'''"二ニニ=-`ヽ、
      /   /:::::; -‐''"        `ーノ
     /   /:::::/           \
     /    /::::::/          | | |  |
     |   |:::::/ /     |  | | | |  |
      |   |::/ / / |  | ||  | | ,ハ .| ,ハ|
      |   |/ / / /| ,ハノ| /|ノレ,ニ|ル' 
     |   |  | / / レ',二、レ′ ,ィイ|゙/   私は只の数ヲタなんかとは付き合わないわ。
.     |   \ ∠イ  ,イイ|    ,`-' |      頭が良くて数学が出来てかっこいい人。それが必要条件よ。
     |     l^,人|  ` `-'     ゝ  |        さらに Ann.of Math に論文書けば十分条件にもなるわよ。
      |      ` -'\       ー'  人          一番嫌いなのは論文数を増やすためにくだらない論文を書いて
    |        /(l     __/  ヽ、           良い論文の出版を遅らせるお馬鹿な人。
     |       (:::::`‐-、__  |::::`、     ヒニニヽ、         あなたの論文が Ann of Math に accept される確率は?
    |      / `‐-、::::::::::`‐-、::::\   /,ニニ、\            それとも最近は Inv. Math. の方が上かしら?
   |      |::::::::::::::::::|` -、:::::::,ヘ ̄|'、  ヒニ二、 \
.   |      /::::::::::::::::::|::::::::\/:::O`、::\   | '、   \
   |      /:::::::::::::::::::/:::::::::::::::::::::::::::::'、::::\ノ  ヽ、  |
  |      |:::::/:::::::::/:::::::::::::::::::::::::::::::::::'、',::::'、  /:\__/‐、
  |      |/:::::::::::/::::::::::::::::::::::::::::::::::O::| '、::| く::::::::::::: ̄|
   |     /_..-'´ ̄`ー-、:::::::::::::::::::::::::::::::::::|/:/`‐'::\;;;;;;;_|
   |    |/::::::::::::::::::::::\:::::::::::::::::::::::::::::|::/::::|::::/:::::::::::/
    |   /:::::::::::::::::::::::::::::::::|:::::::::::::::::::::O::|::|::::::|:::::::::::::::/
327132人目の素数さん:2013/01/29(火) 21:51:15.23
>>323
定数でも変数でも良い
初項に1がないからで良い
328298:2013/01/30(水) 00:30:46.49
数列anが
an=1/(n^2+1)+1/(n^2+2)+・・・+1/(n^2+n)(n=1,2,3・・・)で定義されている
(1)0<an<n/(n^2+1)・・・(A)を示せ
この問いなのですが、証明自体は1/(n^2+1)+1/(n^2+1)+・・・と比較することでできたのですが、(A)の不等式はn=1のときは成り立つのでしょうか?
a1=1/(1^2+1)=1/2なので、このときn/(n^2+1)と等しいと思うのです。よって(A)の右側の不等号は<ではなく≦で表すべきではないのでしょか
よろしくお願いします
329132人目の素数さん:2013/01/30(水) 00:37:54.40
そうだね
330132人目の素数さん:2013/01/30(水) 01:16:13.14
(A)にn≧2みたいな条件ないの?
ないなら≦が正しい
33116:2013/01/30(水) 03:42:16.89
>>329-330
ありがとうございます
安心しました
332132人目の素数さん:2013/01/30(水) 04:13:37.65
この問題答えあってます?
333132人目の素数さん:2013/01/30(水) 04:19:27.48
http://iup.2ch-library.com/i/i0840722-1359487121.jpg

解き方教えてください(´;ω;`)
334132人目の素数さん:2013/01/30(水) 04:30:37.56
>>333
B=[[a,c],[b,d]]という文字のまま
2A+3BとABを書いてここに晒してみろ
335132人目の素数さん:2013/01/30(水) 04:41:12.55
これは分からんww
336132人目の素数さん:2013/01/30(水) 04:42:04.59
最後答え間違ってた。スマン。
もう自己解決できました。ありがとう
337132人目の素数さん:2013/01/30(水) 04:44:18.01
虫食いを埋める感じだよな
次の計算をせよっておかしいだろ
まぁ解決して何より
338132人目の素数さん:2013/01/30(水) 04:50:12.26
微積の問題が全くわからないんだが
教えてください。

http://iup.2ch-library.com/i/i0840724-1359487285.jpg
339132人目の素数さん:2013/01/30(水) 04:51:30.45
10問丸投げか
340132人目の素数さん:2013/01/30(水) 05:37:34.79
>>338
(1)ができないなら微積全部最初からやり直し。
341132人目の素数さん:2013/01/30(水) 05:49:42.86
というか>>338はマルチなんだよな
高校数学に自身ある人、来てくれ。
http://uni.2ch.net/test/read.cgi/math/1359486538/3
342132人目の素数さん:2013/01/30(水) 06:57:42.15
>>338
このレベルは人に聞くもんじゃなくて覚えてるもんだよな
ルール守れないバカは勉強もできないってことか
343132人目の素数さん:2013/01/30(水) 08:32:52.68
846 名前: 受験番号774 投稿日: 2011/09/16(金) 12:36:49.34 ID:kIyvx2yM
遮光率75パーセントのカーテンを二重にかけたら遮光率150パーセントになると思ったのですが
ならないようですがなぜでしょう
344132人目の素数さん:2013/01/30(水) 09:52:38.21
>>327
ありがとう
345132人目の素数さん:2013/01/30(水) 13:31:12.18
>>343
「率」の意味が分からんのか?
346132人目の素数さん:2013/01/30(水) 13:50:57.80
まあネタだろうが、ウォーズマン理論によれば正しい
347132人目の素数さん:2013/01/30(水) 13:51:28.36
93.75%であってるか?
348132人目の素数さん:2013/01/30(水) 14:10:48.20
99.375%だろ
349132人目の素数さん:2013/01/30(水) 14:17:00.13
(1-0.25^2)*100=93.75
直感的にも99までは行かなそうだけど
350132人目の素数さん:2013/01/30(水) 14:20:04.11
カーテン1枚の時点で100あった光が25になってるんだよ
25の光をさらに75%カットするんだから
通す光は25*0.25で6.25だ
だから遮光率100-6.25=93.75%
って・・・質問した本人はおらんねんな
351132人目の素数さん:2013/01/30(水) 14:32:07.08
>>348どうした、具合でも悪いのか
352132人目の素数さん:2013/01/30(水) 15:09:58.82
>>325
そうなんですか。
では微分積分の初歩的なテキストにも載っていそうですね。
353132人目の素数さん:2013/01/30(水) 15:59:56.09
数列{a[n]}は、 0<a[1]<1 , a[n+1] = {n(a[n])^2+2n+1}/{a[n]+3n} (n=1,2,3,....) を満たしているとする。

(1) 0<a[n]<1 (n=1,2,3,...) であることを示せ。


数学的帰納法で証明しようとしたのですか、よくわからなかったです。

一応、(2)(3)と続きがあるのですが、そっちはとけました。
354132人目の素数さん:2013/01/30(水) 16:25:24.40
>>353
0<a[n]<1 ⇒ 0<a[n]^2<a[n]<1 を使って 0<(分子)<(分母) を示せば 0<(分子)/(分母)<1 が言える。
355132人目の素数さん:2013/01/30(水) 21:00:36.22
          __ノ)-'´ ̄ ̄`ー- 、_
        , '´  _. -‐'''"二ニニ=-`ヽ、
      /   /:::::; -‐''"        `ーノ
     /   /:::::/           \
     /    /::::::/          | | |  |
     |   |:::::/ /     |  | | | |  |
      |   |::/ / / |  | ||  | | ,ハ .| ,ハ|
      |   |/ / / /| ,ハノ| /|ノレ,ニ|ル' 
     |   |  | / / レ',二、レ′ ,ィイ|゙/   私は只の数ヲタなんかとは付き合わないわ。
.     |   \ ∠イ  ,イイ|    ,`-' |      頭が良くて数学が出来てかっこいい人。それが必要条件よ。
     |     l^,人|  ` `-'     ゝ  |        さらに Ann.of Math に論文書けば十分条件にもなるわよ。
      |      ` -'\       ー'  人          一番嫌いなのは論文数を増やすためにくだらない論文を書いて
    |        /(l     __/  ヽ、           良い論文の出版を遅らせるお馬鹿な人。
     |       (:::::`‐-、__  |::::`、     ヒニニヽ、         あなたの論文が Ann of Math に accept される確率は?
    |      / `‐-、::::::::::`‐-、::::\   /,ニニ、\            それとも最近は Inv. Math. の方が上かしら?
   |      |::::::::::::::::::|` -、:::::::,ヘ ̄|'、  ヒニ二、 \
.   |      /::::::::::::::::::|::::::::\/:::O`、::\   | '、   \
   |      /:::::::::::::::::::/:::::::::::::::::::::::::::::'、::::\ノ  ヽ、  |
  |      |:::::/:::::::::/:::::::::::::::::::::::::::::::::::'、',::::'、  /:\__/‐、
  |      |/:::::::::::/::::::::::::::::::::::::::::::::::O::| '、::| く::::::::::::: ̄|
   |     /_..-'´ ̄`ー-、:::::::::::::::::::::::::::::::::::|/:/`‐'::\;;;;;;;_|
   |    |/::::::::::::::::::::::\:::::::::::::::::::::::::::::|::/::::|::::/:::::::::::/
    |   /:::::::::::::::::::::::::::::::::|:::::::::::::::::::::O::|::|::::::|:::::::::::::::/
356132人目の素数さん:2013/01/30(水) 21:09:53.22
>>325
書店で高校数学の教科書や参考書を見ましたが、ベキ乗という言葉は
見つかりませんでした。
指数関数はありましたが、指数関数の微分積分の単元を読めばいいんでしょうか?
googleで検索しても冪乗という項目はあるにしても、ベキ乗と関数とが
離れた単語になっています。まして微分積分などという事柄は出てきません。
ただ、Xのn乗とありましたので、n乗のことをベキ乗と呼ぶのかとも思いました。
それでいいんでしょうか?
累乗という言葉も高校の教科書にも参考書にも見当たりません。
難しくてもいいですから、参考文献を教えていただけませんか?
357132人目の素数さん:2013/01/30(水) 21:14:23.36
          __ノ)-'´ ̄ ̄`ー- 、_
        , '´  _. -‐'''"二ニニ=-`ヽ、
      /   /:::::; -‐''"        `ーノ
     /   /:::::/           \
     /    /::::::/          | | |  |
     |   |:::::/ /     |  | | | |  |
      |   |::/ / / |  | ||  | | ,ハ .| ,ハ|
      |   |/ / / /| ,ハノ| /|ノレ,ニ|ル' 
     |   |  | / / レ',二、レ′ ,ィイ|゙/   私は只の数ヲタなんかとは付き合わないわ。
.     |   \ ∠イ  ,イイ|    ,`-' |      頭が良くて数学が出来てかっこいい人。それが必要条件よ。
     |     l^,人|  ` `-'     ゝ  |        さらに Ann.of Math に論文書けば十分条件にもなるわよ。
      |      ` -'\       ー'  人          一番嫌いなのは論文数を増やすためにくだらない論文を書いて
    |        /(l     __/  ヽ、           良い論文の出版を遅らせるお馬鹿な人。
     |       (:::::`‐-、__  |::::`、     ヒニニヽ、         あなたの論文が Ann of Math に accept される確率は?
    |      / `‐-、::::::::::`‐-、::::\   /,ニニ、\            それとも最近は Inv. Math. の方が上かしら?
   |      |::::::::::::::::::|` -、:::::::,ヘ ̄|'、  ヒニ二、 \
.   |      /::::::::::::::::::|::::::::\/:::O`、::\   | '、   \
   |      /:::::::::::::::::::/:::::::::::::::::::::::::::::'、::::\ノ  ヽ、  |
  |      |:::::/:::::::::/:::::::::::::::::::::::::::::::::::'、',::::'、  /:\__/‐、
  |      |/:::::::::::/::::::::::::::::::::::::::::::::::O::| '、::| く::::::::::::: ̄|
   |     /_..-'´ ̄`ー-、:::::::::::::::::::::::::::::::::::|/:/`‐'::\;;;;;;;_|
   |    |/::::::::::::::::::::::\:::::::::::::::::::::::::::::|::/::::|::::/:::::::::::/
    |   /:::::::::::::::::::::::::::::::::|:::::::::::::::::::::O::|::|::::::|:::::::::::::::/
358132人目の素数さん:2013/01/30(水) 21:20:00.49
>>356
べき乗関数とは指数関数のことだよ。
「冪(べき)」は英語のpowerの訳語として使われた漢字。
359132人目の素数さん:2013/01/30(水) 21:22:12.99
↑累乗も同じ意味。同じものを次々と掛けていくこと。
360132人目の素数さん:2013/01/30(水) 21:25:10.06
>>358
> べき乗関数とは指数関数のことだよ。
>>324を見るにy=x^nのことじゃないのか?
361132人目の素数さん:2013/01/30(水) 21:30:19.33
>>360
> >>358
> > べき乗関数とは指数関数のことだよ。
> >>324を見るにy=x^nのことじゃないのか?
ごめん、その通りだ。
359を書きながら気がつかなかった。
362132人目の素数さん:2013/01/30(水) 21:50:21.12
質問です。

次の式で表される曲線Cについて、以下の問いに答えよ。
C:x=t^3-3t , y=t^4-2t^2

(1) 曲線Cのうち、t≧0の部分を曲線Dとする。Dによって2度以上通過される点は存在しないことを示せ。


解答では、
まずD上の点のx,y座標を表す関数を次のように定めておく。
x(t)=t^3-3t,y(t)=t^4-2t^2 (t≧0)
題を背理法で示す。曲線Dが2回以上通る点が存在すると仮定する。
『このとき、次式を満たす異なる2つの実数a,b(0≦a<b)が存在する。
x(a)=x(b),y(a)=y(b) 』

ここの部分が全く分かりません。
なぜそうなるのですか?
363132人目の素数さん:2013/01/30(水) 21:56:03.65
× そうなる
○ そうした
364132人目の素数さん:2013/01/30(水) 21:58:47.07
>>362
・Dによって2度以上通過される点は存在しない
・パラメータtが異なればそれに対応する点も必ず異なる
・D上の点に対応するtは一意に定まる

表現を変えているだけで全部同じ
365132人目の素数さん:2013/01/30(水) 22:03:10.53
>>362
tがt≧0で動いていくとき点P(t):(x(t),y(t))が動いていく、というのはイメージできるかな?
ある点Aをt=aのとき通過するとすればA=P(a):(x(a),y(a))であり、
そのあとまた点P(t)が動いていってt=bになったときP(b):(x(b),y(b))が再びAになったら、
x(a)=x(b)、y(a)=y(b) だろ。
366132人目の素数さん:2013/01/30(水) 22:08:27.91
>>363-365
理解しました!
>>365さん分かりやすかったです、そのイメージをつかんでいなかったです。
367132人目の素数さん:2013/01/30(水) 23:43:52.26
>>360>>361
y=x^nとは何でしょうか?初めて見ました。
指数関数ではないんですね。
368132人目の素数さん:2013/01/31(木) 00:31:38.20
行列ってなんですか?
369132人目の素数さん:2013/01/31(木) 00:34:02.27
>>367
> >>360>>361
> y=x^nとは何でしょうか?初めて見ました。
> 指数関数ではないんですね。

>>1-3 を見て下さい。  
370132人目の素数さん:2013/01/31(木) 00:34:56.63
>>368
教科書にあたってみなさい。
371132人目の素数さん:2013/01/31(木) 01:05:46.31
f(x)=(x+a)e^(-x^2)はx=1で極値をもつ。このとき、次の問いに答えよ。
ただし、必要があればlim(x→∞)xe^(-x^2)=0を用いてよい。
(1)定数aの値を求めよ。
(2)xについての方程式x+k=ke^(x^2)が実数解をもつような定数kの値の範囲を求めよ

(1)はわかる(a=-1/2)のですが、(2)が全くわかりません。
ヒントでも、教えていただけると嬉しいです。よろしくお願いいたします。
372132人目の素数さん:2013/01/31(木) 01:13:57.26
          __ノ)-'´ ̄ ̄`ー- 、_
        , '´  _. -‐'''"二ニニ=-`ヽ、
      /   /:::::; -‐''"        `ーノ
     /   /:::::/           \
     /    /::::::/          | | |  |
     |   |:::::/ /     |  | | | |  |
      |   |::/ / / |  | ||  | | ,ハ .| ,ハ|
      |   |/ / / /| ,ハノ| /|ノレ,ニ|ル' 
     |   |  | / / レ',二、レ′ ,ィイ|゙/   私は只の数ヲタなんかとは付き合わないわ。
.     |   \ ∠イ  ,イイ|    ,`-' |      頭が良くて数学が出来てかっこいい人。それが必要条件よ。
     |     l^,人|  ` `-'     ゝ  |        さらに Ann.of Math に論文書けば十分条件にもなるわよ。
      |      ` -'\       ー'  人          一番嫌いなのは論文数を増やすためにくだらない論文を書いて
    |        /(l     __/  ヽ、           良い論文の出版を遅らせるお馬鹿な人。
     |       (:::::`‐-、__  |::::`、     ヒニニヽ、         あなたの論文が Ann of Math に accept される確率は?
    |      / `‐-、::::::::::`‐-、::::\   /,ニニ、\            それとも最近は Inv. Math. の方が上かしら?
   |      |::::::::::::::::::|` -、:::::::,ヘ ̄|'、  ヒニ二、 \
.   |      /::::::::::::::::::|::::::::\/:::O`、::\   | '、   \
   |      /:::::::::::::::::::/:::::::::::::::::::::::::::::'、::::\ノ  ヽ、  |
  |      |:::::/:::::::::/:::::::::::::::::::::::::::::::::::'、',::::'、  /:\__/‐、
  |      |/:::::::::::/::::::::::::::::::::::::::::::::::O::| '、::| く::::::::::::: ̄|
   |     /_..-'´ ̄`ー-、:::::::::::::::::::::::::::::::::::|/:/`‐'::\;;;;;;;_|
   |    |/::::::::::::::::::::::\:::::::::::::::::::::::::::::|::/::::|::::/:::::::::::/
    |   /:::::::::::::::::::::::::::::::::|:::::::::::::::::::::O::|::|::::::|:::::::::::::::/
373132人目の素数さん:2013/01/31(木) 01:18:04.17
>>371
kがなんであってもx=0という実解をもつね。
問題文を写し間違ってないかい?
374132人目の素数さん:2013/01/31(木) 01:52:29.79
>>371
式変形してfとaがグラフで交わるような条件を求めればいいんじゃないの?
あと方程式の左辺e^(-x~2)だと思うんだが
それとx=1で極値取るのかねその関数
375132人目の素数さん:2013/01/31(木) 04:06:29.94
>>369
>>1-3
n乗のことですね。累乗と同じということですか。
教科書にはn乗と書かれています。累乗の累の字もありません。
n,n+1,・・・を累乗と呼ぶのかと思っていました。
はっきり書いてあるところがないです。
n乗は指数関数とは違うのですか?同じですか?
部分的に言われてもどれがどれだか判りません。
>>324で書いたことを知りたいんですが?
376132人目の素数さん:2013/01/31(木) 04:12:44.03
>>375
指数関数は変数が肩に乗っている
n次関数は本体が変数で肩に乗っているのは定数
中学高校の教科書参考書を見れば説明が出てないか?
377132人目の素数さん:2013/01/31(木) 07:47:22.53
>>375
用語もわからずに(わからずというより知らずに)次の段階に進むのがおかしい。
378132人目の素数さん:2013/01/31(木) 08:20:42.08
>>375
現れている変数がどこに位置するかを見る。

変数(或いは文字)xを2回かけた式 x*x のことを xの2乗 といい x^2 と書く。
(教科書や印刷物でなら、2 は小さなフォントで x の右肩に書かれるのが普通かもしれない。
 そうであっても、或いはx^2 方式でも、ただの記号だ)
x*x*x は xの3乗x^3、以下4乗x^4、5乗x^5・・・ これらをxのベキ(2ベキ、3ベキ)とも呼び、総称してxの累乗という。

指数関数というときは、a^x の形で、で今度はaは定数でxが変化するものを指す。
xが自然数なら、a^x は a を x回かけたもの(x乗、xベキ)だが、
xが負整数、有理数、実数になると、もう x回かける という言い方はしない。
単にx乗。

こんなところで断片的に聞かずに初歩の教科書(纏め的な参考書ではなく、順に書き進められたもの)にあたることを勧める。

 
379132人目の素数さん:2013/01/31(木) 09:34:21.31
          __ノ)-'´ ̄ ̄`ー- 、_
        , '´  _. -‐'''"二ニニ=-`ヽ、
      /   /:::::; -‐''"        `ーノ
     /   /:::::/           \
     /    /::::::/          | | |  |
     |   |:::::/ /     |  | | | |  |
      |   |::/ / / |  | ||  | | ,ハ .| ,ハ|
      |   |/ / / /| ,ハノ| /|ノレ,ニ|ル' 
     |   |  | / / レ',二、レ′ ,ィイ|゙/   私は只の数ヲタなんかとは付き合わないわ。
.     |   \ ∠イ  ,イイ|    ,`-' |      頭が良くて数学が出来てかっこいい人。それが必要条件よ。
     |     l^,人|  ` `-'     ゝ  |        さらに Ann.of Math に論文書けば十分条件にもなるわよ。
      |      ` -'\       ー'  人          一番嫌いなのは論文数を増やすためにくだらない論文を書いて
    |        /(l     __/  ヽ、           良い論文の出版を遅らせるお馬鹿な人。
     |       (:::::`‐-、__  |::::`、     ヒニニヽ、         あなたの論文が Ann of Math に accept される確率は?
    |      / `‐-、::::::::::`‐-、::::\   /,ニニ、\            それとも最近は Inv. Math. の方が上かしら?
   |      |::::::::::::::::::|` -、:::::::,ヘ ̄|'、  ヒニ二、 \
.   |      /::::::::::::::::::|::::::::\/:::O`、::\   | '、   \
   |      /:::::::::::::::::::/:::::::::::::::::::::::::::::'、::::\ノ  ヽ、  |
  |      |:::::/:::::::::/:::::::::::::::::::::::::::::::::::'、',::::'、  /:\__/‐、
  |      |/:::::::::::/::::::::::::::::::::::::::::::::::O::| '、::| く::::::::::::: ̄|
   |     /_..-'´ ̄`ー-、:::::::::::::::::::::::::::::::::::|/:/`‐'::\;;;;;;;_|
   |    |/::::::::::::::::::::::\:::::::::::::::::::::::::::::|::/::::|::::/:::::::::::/
    |   /:::::::::::::::::::::::::::::::::|:::::::::::::::::::::O::|::|::::::|:::::::::::::::/
380132人目の素数さん:2013/01/31(木) 09:54:37.51
そこからかよ!的な質問は「教科書読め」でじゅうぶん。
長文回答するな。
381132人目の素数さん:2013/01/31(木) 10:13:49.96
失せろ。
382132人目の素数さん:2013/01/31(木) 10:25:35.98
> ・まずは教科書、参考書、web検索などで調べるようにしましょう。(特に基本的な公式など)
383132人目の素数さん:2013/01/31(木) 11:03:38.62
> 324 名前:132人目の素数さん [sage] 投稿日:2013/01/29(火) 19:56:12.64
> 初歩的な質問です。
> 『直観でわかる微分積分』にありました。(66〜70ページ付近)

この時点でもうすでに分かってなかったのかよwww
今までのやりとりはムダじゃねーかw
384132人目の素数さん:2013/01/31(木) 11:27:33.84
>>378
> こんなところで断片的に聞かずに初歩の教科書(纏め的な参考書ではなく、順に書き進められたもの)にあたることを勧める。
385132人目の素数さん:2013/01/31(木) 12:43:08.60
数列の和を求める問題です。
n^2+(n+1)^2+(n+2)^2+…+(2n-1)^2
=n^2+(n+1)^2+(n+2)^2+…+{n+(n-1)}^2
=Σ(n+k)^2[k=0からn-1]
={n(2n-1)(7n-1)}/6

2行目はkのところが(k-1)でも合っていますか?
それとΣ(n+k)^2[k=0からn-1]で計算したとき、k=0からになるのはなぜですか?
386132人目の素数さん:2013/01/31(木) 12:47:56.20
> それとk=0からなのは初項に1がないからですか?
> 教えてください

やっぱりお前分かって無いじゃん

ここで類題ごとにひとつずつ訊ねるのはムダだよ
もっと別の理解したほうがいい
387132人目の素数さん:2013/01/31(木) 13:27:26.42
>>386類題まで聞くつもりはなかったのですが新たにわからないことがあったので
何かおすすめの参考書でもありますか?

補足するとkを(k-1)に置き換えて計算するならば、Σ(n+k)^2[k=-1からn-1]になると思ったのですがなぜ模範解答はΣ(n+k)^2[k=0からn-1]なのかわからないということです。
388132人目の素数さん:2013/01/31(木) 13:33:10.75
えっ、Σ自体分かってないじゃんΣ(゚Д゚)
389132人目の素数さん:2013/01/31(木) 13:45:43.38
>>386
無駄ってなんだよ、一度に丸投げされるよりましだろうが


>>387
連続的な数列なんだからn-1までとなればΣを用いてもn-1まで
適当な数入れて確かめてみるといい
390132人目の素数さん:2013/01/31(木) 13:50:10.13
すみません、力を貸してください・・・

三角形abcは∠abc=90°の直角二等辺三角形であり、辺bcの中点をdとする。
辺ac上に点e、辺ab上にfがあり、de=3、ef=4、∠def=θ、bd=xとする。
(6)xを求めよ

って言う問題がわかりません・・・

一応(1)〜(5)ものっけときます。
誘導になってるんじゃないかと睨んでいるのですが・・・

(1)∠afeをθを用いて表せ(2)ehの長さをsinθの簡単な式で表せ(3)ceの長さをsinθの簡単な式で表せ(4)aeの長さをsinθの簡単な式で表せ(5)sinθをxの簡単な式で表せ

一応私が出した答え・・・(合っているかは不明)
(1)(180−θ)°(2)3sinθ(3)3√2sinθ(4)4√2sinθ(5)2x/7(6)21/√37
(6)も出てはいるのですが、(1)〜(5)を全部使ってないし、無理があると思います。
めんどくさそうで有理化してないしやっぱりちがうかなぁ。
(三角形cedでch+hd=xcecos45°+edcos45°=x
 3√2sinθ×1/√2+3cosθ=x3sinθ+3cosθ=x
 3×2/7x+3cosθ=x3cosθ=1/7xcosθ=1/21x
 sin^2θ+cos^2θ=1よりx^2=21^2/37x=21/√37って出しました)

答えがないので解答はわかりません・・・解答が合ってるかどうかは気になさらないでいいので解法お願いします(´;ω;`)
(TAの範囲で解けるはずです)
391132人目の素数さん:2013/01/31(木) 14:10:11.24
>>391(2)のHってなんだい?
392132人目の素数さん:2013/01/31(木) 14:11:11.24
安価ミス>>390です
393132人目の素数さん:2013/01/31(木) 14:18:14.41
(三角形cedでch+hd=x
cecos45°+edcos45°=x
 3√2sinθ×1/√2+3cosθ=x
3sinθ+3cosθ=x
 3×2/7x+3cosθ=x
3cosθ=1/7x
cosθ=1/21x
 sin^2θ+cos^2θ=1より
x^2=21^2/37
x=21/√37って出しました) です。
改行しすぎで投稿できなかったので改行減らしたらへんなことになってました。すみません。
394132人目の素数さん:2013/01/31(木) 14:20:22.21
あ、hは私が勝手においた点です!
eh⊥abになるようにab上に置きました。
395132人目の素数さん:2013/01/31(木) 14:21:24.02
まちがえましたあああ
eh⊥cb,cb上です><
396132人目の素数さん:2013/01/31(木) 14:23:57.22
勝手に置いたのは点gでした。eg⊥ab,ab上に。
(出てきてないけど(4)を解く時に使いました)
397132人目の素数さん:2013/01/31(木) 14:37:54.29
書き込む前に確認できないのかね
うぜえんだが
398132人目の素数さん:2013/01/31(木) 14:58:58.64
一体どんな器具使ってるのやら
399132人目の素数さん:2013/01/31(木) 15:24:05.54
>>397
気をつけてはいたのですが。申し訳ありませんでした。
400132人目の素数さん:2013/01/31(木) 15:31:24.94
sageを覚えましょう
401132人目の素数さん:2013/01/31(木) 16:06:30.17
          __ノ)-'´ ̄ ̄`ー- 、_
        , '´  _. -‐'''"二ニニ=-`ヽ、
      /   /:::::; -‐''"        `ーノ
     /   /:::::/           \
     /    /::::::/          | | |  |
     |   |:::::/ /     |  | | | |  |
      |   |::/ / / |  | ||  | | ,ハ .| ,ハ|
      |   |/ / / /| ,ハノ| /|ノレ,ニ|ル' 
     |   |  | / / レ',二、レ′ ,ィイ|゙/   私は只の数ヲタなんかとは付き合わないわ。
.     |   \ ∠イ  ,イイ|    ,`-' |      頭が良くて数学が出来てかっこいい人。それが必要条件よ。
     |     l^,人|  ` `-'     ゝ  |        さらに Ann.of Math に論文書けば十分条件にもなるわよ。
      |      ` -'\       ー'  人          一番嫌いなのは論文数を増やすためにくだらない論文を書いて
    |        /(l     __/  ヽ、           良い論文の出版を遅らせるお馬鹿な人。
     |       (:::::`‐-、__  |::::`、     ヒニニヽ、         あなたの論文が Ann of Math に accept される確率は?
    |      / `‐-、::::::::::`‐-、::::\   /,ニニ、\            それとも最近は Inv. Math. の方が上かしら?
   |      |::::::::::::::::::|` -、:::::::,ヘ ̄|'、  ヒニ二、 \
.   |      /::::::::::::::::::|::::::::\/:::O`、::\   | '、   \
   |      /:::::::::::::::::::/:::::::::::::::::::::::::::::'、::::\ノ  ヽ、  |
  |      |:::::/:::::::::/:::::::::::::::::::::::::::::::::::'、',::::'、  /:\__/‐、
  |      |/:::::::::::/::::::::::::::::::::::::::::::::::O::| '、::| く::::::::::::: ̄|
   |     /_..-'´ ̄`ー-、:::::::::::::::::::::::::::::::::::|/:/`‐'::\;;;;;;;_|
   |    |/::::::::::::::::::::::\:::::::::::::::::::::::::::::|::/::::|::::/:::::::::::/
    |   /:::::::::::::::::::::::::::::::::|:::::::::::::::::::::O::|::|::::::|:::::::::::::::/
402132人目の素数さん:2013/01/31(木) 17:04:12.77
>>376
ベキ乗という項目は教科書に載っていないのです。
>>377
いわゆる数学事典というものにもベキ乗関数のことは全く載っていません。
>>378
ベキ乗と指数のちがいはわかりました。ありがとうございます。

ベキ乗関数の微分積分の解説は大学数学の参考書に載っていますか?
微分積分の基礎テキストは平行して読んでいますが、そこには載っていないので、
難しいテキストに載っていればよいのですが。

>>380
教科書に載っていないのです。
403132人目の素数さん:2013/01/31(木) 17:25:35.68
383:132人目の素数さんsage2013/01/31(木) 11:03:38.62
> 324 名前:132人目の素数さん [sage] 投稿日:2013/01/29(火) 19:56:12.64
> 初歩的な質問です。
> 『直観でわかる微分積分』にありました。(66〜70ページ付近)

この時点でもうすでに分かってなかったのかよwww
今までのやりとりはムダじゃねーかw


386:132人目の素数さんsage2013/01/31(木) 12:47:56.20
> それとk=0からなのは初項に1がないからですか?
> 教えてください

やっぱりお前分かって無いじゃん

ここで類題ごとにひとつずつ訊ねるのはムダだよ
もっと別の理解したほうがいい
404132人目の素数さん:2013/01/31(木) 19:00:35.36
120で割ると1余る ⇔ 2,3,5のどれで割っても1余る (∵ 120=2^3*3*%)

という認識で合っていますか?
405132人目の素数さん:2013/01/31(木) 19:02:11.83
31ディスってんなよ
406132人目の素数さん:2013/01/31(木) 19:26:27.78
ABを直径とする円がある。
線分AB上に点Hをとり、Hを通りABに垂直な直線と円の交点をC, Dとおく。
Bにおける円の接線と直線ACの交点をP、Aにおける円の接線と直線BDの交点をQとする。
PHQが一直線になることを示すにはどうるればいいでしょうか。
407132人目の素数さん:2013/01/31(木) 19:36:21.82
丸投げじゃん
408132人目の素数さん:2013/01/31(木) 19:38:44.83
-1.5の整数部分は-2小数部分は+0.5ですか?
409132人目の素数さん:2013/01/31(木) 19:50:55.71
          __ノ)-'´ ̄ ̄`ー- 、_
        , '´  _. -‐'''"二ニニ=-`ヽ、
      /   /:::::; -‐''"        `ーノ
     /   /:::::/           \
     /    /::::::/          | | |  |
     |   |:::::/ /     |  | | | |  |
      |   |::/ / / |  | ||  | | ,ハ .| ,ハ|
      |   |/ / / /| ,ハノ| /|ノレ,ニ|ル' 
     |   |  | / / レ',二、レ′ ,ィイ|゙/   私は只の数ヲタなんかとは付き合わないわ。
.     |   \ ∠イ  ,イイ|    ,`-' |      頭が良くて数学が出来てかっこいい人。それが必要条件よ。
     |     l^,人|  ` `-'     ゝ  |        さらに Ann.of Math に論文書けば十分条件にもなるわよ。
      |      ` -'\       ー'  人          一番嫌いなのは論文数を増やすためにくだらない論文を書いて
    |        /(l     __/  ヽ、           良い論文の出版を遅らせるお馬鹿な人。
     |       (:::::`‐-、__  |::::`、     ヒニニヽ、         あなたの論文が Ann of Math に accept される確率は?
    |      / `‐-、::::::::::`‐-、::::\   /,ニニ、\            それとも最近は Inv. Math. の方が上かしら?
   |      |::::::::::::::::::|` -、:::::::,ヘ ̄|'、  ヒニ二、 \
.   |      /::::::::::::::::::|::::::::\/:::O`、::\   | '、   \
   |      /:::::::::::::::::::/:::::::::::::::::::::::::::::'、::::\ノ  ヽ、  |
  |      |:::::/:::::::::/:::::::::::::::::::::::::::::::::::'、',::::'、  /:\__/‐、
  |      |/:::::::::::/::::::::::::::::::::::::::::::::::O::| '、::| く::::::::::::: ̄|
   |     /_..-'´ ̄`ー-、:::::::::::::::::::::::::::::::::::|/:/`‐'::\;;;;;;;_|
   |    |/::::::::::::::::::::::\:::::::::::::::::::::::::::::|::/::::|::::/:::::::::::/
    |   /:::::::::::::::::::::::::::::::::|:::::::::::::::::::::O::|::|::::::|:::::::::::::::/
410132人目の素数さん:2013/01/31(木) 19:51:24.92
>>408
定義系に拠るとしか
なので定義を持って来い
411132人目の素数さん:2013/01/31(木) 19:53:44.66
>>406
座標設定して計算ごり押しでも解ける
412132人目の素数さん:2013/01/31(木) 19:55:04.59
> 整数部分は-2

こりゃ一体どういうことだよ
説明しろ
413132人目の素数さん:2013/01/31(木) 19:59:37.75
>>408
yes
414132人目の素数さん:2013/01/31(木) 20:01:54.76
>>406 これ正しいか?
415132人目の素数さん:2013/01/31(木) 20:10:10.92
nを正の整数、aを実数とする。すべての整数mに対して、
m^2-(a-1)m+an^2/2n+1>0
が成り立つようなaの範囲をnを用いて表せ
416132人目の素数さん:2013/01/31(木) 20:13:52.55
偉そうに命令するな
417132人目の素数さん:2013/01/31(木) 20:14:56.76
>>415
> nを正の整数、aを実数とする。すべての整数mに対して、
> m^2-(a-1)m+an^2/2n+1>0
> が成り立つようなaの範囲をnを用いて表せ
式中の /  の分子、分母をはっきりさせよ。
418132人目の素数さん:2013/01/31(木) 20:16:07.26
a,b,pを実数の定数として2つの方程式
x^2+ax+b=0....@
x^2+(a-2p)x-ap+b=0....A
@が異なる実数解をもつとする。。p=0でないき、@の2つの実数解k,l(k<l)
とAの2つの実数解m,n(m<n)の大小関係を比較して小さい順に並べよ
419132人目の素数さん:2013/01/31(木) 20:16:59.96
断る
420132人目の素数さん:2013/01/31(木) 20:22:41.96
問題書けば解答が自動的に得られると勘違いしてるやつが多いが>>1も読めんのか
知恵遅れ行っとけks
421132人目の素数さん:2013/01/31(木) 20:27:25.19
m^2-(a-1)m+a×n×n÷(2n+1)>0
です
422132人目の素数さん:2013/01/31(木) 20:36:21.86
>>418
丸2の左辺を f(x) とおくとき、 f(k)とf(l)の符号を調べて、
y=f(x)のグラフの概形を考える。
423132人目の素数さん:2013/01/31(木) 20:36:45.47
f(x)=2cos(x^2)を微分したf'(x)はいくつになりますか教えてください
424132人目の素数さん:2013/01/31(木) 20:44:38.53
すみません解決しました
425132人目の素数さん:2013/01/31(木) 20:46:45.43
>>424
騙るな
426132人目の素数さん:2013/01/31(木) 20:52:38.14
>>415
東大97年
427132人目の素数さん:2013/01/31(木) 20:53:36.36
>>423
導関数の定義から一意に定まるからひとつ
428132人目の素数さん:2013/01/31(木) 21:12:28.14
429132人目の素数さん:2013/01/31(木) 21:13:04.50
a,b,pを実数の定数として2つの方程式
x^2+ax+b=0....@
x^2+(a-2p)x-ap+b=0....A
@が異なる実数解をもつとする。。p=0でないき、@の2つの実数解k,l(k<l)
とAの2つの実数解m,n(m<n)の大小関係を比較して小さい順に並べよ

お願いします
430132人目の素数さん:2013/01/31(木) 21:14:59.80
しつこいぞバカ
431132人目の素数さん:2013/01/31(木) 21:23:46.83
http://i.imgur.com/23be5ja.jpg
すみません
この画像の角xが60°になるんですけど
なんでそうなるか分かる人いますか?
432132人目の素数さん:2013/01/31(木) 21:23:49.69
433132人目の素数さん:2013/01/31(木) 21:24:47.53
434132人目の素数さん:2013/01/31(木) 21:27:11.17
>>431
いるよ
435132人目の素数さん:2013/01/31(木) 21:28:15.00
>>431
追記、説明お願いします
436132人目の素数さん:2013/01/31(木) 21:34:07.89
          __ノ)-'´ ̄ ̄`ー- 、_
        , '´  _. -‐'''"二ニニ=-`ヽ、
      /   /:::::; -‐''"        `ーノ
     /   /:::::/           \
     /    /::::::/          | | |  |
     |   |:::::/ /     |  | | | |  |
      |   |::/ / / |  | ||  | | ,ハ .| ,ハ|
      |   |/ / / /| ,ハノ| /|ノレ,ニ|ル' 
     |   |  | / / レ',二、レ′ ,ィイ|゙/   私は只の数ヲタなんかとは付き合わないわ。
.     |   \ ∠イ  ,イイ|    ,`-' |      頭が良くて数学が出来てかっこいい人。それが必要条件よ。
     |     l^,人|  ` `-'     ゝ  |        さらに Ann.of Math に論文書けば十分条件にもなるわよ。
      |      ` -'\       ー'  人          一番嫌いなのは論文数を増やすためにくだらない論文を書いて
    |        /(l     __/  ヽ、           良い論文の出版を遅らせるお馬鹿な人。
     |       (:::::`‐-、__  |::::`、     ヒニニヽ、         あなたの論文が Ann of Math に accept される確率は?
    |      / `‐-、::::::::::`‐-、::::\   /,ニニ、\            それとも最近は Inv. Math. の方が上かしら?
   |      |::::::::::::::::::|` -、:::::::,ヘ ̄|'、  ヒニ二、 \
.   |      /::::::::::::::::::|::::::::\/:::O`、::\   | '、   \
   |      /:::::::::::::::::::/:::::::::::::::::::::::::::::'、::::\ノ  ヽ、  |
  |      |:::::/:::::::::/:::::::::::::::::::::::::::::::::::'、',::::'、  /:\__/‐、
  |      |/:::::::::::/::::::::::::::::::::::::::::::::::O::| '、::| く::::::::::::: ̄|
   |     /_..-'´ ̄`ー-、:::::::::::::::::::::::::::::::::::|/:/`‐'::\;;;;;;;_|
   |    |/::::::::::::::::::::::\:::::::::::::::::::::::::::::|::/::::|::::/:::::::::::/
    |   /:::::::::::::::::::::::::::::::::|:::::::::::::::::::::O::|::|::::::|:::::::::::::::/
437132人目の素数さん:2013/01/31(木) 21:40:21.08
みえた!一見正方形に見えるが実は菱型
438132人目の素数さん:2013/01/31(木) 21:48:01.46
http://beebee2see.appspot.com/i/azuYubfgBww.jpg

なぜ、PQ^2がこのように表されるのか分かりません。教えてください。
439132人目の素数さん:2013/01/31(木) 21:50:15.15
代入しただけじゃないか
440132人目の素数さん:2013/01/31(木) 21:51:14.11
>>431
余弦定理を2回使うことで、問題のxを頂角とする三角形が正三角形であることが分る。
441132人目の素数さん:2013/01/31(木) 21:55:08.49
>>439
2点間の距離公式とは若干違ってる気がします。なんか別のやり方ですか?
442132人目の素数さん:2013/01/31(木) 22:00:49.07
>>441
三平方
443132人目の素数さん:2013/01/31(木) 22:02:22.64
>>440
なるほど、分かりました
ありがとうございます。
444132人目の素数さん:2013/01/31(木) 22:04:25.30
>>441
http://i.imgur.com/zWUmFiH.jpg
傾きとx座標の差から2点間の距離を求める方法.
445132人目の素数さん:2013/01/31(木) 22:04:42.50
>>431
60°とは限らないんじゃないか?
446132人目の素数さん:2013/01/31(木) 22:06:18.17
>>444
なるほど、そんな手があったんですか
すごく楽にできますね...
ありがとうございます
447132人目の素数さん:2013/01/31(木) 22:07:54.94
>>438
あ、あと後半の式の絶対値は必要ですか?
2乗してるのでいらない気がするんですが
448132人目の素数さん:2013/01/31(木) 22:10:50.40
2乗だから括弧でも絶対値記号でも同じ
449132人目の素数さん:2013/01/31(木) 22:12:40.22
なら解答はなぜ分けたんでしょうか?
450132人目の素数さん:2013/01/31(木) 22:15:41.94
というか>>431は(外の菱形が正方形ならだが)
上の三角形が正三角形なら下の三角形の底角が15度になることは
小学校の算数でも左右の二等辺三角形を介して容易にいえる
そこから攻めちゃだめかいな
451132人目の素数さん:2013/01/31(木) 22:16:14.26
行列の対象変換の公式を記述式で使うのはNGですか?
452132人目の素数さん:2013/01/31(木) 22:18:25.61
受験板できけ
453132人目の素数さん:2013/01/31(木) 22:22:31.25
>>449
値は一致するが、あくまでも「距離(ここではx座標の差の絶対値)」の2乗であり、「x座標の差」の2乗ではないことを意識しているのでは?
454132人目の素数さん:2013/01/31(木) 22:23:12.11
>>431
図は正方形としてバカ正直な解答例を。
正方形の頂点を左上から左周りでA,B,C,Dとし、頂角xを与える内部の点をE
EからBCに下した垂線の足をFとする。
正方形の一辺の長さを2a、BE=x、EF=yとおけば、
(1)x^2=y^2+a^2
∠EBF=15°に余弦定理を適用して、
(2)y^2=x^2+a^2-2axcos(15°)
(1)と(2)から y^2を消去してからxについて解くと
(3)x=a/cos(15°)
僂DEに余弦定理を適用して
(4)ED^2=x^2+4a^2-4cos(75°)
(3),(4)から
ED^2=a^2((1/cos(15°))^2+4-4cos(75°)/cos(15°))
15°=45°-30°、75°=45°+30°を使えば
cos(15°)=(√6-√2)/4、cos(75°)=(√6+√2)/4 であるから
これらより、ED^2=4a^2、即ちED=2a。
よって僊EDは正三角形であるからx=60°である。
455132人目の素数さん:2013/01/31(木) 22:23:44.89
>>453
ありがとうございました!
456132人目の素数さん:2013/01/31(木) 22:51:33.33
>>415
まず、mが実数の場合に常に成立する条件として(当然整数mについても成り立つ)
判別式<0から 1/(2n+1)<a<2n+1。これは求めるaの範囲の一部である。
特にm=0の時にも成立することから a>0が必要。
また特にm=nの時にも成立することから a<2n+1が必要だが、これは既に得られている。
残りは 0<a≦1/(2n+1)のときであるが、このときaは0<a<1を満たすことから-1/2<(a-1)/2<0となり
左辺の2次式の軸の位置から、m=0とm=-1の時に左辺の値が正であればよいが、
m=0の時は、済んでいる。あとm=-1のとき左辺=a+an^2/(2n+1)>0でよい。
以上から 0<a<2n+1
457132人目の素数さん:2013/01/31(木) 22:54:15.40
>>456十分性は?
458132人目の素数さん:2013/01/31(木) 23:06:25.67
読み取れ
459132人目の素数さん:2013/01/31(木) 23:22:24.64
>>378
あなたのレスでだいたい判りました。
それに当たる関数を探してみます。
ありがとうございました。
>>403
レスをコピペする意味がわかりません。初歩的ではないということですか?
初歩的なのか初歩的でないのか不明であることも含めて初歩的と表現しました。
あなたがたが初歩的であると解釈したのなら、初歩的であると言えます。
それも含めて伺ったのです。
つまり高校数学の全部を当たれといわけではない以上、どの部分が初歩的であるのか、
初歩的でないのか、判ろうはずがありません。
460132人目の素数さん:2013/01/31(木) 23:23:20.41
a,b,pを実数の定数として2つの方程式
x^2+ax+b=0....@
x^2+(a-2p)x-ap+b=0....A
@が異なる実数解をもつとする。。p=0でないき、@の2つの実数解k,l(k<l)
とAの2つの実数解m,n(m<n)の大小関係を比較して小さい順に並べよ
461132人目の素数さん:2013/01/31(木) 23:24:16.13
462132人目の素数さん:2013/01/31(木) 23:26:21.46
これは面白い
463132人目の素数さん:2013/01/31(木) 23:28:46.76
464132人目の素数さん:2013/01/31(木) 23:32:28.12
465132人目の素数さん:2013/01/31(木) 23:33:46.02
かかかか
466132人目の素数さん:2013/01/31(木) 23:34:31.34
ててて
467132人目の素数さん:2013/01/31(木) 23:35:28.36
すすすすす
468132人目の素数さん:2013/01/31(木) 23:36:38.90
めめめめめめめめ
469132人目の素数さん:2013/01/31(木) 23:38:37.93
dxyf
470132人目の素数さん:2013/02/01(金) 00:20:14.30
471132人目の素数さん:2013/02/01(金) 00:32:09.60
>>457
一応要点を書き直して。
m=0とm=nとから0<a<2n+1が必要、が出る。
このうち、1/(2n+1)<a<2n+1 が十分なことは判別式が負になることから出る。
残りの0<a≦1/(2n+1)なら、0<a<1となるから-1/2<(a-1)/2<0。
2次関数が、軸(それは整数値ではない)を挟む、m=-1とm=0で正だから、任意の整数で正。
結局、0<a<2n+1は十分。
472132人目の素数さん:2013/02/01(金) 04:12:13.92
>>427
面白い!
473132人目の素数さん:2013/02/01(金) 04:27:05.62
寒いな
474132人目の素数さん:2013/02/01(金) 05:57:30.32
なんで画像サイズ小さくしない奴多いの?
475132人目の素数さん:2013/02/01(金) 09:35:55.69
cos(sinx) > sin(cosx) って常に成り立ちますか
476132人目の素数さん:2013/02/01(金) 09:45:23.13
はい
常に成り立ちます
477132人目の素数さん:2013/02/01(金) 09:48:25.82
うん
478132人目の素数さん:2013/02/01(金) 12:30:36.69
常識だな
479132人目の素数さん:2013/02/01(金) 12:37:40.17
証明はどうするんですか?
480132人目の素数さん:2013/02/01(金) 12:41:32.10
俺がなるといったらなるんだよ
カラスは白だし猫はワンと鳴く
481132人目の素数さん:2013/02/01(金) 13:03:24.75
482132人目の素数さん:2013/02/01(金) 13:15:18.29
ニュートンは数学者ですか?
483132人目の素数さん:2013/02/01(金) 13:18:45.34
ドシロウト向け科学雑誌
484132人目の素数さん:2013/02/01(金) 13:54:07.85
√2<π/2 だから sin x+cos x=√2 sin(x+π/4)<π/2
∴ cos x<π/2−sin x
sin x>0 の範囲では −π/2<−1<cos x<π/2−sin x<π/2
−π/2〜π/2 の範囲で sin は単調だから
sin(cos x)<sin(π/2−sin x)=cos(sin x)
sin x<0 の範囲はご自分で
485132人目の素数さん:2013/02/01(金) 18:15:13.37
>>475
cos(sinx)-sin(cosx)=cos(sinx)-cos(π/2-cosx)
あとは和積とπ/2>√2でおしまい
486132人目の素数さん:2013/02/01(金) 18:28:54.95
          __ノ)-'´ ̄ ̄`ー- 、_
        , '´  _. -‐'''"二ニニ=-`ヽ、
      /   /:::::; -‐''"        `ーノ
     /   /:::::/           \
     /    /::::::/          | | |  |
     |   |:::::/ /     |  | | | |  |
      |   |::/ / / |  | ||  | | ,ハ .| ,ハ|
      |   |/ / / /| ,ハノ| /|ノレ,ニ|ル' 
     |   |  | / / レ',二、レ′ ,ィイ|゙/   私は只の数ヲタなんかとは付き合わないわ。
.     |   \ ∠イ  ,イイ|    ,`-' |      頭が良くて数学が出来てかっこいい人。それが必要条件よ。
     |     l^,人|  ` `-'     ゝ  |        さらに Ann.of Math に論文書けば十分条件にもなるわよ。
      |      ` -'\       ー'  人          一番嫌いなのは論文数を増やすためにくだらない論文を書いて
    |        /(l     __/  ヽ、           良い論文の出版を遅らせるお馬鹿な人。
     |       (:::::`‐-、__  |::::`、     ヒニニヽ、         あなたの論文が Ann of Math に accept される確率は?
    |      / `‐-、::::::::::`‐-、::::\   /,ニニ、\            それとも最近は Inv. Math. の方が上かしら?
   |      |::::::::::::::::::|` -、:::::::,ヘ ̄|'、  ヒニ二、 \
.   |      /::::::::::::::::::|::::::::\/:::O`、::\   | '、   \
   |      /:::::::::::::::::::/:::::::::::::::::::::::::::::'、::::\ノ  ヽ、  |
  |      |:::::/:::::::::/:::::::::::::::::::::::::::::::::::'、',::::'、  /:\__/‐、
  |      |/:::::::::::/::::::::::::::::::::::::::::::::::O::| '、::| く::::::::::::: ̄|
   |     /_..-'´ ̄`ー-、:::::::::::::::::::::::::::::::::::|/:/`‐'::\;;;;;;;_|
   |    |/::::::::::::::::::::::\:::::::::::::::::::::::::::::|::/::::|::::/:::::::::::/
    |   /:::::::::::::::::::::::::::::::::|:::::::::::::::::::::O::|::|::::::|:::::::::::::::/
487132人目の素数さん:2013/02/01(金) 20:09:43.55
488132人目の素数さん:2013/02/01(金) 20:56:06.62
ju0j
489132人目の素数さん:2013/02/01(金) 20:56:53.50
まんかな
490132人目の素数さん:2013/02/01(金) 21:04:40.88
          __ノ)-'´ ̄ ̄`ー- 、_
        , '´  _. -‐'''"二ニニ=-`ヽ、
      /   /:::::; -‐''"        `ーノ
     /   /:::::/           \
     /    /::::::/          | | |  |
     |   |:::::/ /     |  | | | |  |
      |   |::/ / / |  | ||  | | ,ハ .| ,ハ|
      |   |/ / / /| ,ハノ| /|ノレ,ニ|ル' 
     |   |  | / / レ',二、レ′ ,ィイ|゙/   私は只の数ヲタなんかとは付き合わないわ。
.     |   \ ∠イ  ,イイ|    ,`-' |      頭が良くて数学が出来てかっこいい人。それが必要条件よ。
     |     l^,人|  ` `-'     ゝ  |        さらに Ann.of Math に論文書けば十分条件にもなるわよ。
      |      ` -'\       ー'  人          一番嫌いなのは論文数を増やすためにくだらない論文を書いて
    |        /(l     __/  ヽ、           良い論文の出版を遅らせるお馬鹿な人。
     |       (:::::`‐-、__  |::::`、     ヒニニヽ、         あなたの論文が Ann of Math に accept される確率は?
    |      / `‐-、::::::::::`‐-、::::\   /,ニニ、\            それとも最近は Inv. Math. の方が上かしら?
   |      |::::::::::::::::::|` -、:::::::,ヘ ̄|'、  ヒニ二、 \
.   |      /::::::::::::::::::|::::::::\/:::O`、::\   | '、   \
   |      /:::::::::::::::::::/:::::::::::::::::::::::::::::'、::::\ノ  ヽ、  |
  |      |:::::/:::::::::/:::::::::::::::::::::::::::::::::::'、',::::'、  /:\__/‐、
  |      |/:::::::::::/::::::::::::::::::::::::::::::::::O::| '、::| く::::::::::::: ̄|
   |     /_..-'´ ̄`ー-、:::::::::::::::::::::::::::::::::::|/:/`‐'::\;;;;;;;_|
   |    |/::::::::::::::::::::::\:::::::::::::::::::::::::::::|::/::::|::::/:::::::::::/
    |   /:::::::::::::::::::::::::::::::::|:::::::::::::::::::::O::|::|::::::|:::::::::::::::/
491132人目の素数さん:2013/02/01(金) 21:47:15.17
11*11=121
12*12=142
13*13=169をみてビビっときてn^2+n=(n+1)-(n+1)ってしたら
ただの因数分解の式になった(´・ω・`)
このビビットした感覚が恐ろしく気持ちよかったんだけど
数学得意な人はみんなこんな感じなんですか
492132人目の素数さん:2013/02/01(金) 21:49:38.13
>>491
> 12*12=142
> n^2+n=(n+1)-(n+1)
間違ってる
493132人目の素数さん:2013/02/01(金) 21:57:35.06
1辺が10cmの正三角形ABCにおいて、辺AB上を動くPと辺BC上を動くQがあります。
Pは、点Aから点Bに向かって毎分1cmの速さで動き、
Qは、点Bから点Cに向かってPの2倍の速さで動きます。
PQ間の距離が最小になるのは、スタートしてから何分後ですか。

という問題の解き方で
PQ^2=(10−x)^2+(2x)^2−2(10−x)×2x×cos60
   =100−20x+x^2+4x^2−2(10−x)×2x×(1/2)
   =7x^2−40x+100
までは理解できたんですが

7x^2−40x+100
=7(x^2−(40/7)x+(20/7)^2)−20^2/7+100
=7(x-20/7)^2+300/7
この手順が解りません
どうしてこうなるのか順を追って解説して貰えないでしょうか
お願いします
494132人目の素数さん:2013/02/01(金) 22:01:34.46
平方完成しただけじゃん
教科書百万回嫁
495132人目の素数さん:2013/02/01(金) 22:09:21.08
          __ノ)-'´ ̄ ̄`ー- 、_
        , '´  _. -‐'''"二ニニ=-`ヽ、
      /   /:::::; -‐''"        `ーノ
     /   /:::::/           \
     /    /::::::/          | | |  |
     |   |:::::/ /     |  | | | |  |
      |   |::/ / / |  | ||  | | ,ハ .| ,ハ|
      |   |/ / / /| ,ハノ| /|ノレ,ニ|ル' 
     |   |  | / / レ',二、レ′ ,ィイ|゙/   私は只の数ヲタなんかとは付き合わないわ。
.     |   \ ∠イ  ,イイ|    ,`-' |      頭が良くて数学が出来てかっこいい人。それが必要条件よ。
     |     l^,人|  ` `-'     ゝ  |        さらに Ann.of Math に論文書けば十分条件にもなるわよ。
      |      ` -'\       ー'  人          一番嫌いなのは論文数を増やすためにくだらない論文を書いて
    |        /(l     __/  ヽ、           良い論文の出版を遅らせるお馬鹿な人。
     |       (:::::`‐-、__  |::::`、     ヒニニヽ、         あなたの論文が Ann of Math に accept される確率は?
    |      / `‐-、::::::::::`‐-、::::\   /,ニニ、\            それとも最近は Inv. Math. の方が上かしら?
   |      |::::::::::::::::::|` -、:::::::,ヘ ̄|'、  ヒニ二、 \
.   |      /::::::::::::::::::|::::::::\/:::O`、::\   | '、   \
   |      /:::::::::::::::::::/:::::::::::::::::::::::::::::'、::::\ノ  ヽ、  |
  |      |:::::/:::::::::/:::::::::::::::::::::::::::::::::::'、',::::'、  /:\__/‐、
  |      |/:::::::::::/::::::::::::::::::::::::::::::::::O::| '、::| く::::::::::::: ̄|
   |     /_..-'´ ̄`ー-、:::::::::::::::::::::::::::::::::::|/:/`‐'::\;;;;;;;_|
   |    |/::::::::::::::::::::::\:::::::::::::::::::::::::::::|::/::::|::::/:::::::::::/
    |   /:::::::::::::::::::::::::::::::::|:::::::::::::::::::::O::|::|::::::|:::::::::::::::/
496132人目の素数さん:2013/02/01(金) 22:29:14.37
なきなんきの
497493:2013/02/01(金) 22:47:53.92
>>494
卒業から15年なので教科書も無く
工業科で数学は流しただけみたいなものだったので
習った覚えも無く名前すら知りませんでした
平方完成と言うんですね
調べてみます
ありがとうございました
498132人目の素数さん:2013/02/01(金) 23:16:45.03
まこきりるこ
499132人目の素数さん:2013/02/01(金) 23:26:09.32
数学たのしいです
500132人目の素数さん:2013/02/02(土) 01:17:28.79
π/2>√2
を√2=1.4142... π=3.14159...
を使わずに証明できますか?
501132人目の素数さん:2013/02/02(土) 01:25:43.91
π>3=√9>√8=2√2
最初の不等式は内接三角形を考えて得られる
502132人目の素数さん:2013/02/02(土) 01:27:12.08
>>500
弧は弦より長い
503132人目の素数さん:2013/02/02(土) 01:27:46.09
あちゃ、円と内接六角形ね
504132人目の素数さん:2013/02/02(土) 01:28:03.97
半径1角pi/2の弧ABの長さはpi/2
ABの長さは√2
505132人目の素数さん:2013/02/02(土) 01:46:38.78
なるほどありがとうございます
506狢 ◆yEy4lYsULH68 :2013/02/02(土) 11:48:11.96
低脳は無駄。

507132人目の素数さん:2013/02/02(土) 12:20:38.02
y=sin(θ+5/6π)-cosθの最小値、最大値を求めよ

これで、まずsin(θ+5/6π)の部分を加法定理でバラしました。
=sinθcosθ5/6π+cosθsin5/6π
答えでは、ここから
=-√3/2sinθ+1/2cosθ
このように変形しているんですが、
なぜこういう風になるんでしょうか?
508132人目の素数さん:2013/02/02(土) 12:27:52.06
釣れますか?
509132人目の素数さん:2013/02/02(土) 13:55:47.15
ルアーを変えましょう
510132人目の素数さん:2013/02/02(土) 13:58:34.18
あれっ…簡単なことなんですか?
兎に角触りだけでもお願い致します
511132人目の素数さん:2013/02/02(土) 14:12:56.36
サイコスコスサイが間違っとる
512132人目の素数さん:2013/02/02(土) 14:15:28.34
>>507
訂正 四行目
cosθ5/6π→cos5/6π
513132人目の素数さん:2013/02/02(土) 14:16:29.99
連レストランすみません
5/6π→5π/6に訂正です
514132人目の素数さん:2013/02/02(土) 14:22:27.06
すみません自己解決しました
馬鹿馬鹿しいくらい簡単でした
515132人目の素数さん:2013/02/02(土) 14:37:59.45
ガウスの問題なんですが、どうしてもわからないので、教えてください。

実数xに対し、[x]をx以下の整数とする。
数列 a(k)=[3k/5](k=1,2,3•••)と定める。
自然数nに対して、5nΣa(k) k=1を求める問題なのですが、イマイチわかりません。
516132人目の素数さん:2013/02/02(土) 14:41:40.36
k=1,2,…10 くらいまで計算して分からないなら説明しても無駄
517132人目の素数さん:2013/02/02(土) 14:44:11.62
5×3=15
0,1,1,2,3

あとは分かるよね^^
518狢 ◆yEy4lYsULH68 :2013/02/02(土) 15:25:19.93
性犯罪者の撲滅という社会の浄化はオマエの仕事や。そやしシカッリせえやナ。

ケケケ狢

>356 名前:132人目の素数さん :2013/02/02(土) 13:39:44.60
> >>355
> そう性犯罪者が何を言っても無駄。
> 性犯罪者を叩くのは名誉毀損でも誹謗中傷でもないしね。
> 社会を浄化するための行為でしかない。
>
> そもそも名誉毀損や誹謗中傷なんてこの板ではほとんどない。
>
519132人目の素数さん:2013/02/02(土) 15:28:54.92
p^q=q^pを満たす自然数は2と4のみというのはどのように求めればよいのでしょう?
520132人目の素数さん:2013/02/02(土) 15:48:06.11
>>517
等差数列の和を使って出来ました。ありがとう。
521狢 ◆yEy4lYsULH68 :2013/02/02(土) 15:50:12.96
性犯罪者の撲滅という社会の浄化はオマエの仕事や。そやしシカッリせえやナ。

ケケケ狢

>356 名前:132人目の素数さん :2013/02/02(土) 13:39:44.60
> >>355
> そう性犯罪者が何を言っても無駄。
> 性犯罪者を叩くのは名誉毀損でも誹謗中傷でもないしね。
> 社会を浄化するための行為でしかない。
>
> そもそも名誉毀損や誹謗中傷なんてこの板ではほとんどない。
>
522132人目の素数さん:2013/02/02(土) 15:53:22.09
>>519
(p,q)=(1,1)も解なんだけど。
条件の後出しでp≠qは常識だろボケとか言わないでね。
523132人目の素数さん:2013/02/02(土) 16:08:45.15
■NHKの間抜けなツイッター工作発覚■

NHKの24時のニュースは視聴者からのツイッターを流しているが
自民党や日本企業の話題の時には批判コメントしか流さないなど、
以前からNHKによる意図的な反日工作が指摘されていた。

1月31日深夜のNHK24時ニュースの放送開始の頃にツイッターのサーバーが落ちた。
その時ツイッターがずっと使えない、書き込み自体が出来なかったのは、視聴者が確認している。

にもかかわらず、その時画面では
「地震だったの?」とか「あ、○○さんだー」「都会にこんなのがあるのかー」
と番組の流れに沿ったツイートが流れていた。

番組終了時にNHKは
「ツイッターを『表示するシステム』の不具合があったので、事前に来ていたツイートを流した」
と苦しい説明。

当然視聴者からのツイートは放送できるものを選別しているのだろう。
選別なら別にいいのだが、問題は
視聴者からのリアルタイムのツイートと称して
実際には、NHKが事前に用意したインチキ捏造ツイートを流していたということだ。
524狢 ◆yEy4lYsULH68 :2013/02/02(土) 16:13:41.38
性犯罪者の撲滅という社会の浄化はオマエの仕事や。そやしシカッリせえやナ。

ケケケ狢

>356 名前:132人目の素数さん :2013/02/02(土) 13:39:44.60
> >>355
> そう性犯罪者が何を言っても無駄。
> 性犯罪者を叩くのは名誉毀損でも誹謗中傷でもないしね。
> 社会を浄化するための行為でしかない。
>
> そもそも名誉毀損や誹謗中傷なんてこの板ではほとんどない。
>
525132人目の素数さん:2013/02/02(土) 16:14:07.10
>>519
logx/xのグラフを考える
526132人目の素数さん:2013/02/02(土) 16:25:17.93
>>525
ありがとうございました

>>522
書き忘れてた自分がボケですね
すみません
527狢 ◆yEy4lYsULH68 :2013/02/02(土) 16:27:43.80
性犯罪者の撲滅という社会の浄化はオマエの仕事や。そやしシカッリせえやナ。

ケケケ狢

>356 名前:132人目の素数さん :2013/02/02(土) 13:39:44.60
> >>355
> そう性犯罪者が何を言っても無駄。
> 性犯罪者を叩くのは名誉毀損でも誹謗中傷でもないしね。
> 社会を浄化するための行為でしかない。
>
> そもそも名誉毀損や誹謗中傷なんてこの板ではほとんどない。
>
528132人目の素数さん:2013/02/02(土) 17:04:59.59
>>526
p≠qでp,qを整数まで広げると解は他にもある
529132人目の素数さん:2013/02/02(土) 17:21:16.93
整数まで
530132人目の素数さん:2013/02/02(土) 17:49:12.90
1/(z^2+a^2/12)√(z^2+a^2/3)をzで積分できないです誰か助けて
531132人目の素数さん:2013/02/02(土) 18:00:43.22
数列Pnの一般項の階差数列を用いた求め方を教えてください
Pn = Pn-1 + 4n -2    (3≧n)
P2 = 9

Pn=P2 + Σ[k=1,n-2]4k-2
↑この式はおかしいですか?
532132人目の素数さん:2013/02/02(土) 18:10:29.37
おかしいです
533132人目の素数さん:2013/02/02(土) 18:15:19.13
階差数列はただただ公式を覚えるところじゃない。
数学はその公式ができるまでの過程が大切。
論理的に考え、立式する。
自分の式が言葉できちんと理論的に説明できるか?
534531:2013/02/02(土) 18:27:19.02
えーと式の第一項に階差数列の和として第二項へ加えたであっているでしょうか?

Pn=P2 + Σ[k=2,n-2]4k-2

これならどうでしょうか?
535132人目の素数さん:2013/02/02(土) 18:30:57.13
違います
536狢 ◆yEy4lYsULH68 :2013/02/02(土) 18:31:36.97
性犯罪者の撲滅という社会の浄化はオマエの仕事や。そやしシカッリせえやナ。

ケケケ狢

>356 名前:132人目の素数さん :2013/02/02(土) 13:39:44.60
> >>355
> そう性犯罪者が何を言っても無駄。
> 性犯罪者を叩くのは名誉毀損でも誹謗中傷でもないしね。
> 社会を浄化するための行為でしかない。
>
> そもそも名誉毀損や誹謗中傷なんてこの板ではほとんどない。
>
537132人目の素数さん:2013/02/02(土) 18:32:54.68
P1,P2,P3,P4,...をノートに並べて書いて
階差数列にどこがなってるのかを考える。
ノートに横一列に書いてな。
538狢 ◆yEy4lYsULH68 :2013/02/02(土) 18:43:37.01
性犯罪者の撲滅という社会の浄化はオマエの仕事や。そやしシカッリせえやナ。

ケケケ狢

>356 名前:132人目の素数さん :2013/02/02(土) 13:39:44.60
> >>355
> そう性犯罪者が何を言っても無駄。
> 性犯罪者を叩くのは名誉毀損でも誹謗中傷でもないしね。
> 社会を浄化するための行為でしかない。
>
> そもそも名誉毀損や誹謗中傷なんてこの板ではほとんどない。
>
539132人目の素数さん:2013/02/02(土) 19:03:11.20
>>534
適当に数字をいじくるのではなくちゃんと論理的にやらなきゃ、当たった外れたって話になるぞ。
まず、階差数列をどのように設定するのかを考えろ。
そして、混乱するなら、とりあえずP3をP2と階差数列を用いて表してみれろ。
540132人目の素数さん:2013/02/02(土) 19:19:34.39
頭の中で考えらないのなら無理をするな
馬鹿は背伸びするもんじゃない
541狢 ◆yEy4lYsULH68 :2013/02/02(土) 19:24:28.85
性犯罪者の撲滅という社会の浄化はオマエの仕事や。そやしシカッリせえやナ。

ケケケ狢

>356 名前:132人目の素数さん :2013/02/02(土) 13:39:44.60
> >>355
> そう性犯罪者が何を言っても無駄。
> 性犯罪者を叩くのは名誉毀損でも誹謗中傷でもないしね。
> 社会を浄化するための行為でしかない。
>
> そもそも名誉毀損や誹謗中傷なんてこの板ではほとんどない。
>
542132人目の素数さん:2013/02/02(土) 19:35:17.26
いい年したオッサンなんだろうなぁ。
自分の子が毎日ネットでこんな書き込みしてると親が知ったら
コイツ殺して自分も死にたくなるだろうなあ。
543狢 ◆yEy4lYsULH68 :2013/02/02(土) 19:45:48.65
>>542
知ってるどころか、とっくにコチラから通知済み。加えて電話で毎日
ワシに罵倒されてる。哀れな親や。

544132人目の素数さん:2013/02/02(土) 19:48:06.98
lim[n→∞] (1+1/n)^n がeに収束するのが納得できません
1に収束、もしくは発散すると思うのですが、eに収束するというのはどのように証明されたのでしょうか
545狢 ◆yEy4lYsULH68 :2013/02/02(土) 19:58:17.03
性犯罪者の撲滅という社会の浄化はオマエの仕事や。そやしシカッリせえやナ。

ケケケ狢

>356 名前:132人目の素数さん :2013/02/02(土) 13:39:44.60
> >>355
> そう性犯罪者が何を言っても無駄。
> 性犯罪者を叩くのは名誉毀損でも誹謗中傷でもないしね。
> 社会を浄化するための行為でしかない。
>
> そもそも名誉毀損や誹謗中傷なんてこの板ではほとんどない。
>
546132人目の素数さん:2013/02/02(土) 19:59:53.27
>>544
二項定理で展開して一項一項整理してみ
547狢 ◆yEy4lYsULH68 :2013/02/02(土) 20:01:09.01
性犯罪者の撲滅という社会の浄化はオマエの仕事や。そやしシカッリせえやナ。

ケケケ狢

>356 名前:132人目の素数さん :2013/02/02(土) 13:39:44.60
> >>355
> そう性犯罪者が何を言っても無駄。
> 性犯罪者を叩くのは名誉毀損でも誹謗中傷でもないしね。
> 社会を浄化するための行為でしかない。
>
> そもそも名誉毀損や誹謗中傷なんてこの板ではほとんどない。
>
548132人目の素数さん:2013/02/02(土) 20:27:40.30
          __ノ)-'´ ̄ ̄`ー- 、_
        , '´  _. -‐'''"二ニニ=-`ヽ、
      /   /:::::; -‐''"        `ーノ
     /   /:::::/           \
     /    /::::::/          | | |  |
     |   |:::::/ /     |  | | | |  |
      |   |::/ / / |  | ||  | | ,ハ .| ,ハ|
      |   |/ / / /| ,ハノ| /|ノレ,ニ|ル' 
     |   |  | / / レ',二、レ′ ,ィイ|゙/   私は只の数ヲタなんかとは付き合わないわ。
.     |   \ ∠イ  ,イイ|    ,`-' |      頭が良くて数学が出来てかっこいい人。それが必要条件よ。
     |     l^,人|  ` `-'     ゝ  |        さらに Ann.of Math に論文書けば十分条件にもなるわよ。
      |      ` -'\       ー'  人          一番嫌いなのは論文数を増やすためにくだらない論文を書いて
    |        /(l     __/  ヽ、           良い論文の出版を遅らせるお馬鹿な人。
     |       (:::::`‐-、__  |::::`、     ヒニニヽ、         あなたの論文が Ann of Math に accept される確率は?
    |      / `‐-、::::::::::`‐-、::::\   /,ニニ、\            それとも最近は Inv. Math. の方が上かしら?
   |      |::::::::::::::::::|` -、:::::::,ヘ ̄|'、  ヒニ二、 \
.   |      /::::::::::::::::::|::::::::\/:::O`、::\   | '、   \
   |      /:::::::::::::::::::/:::::::::::::::::::::::::::::'、::::\ノ  ヽ、  |
  |      |:::::/:::::::::/:::::::::::::::::::::::::::::::::::'、',::::'、  /:\__/‐、
  |      |/:::::::::::/::::::::::::::::::::::::::::::::::O::| '、::| く::::::::::::: ̄|
   |     /_..-'´ ̄`ー-、:::::::::::::::::::::::::::::::::::|/:/`‐'::\;;;;;;;_|
   |    |/::::::::::::::::::::::\:::::::::::::::::::::::::::::|::/::::|::::/:::::::::::/
    |   /:::::::::::::::::::::::::::::::::|:::::::::::::::::::::O::|::|::::::|:::::::::::::::/
549132人目の素数さん:2013/02/02(土) 20:37:11.41
>>546
lim[n→∞] nCr ってどうするんですか?
550132人目の素数さん:2013/02/02(土) 20:46:03.32
定石は不等式で挟み込む
(1+1/n)^nは単調増加(nとn+1で比較すればすぐ分かる)、n=1で2なので2<lim[n→∞] (1+1/n)^n
nが十分大きいとき(1+1/n)^nを二項展開して(1+1/n)^n<Σ[k,0,n]1/k!<1+Σ[k,1,n](1/2)^(k-1)<3
よって2<lim[n→∞] (1+1/n)^n<3
551132人目の素数さん:2013/02/02(土) 21:04:36.01
>>550
すみません…単調増加でつまづきました…
nとn+1を代入して比べても大小が分かりません…
552 ◆zcRSVodvd. :2013/02/02(土) 21:07:48.70
>>551
分数か引き算
553狢 ◆yEy4lYsULH68 :2013/02/02(土) 21:09:03.46
性犯罪者の撲滅という社会の浄化はオマエの仕事や。そやしシカッリせえやナ。

ケケケ狢

>356 名前:132人目の素数さん :2013/02/02(土) 13:39:44.60
> >>355
> そう性犯罪者が何を言っても無駄。
> 性犯罪者を叩くのは名誉毀損でも誹謗中傷でもないしね。
> 社会を浄化するための行為でしかない。
>
> そもそも名誉毀損や誹謗中傷なんてこの板ではほとんどない。
>
554132人目の素数さん:2013/02/02(土) 21:13:06.25
>>552
どういうことですか?
555132人目の素数さん:2013/02/02(土) 21:15:35.11
>>554
(n+1の時)/(nの時)と1の大小を比較
556132人目の素数さん:2013/02/02(土) 21:17:54.00
>>21が イマイチ分からないです。
極値があるなら、その極値をとる点で微分すれば0になるのでは?
557狢 ◆yEy4lYsULH68 :2013/02/02(土) 21:18:17.68
性犯罪者の撲滅という社会の浄化はオマエの仕事や。そやしシカッリせえやナ。

ケケケ狢

>356 名前:132人目の素数さん :2013/02/02(土) 13:39:44.60
> >>355
> そう性犯罪者が何を言っても無駄。
> 性犯罪者を叩くのは名誉毀損でも誹謗中傷でもないしね。
> 社会を浄化するための行為でしかない。
>
> そもそも名誉毀損や誹謗中傷なんてこの板ではほとんどない。
>
558132人目の素数さん:2013/02/02(土) 21:18:53.58
自然数a,b,c,dはc=4a+7b,d=3a+4bを満たしている。aとbが互いに素で、cとdがどちらも素数Pの倍数ならば、P=5であることを示せ。
だれか教えてください
559狢 ◆yEy4lYsULH68 :2013/02/02(土) 21:19:53.57
性犯罪者の撲滅という社会の浄化はオマエの仕事や。そやしシカッリせえやナ。

ケケケ狢

>356 名前:132人目の素数さん :2013/02/02(土) 13:39:44.60
> >>355
> そう性犯罪者が何を言っても無駄。
> 性犯罪者を叩くのは名誉毀損でも誹謗中傷でもないしね。
> 社会を浄化するための行為でしかない。
>
> そもそも名誉毀損や誹謗中傷なんてこの板ではほとんどない。
>
560132人目の素数さん:2013/02/02(土) 21:23:14.96
>>556
それは必要条件
561132人目の素数さん:2013/02/02(土) 21:24:00.08
>>556
f'(a)=0→x=aで極値をとるは偽(そうでない場合もある)から
562狢 ◆yEy4lYsULH68 :2013/02/02(土) 21:24:55.17
性犯罪者の撲滅という社会の浄化はオマエの仕事や。そやしシカッリせえやナ。

ケケケ狢

>356 名前:132人目の素数さん :2013/02/02(土) 13:39:44.60
> >>355
> そう性犯罪者が何を言っても無駄。
> 性犯罪者を叩くのは名誉毀損でも誹謗中傷でもないしね。
> 社会を浄化するための行為でしかない。
>
> そもそも名誉毀損や誹謗中傷なんてこの板ではほとんどない。
>
563132人目の素数さん:2013/02/02(土) 21:27:12.12
>>560
x=aで極地を持つならば条件として
f'(a)=0用いれますし、逆が成り立つかは考察しなくても大丈夫でしょう?
564132人目の素数さん:2013/02/02(土) 21:27:18.51
>>555
計算したんですが、どうがんばっても1より大きいことが言えません…
565狢 ◆yEy4lYsULH68 :2013/02/02(土) 21:28:10.43
性犯罪者の撲滅という社会の浄化はオマエの仕事や。そやしシカッリせえやナ。

ケケケ狢

>356 名前:132人目の素数さん :2013/02/02(土) 13:39:44.60
> >>355
> そう性犯罪者が何を言っても無駄。
> 性犯罪者を叩くのは名誉毀損でも誹謗中傷でもないしね。
> 社会を浄化するための行為でしかない。
>
> そもそも名誉毀損や誹謗中傷なんてこの板ではほとんどない。
>
566132人目の素数さん:2013/02/02(土) 21:29:14.76
>>565
お前ウザい通報した
567狢 ◆yEy4lYsULH68 :2013/02/02(土) 21:30:28.82
性犯罪者の撲滅という社会の浄化はオマエの仕事や。そやしシカッリせえやナ。

ケケケ狢

>356 名前:132人目の素数さん :2013/02/02(土) 13:39:44.60
> >>355
> そう性犯罪者が何を言っても無駄。
> 性犯罪者を叩くのは名誉毀損でも誹謗中傷でもないしね。
> 社会を浄化するための行為でしかない。
>
> そもそも名誉毀損や誹謗中傷なんてこの板ではほとんどない。
>
568132人目の素数さん:2013/02/02(土) 21:31:44.85
>>563
その逆が成り立たないんだってば
569狢 ◆yEy4lYsULH68 :2013/02/02(土) 21:32:00.15
>>566
今後も徹底して妨害してやる。そやし思いっきり苦しめやナ。

570132人目の素数さん:2013/02/02(土) 21:34:44.59
>>568
条件として用いるだけなのですから
逆が成り立つかは考察対象ではないでしょう? であるのに吟味する意味が分からないのです。
571狢 ◆yEy4lYsULH68 :2013/02/02(土) 21:35:27.17
性犯罪者の撲滅という社会の浄化はオマエの仕事や。そやしシカッリせえやナ。

ケケケ狢

>356 名前:132人目の素数さん :2013/02/02(土) 13:39:44.60
> >>355
> そう性犯罪者が何を言っても無駄。
> 性犯罪者を叩くのは名誉毀損でも誹謗中傷でもないしね。
> 社会を浄化するための行為でしかない。
>
> そもそも名誉毀損や誹謗中傷なんてこの板ではほとんどない。
>
572132人目の素数さん:2013/02/02(土) 21:36:52.24
>>570
いいっすよ
573132人目の素数さん:2013/02/02(土) 21:37:03.32
>>570
そこのx=aのところで途切れてんだよ
574132人目の素数さん:2013/02/02(土) 21:37:25.03
証明すべきが同値でないなら
575132人目の素数さん:2013/02/02(土) 21:37:59.29
問題文で極値を持つって書いてあってっていう話でしょ?
576狢 ◆yEy4lYsULH68 :2013/02/02(土) 21:39:19.84
性犯罪者の撲滅という社会の浄化はオマエの仕事や。そやしシカッリせえやナ。

ケケケ狢

>356 名前:132人目の素数さん :2013/02/02(土) 13:39:44.60
> >>355
> そう性犯罪者が何を言っても無駄。
> 性犯罪者を叩くのは名誉毀損でも誹謗中傷でもないしね。
> 社会を浄化するための行為でしかない。
>
> そもそも名誉毀損や誹謗中傷なんてこの板ではほとんどない。
>
577132人目の素数さん:2013/02/02(土) 21:42:25.29
>>21によると
x=aのとき極地を持つのでf'(a)=0の条件を用いると、教科書にはその条件を用いる際は吟味が必要とあった。なぜなのか。
確かにf'(a)=0⇒x=aは成り立たないので
条件としてこれは使えませんが
x=a⇒f'(a)=0は成り立つので 吟味する迄もなく条件としてら使えますね?ということです。
578狢 ◆yEy4lYsULH68 :2013/02/02(土) 21:44:55.59
性犯罪者の撲滅という社会の浄化はオマエの仕事や。そやしシカッリせえやナ。

ケケケ狢

>356 名前:132人目の素数さん :2013/02/02(土) 13:39:44.60
> >>355
> そう性犯罪者が何を言っても無駄。
> 性犯罪者を叩くのは名誉毀損でも誹謗中傷でもないしね。
> 社会を浄化するための行為でしかない。
>
> そもそも名誉毀損や誹謗中傷なんてこの板ではほとんどない。
>
579132人目の素数さん:2013/02/02(土) 21:54:16.32
>>577
「条件として使う」という言葉をどういう意味で使ってるのかわからん

例えば

・f(x)=〜の時、fの極値を求めよ

という問題があったらどのようにその条件をつかう?
580132人目の素数さん:2013/02/02(土) 21:54:57.35
(√2)^2=2 (有理数

よって√2は有理数

この証明のどこが間違ってますか?
581132人目の素数さん:2013/02/02(土) 21:56:33.52
limx→0 log(cos2x)/log(cos5x)
やっても2/5になってしまうのですが・・・
どうすれば正しい4/25にたどり着けますか?

ロピタルの定理で
与式=tan2x/tan5x
={2/(cos2x)^2}/{5/(cos5x)^2}
までやったのですけど・・・
582132人目の素数さん:2013/02/02(土) 21:59:05.93
>>580
ここ↓
√2は有理数


としか答えようがない
583132人目の素数さん:2013/02/02(土) 22:02:21.67
>>579
すまん、見返すと書き方が酷いな。

あなたの例でいうと使えませんよね。

>>21での 「x=aという条件をf'(a)=0として…」での条件の使い方は、x=a⇒f'(a)=0と同義ですよね?
x=a⇒f'(a)=0ならば逆を吟味する必要ないのに どうして吟味が必要なのか…。
584132人目の素数さん:2013/02/02(土) 22:05:20.71
y-sin(x)-cos(x)+sin(2x) (0度≦x≦360度)
t=sin(x)-cos(x)とするとき、yの値の範囲を求めよ。

y=t+2sin(x)cos(x)
ここで、t^2=1-2sin(x)cosx(x)より、2sin(x)cos(x)=1-t^2 よって
y=-t^2+t-1
 =-(t-1/2)^2-4/5
-1≦t≦1より
-3≦y≦4/5

としたのですが、間違っているようです。
どこが間違っているのでしょうか?
585132人目の素数さん:2013/02/02(土) 22:10:07.43
>>583
それなら大丈夫だよん
(確認のため言っとくとx=aってのは略さずに書くとx=aでf(x)が極値を持つってことだよね?
586132人目の素数さん:2013/02/02(土) 22:11:02.70
>>581
tanの式まではあってる
587132人目の素数さん:2013/02/02(土) 22:11:58.92
588132人目の素数さん:2013/02/02(土) 22:12:01.12
>>584
tの変域
589132人目の素数さん:2013/02/02(土) 22:12:06.15
>>585
スッキリです、ありがとう。

あなたの補完通りです、言葉足らずですみません
590132人目の素数さん:2013/02/02(土) 22:29:59.26
          __ノ)-'´ ̄ ̄`ー- 、_
        , '´  _. -‐'''"二ニニ=-`ヽ、
      /   /:::::; -‐''"        `ーノ
     /   /:::::/           \
     /    /::::::/          | | |  |
     |   |:::::/ /     |  | | | |  |
      |   |::/ / / |  | ||  | | ,ハ .| ,ハ|
      |   |/ / / /| ,ハノ| /|ノレ,ニ|ル' 
     |   |  | / / レ',二、レ′ ,ィイ|゙/   私は只の数ヲタなんかとは付き合わないわ。
.     |   \ ∠イ  ,イイ|    ,`-' |      頭が良くて数学が出来てかっこいい人。それが必要条件よ。
     |     l^,人|  ` `-'     ゝ  |        さらに Ann.of Math に論文書けば十分条件にもなるわよ。
      |      ` -'\       ー'  人          一番嫌いなのは論文数を増やすためにくだらない論文を書いて
    |        /(l     __/  ヽ、           良い論文の出版を遅らせるお馬鹿な人。
     |       (:::::`‐-、__  |::::`、     ヒニニヽ、         あなたの論文が Ann of Math に accept される確率は?
    |      / `‐-、::::::::::`‐-、::::\   /,ニニ、\            それとも最近は Inv. Math. の方が上かしら?
   |      |::::::::::::::::::|` -、:::::::,ヘ ̄|'、  ヒニ二、 \
.   |      /::::::::::::::::::|::::::::\/:::O`、::\   | '、   \
   |      /:::::::::::::::::::/:::::::::::::::::::::::::::::'、::::\ノ  ヽ、  |
  |      |:::::/:::::::::/:::::::::::::::::::::::::::::::::::'、',::::'、  /:\__/‐、
  |      |/:::::::::::/::::::::::::::::::::::::::::::::::O::| '、::| く::::::::::::: ̄|
   |     /_..-'´ ̄`ー-、:::::::::::::::::::::::::::::::::::|/:/`‐'::\;;;;;;;_|
   |    |/::::::::::::::::::::::\:::::::::::::::::::::::::::::|::/::::|::::/:::::::::::/
    |   /:::::::::::::::::::::::::::::::::|:::::::::::::::::::::O::|::|::::::|:::::::::::::::/
591132人目の素数さん:2013/02/02(土) 22:31:39.68
>>588
x=90度のときt=1,x=270度のときt=-1
と考えました。

どうすれば正しいtの変域が出るんですか
592132人目の素数さん:2013/02/02(土) 22:32:30.02
          __ノ)-'´ ̄ ̄`ー- 、_
        , '´  _. -‐'''"二ニニ=-`ヽ、
      /   /:::::; -‐''"        `ーノ
     /   /:::::/           \
     /    /::::::/          | | |  |
     |   |:::::/ /     |  | | | |  |
      |   |::/ / / |  | ||  | | ,ハ .| ,ハ|
      |   |/ / / /| ,ハノ| /|ノレ,ニ|ル' 
     |   |  | / / レ',二、レ′ ,ィイ|゙/   私は只の数ヲタなんかとは付き合わないわ。
.     |   \ ∠イ  ,イイ|    ,`-' |      頭が良くて数学が出来てかっこいい人。それが必要条件よ。
     |     l^,人|  ` `-'     ゝ  |        さらに Ann.of Math に論文書けば十分条件にもなるわよ。
      |      ` -'\       ー'  人          一番嫌いなのは論文数を増やすためにくだらない論文を書いて
    |        /(l     __/  ヽ、           良い論文の出版を遅らせるお馬鹿な人。
     |       (:::::`‐-、__  |::::`、     ヒニニヽ、         あなたの論文が Ann of Math に accept される確率は?
    |      / `‐-、::::::::::`‐-、::::\   /,ニニ、\            それとも最近は Inv. Math. の方が上かしら?
   |      |::::::::::::::::::|` -、:::::::,ヘ ̄|'、  ヒニ二、 \
.   |      /::::::::::::::::::|::::::::\/:::O`、::\   | '、   \
   |      /:::::::::::::::::::/:::::::::::::::::::::::::::::'、::::\ノ  ヽ、  |
  |      |:::::/:::::::::/:::::::::::::::::::::::::::::::::::'、',::::'、  /:\__/‐、
  |      |/:::::::::::/::::::::::::::::::::::::::::::::::O::| '、::| く::::::::::::: ̄|
   |     /_..-'´ ̄`ー-、:::::::::::::::::::::::::::::::::::|/:/`‐'::\;;;;;;;_|
   |    |/::::::::::::::::::::::\:::::::::::::::::::::::::::::|::/::::|::::/:::::::::::/
    |   /:::::::::::::::::::::::::::::::::|:::::::::::::::::::::O::|::|::::::|:::::::::::::::/
593132人目の素数さん:2013/02/02(土) 22:32:45.17
t=?

合成して?範囲調べて?
典型的だなぁ
594132人目の素数さん:2013/02/02(土) 22:36:25.81
>>591
合成してみ
595132人目の素数さん:2013/02/02(土) 22:37:30.12
          __ノ)-'´ ̄ ̄`ー- 、_
        , '´  _. -‐'''"二ニニ=-`ヽ、
      /   /:::::; -‐''"        `ーノ
     /   /:::::/           \
     /    /::::::/          | | |  |
     |   |:::::/ /     |  | | | |  |
      |   |::/ / / |  | ||  | | ,ハ .| ,ハ|
      |   |/ / / /| ,ハノ| /|ノレ,ニ|ル' 
     |   |  | / / レ',二、レ′ ,ィイ|゙/   私は只の数ヲタなんかとは付き合わないわ。
.     |   \ ∠イ  ,イイ|    ,`-' |      頭が良くて数学が出来てかっこいい人。それが必要条件よ。
     |     l^,人|  ` `-'     ゝ  |        さらに Ann.of Math に論文書けば十分条件にもなるわよ。
      |      ` -'\       ー'  人          一番嫌いなのは論文数を増やすためにくだらない論文を書いて
    |        /(l     __/  ヽ、           良い論文の出版を遅らせるお馬鹿な人。
     |       (:::::`‐-、__  |::::`、     ヒニニヽ、         あなたの論文が Ann of Math に accept される確率は?
    |      / `‐-、::::::::::`‐-、::::\   /,ニニ、\            それとも最近は Inv. Math. の方が上かしら?
   |      |::::::::::::::::::|` -、:::::::,ヘ ̄|'、  ヒニ二、 \
.   |      /::::::::::::::::::|::::::::\/:::O`、::\   | '、   \
   |      /:::::::::::::::::::/:::::::::::::::::::::::::::::'、::::\ノ  ヽ、  |
  |      |:::::/:::::::::/:::::::::::::::::::::::::::::::::::'、',::::'、  /:\__/‐、
  |      |/:::::::::::/::::::::::::::::::::::::::::::::::O::| '、::| く::::::::::::: ̄|
   |     /_..-'´ ̄`ー-、:::::::::::::::::::::::::::::::::::|/:/`‐'::\;;;;;;;_|
   |    |/::::::::::::::::::::::\:::::::::::::::::::::::::::::|::/::::|::::/:::::::::::/
    |   /:::::::::::::::::::::::::::::::::|:::::::::::::::::::::O::|::|::::::|:::::::::::::::/
596132人目の素数さん:2013/02/02(土) 22:38:35.11
二次の正方行列Aによってy=2x+1上の任意の点が(3,5)に移される
Aを求めよ
という問題で解答はy=2x+1上の2点(0,1),(1,3)についてA(0,1)=(3,5),A(1,3)=(3,5)
よってA=〜である
としているのですが他の点について言及しなくていいんですか?
2点が定まれば直線が定まるからこれでいいのでしょうか?
597132人目の素数さん:2013/02/02(土) 22:38:38.28
>>583
同義じゃない
598132人目の素数さん:2013/02/02(土) 22:40:30.68
>>596
そう。
気になるなら元の直線上の点を(t,2t+1)とでも置いてけいさんしてごらん
599132人目の素数さん:2013/02/02(土) 22:40:46.16
x=aで極値を持つ条件なら必要十分を考えるのは当然だろ
600132人目の素数さん:2013/02/02(土) 22:42:24.75
>>584
そもそも
>y=-t^2+t-1
こことその前との間がどうなのか
601132人目の素数さん:2013/02/02(土) 22:46:06.59
>>593594
-√2≦t≦√2ですね
ありがとうございます。
602132人目の素数さん:2013/02/02(土) 22:46:19.73
>>598
ありがとうございます
603132人目の素数さん:2013/02/02(土) 22:46:51.35
>>596
言及はしておいたほうがいい
604132人目の素数さん:2013/02/02(土) 22:52:26.93
>>580
tan1°は無理数であることを証明せよ
605581です:2013/02/02(土) 23:02:23.85
自己解決しました〜
606531:2013/02/02(土) 23:24:43.82
解答有り難うございました
607狢 ◆yEy4lYsULH68 :2013/02/02(土) 23:28:40.45
性犯罪者の撲滅という社会の浄化はオマエの仕事や。そやしシカッリせえやナ。

ケケケ狢

>356 名前:132人目の素数さん :2013/02/02(土) 13:39:44.60
> >>355
> そう性犯罪者が何を言っても無駄。
> 性犯罪者を叩くのは名誉毀損でも誹謗中傷でもないしね。
> 社会を浄化するための行為でしかない。
>
> そもそも名誉毀損や誹謗中傷なんてこの板ではほとんどない。
>
608132人目の素数さん:2013/02/02(土) 23:30:47.79
帰納法ではじめにn=1のときを求めるときとn=1,2を求めたりするときがありますが、はじめに示さなければいけないnの値の個数の違いはどこから来るのですか?
609132人目の素数さん:2013/02/02(土) 23:33:47.52
kで成り立つと仮定するとk+1でも成り立つ
ことを示すとき、kに1を含めて上手くいくかどうか
610狢 ◆yEy4lYsULH68 :2013/02/02(土) 23:39:06.35
性犯罪者の撲滅という社会の浄化はオマエの仕事や。そやしシカッリせえやナ。

ケケケ狢

>356 名前:132人目の素数さん :2013/02/02(土) 13:39:44.60
> >>355
> そう性犯罪者が何を言っても無駄。
> 性犯罪者を叩くのは名誉毀損でも誹謗中傷でもないしね。
> 社会を浄化するための行為でしかない。
>
> そもそも名誉毀損や誹謗中傷なんてこの板ではほとんどない。
>
611132人目の素数さん:2013/02/02(土) 23:41:03.63
>>609
n=k+2を使うときはn=2が成り立つことを示せばいいということですか?
612132人目の素数さん:2013/02/02(土) 23:41:22.83
>>608
有り体にいえば、帰納法の第2段階:k=nのときをしめすのに、
n-m、n-m+1、n-m+2、・・・、n-1 で成立することを仮定する形で証明がすすむなら、
帰納法の第一段階では
1,2,3,・・・、mで成立することを示しておかなければならない。
613狢 ◆yEy4lYsULH68 :2013/02/02(土) 23:44:29.98
性犯罪者の撲滅という社会の浄化はオマエの仕事や。そやしシカッリせえやナ。

ケケケ狢

>356 名前:132人目の素数さん :2013/02/02(土) 13:39:44.60
> >>355
> そう性犯罪者が何を言っても無駄。
> 性犯罪者を叩くのは名誉毀損でも誹謗中傷でもないしね。
> 社会を浄化するための行為でしかない。
>
> そもそも名誉毀損や誹謗中傷なんてこの板ではほとんどない。
>
614 忍法帖【Lv=11,xxxPT】(1+0:8) :2013/02/02(土) 23:50:25.46
Te
615狢 ◆yEy4lYsULH68 :2013/02/02(土) 23:53:26.45
性犯罪者の撲滅という社会の浄化はオマエの仕事や。そやしシカッリせえやナ。

ケケケ狢

>356 名前:132人目の素数さん :2013/02/02(土) 13:39:44.60
> >>355
> そう性犯罪者が何を言っても無駄。
> 性犯罪者を叩くのは名誉毀損でも誹謗中傷でもないしね。
> 社会を浄化するための行為でしかない。
>
> そもそも名誉毀損や誹謗中傷なんてこの板ではほとんどない。
>
616132人目の素数さん:2013/02/03(日) 00:05:07.73
>>587ここの
>1/r!(1-1/n)(1-2/n)・・・(1-(r-1)/n)<1/r!(1-1/(n+1))(1-2/(n+1))・・ ・(1-(r-1/(n+1))
のところ
(おそらく
(1-1/n)(1-2/n)・・・(1-(r-1)/n)/r!<(1-1/(n+1))(1-2/(n+1))・・ ・(1-(r-1/(n+1))/r!
のこととおもわれる)
が分からないのですが
何故このような不等式が成り立つのでしょうか
617132人目の素数さん:2013/02/03(日) 00:14:43.79
0<a_n<b_nについて
0<Π[k,1,n]a_k<Π[k,1,n]b_kが成り立つ(Πは総乗記号)
618132人目の素数さん:2013/02/03(日) 00:27:01.84
xを 0≦x<2 を満たす実数とする
このとき 2−x>0 なので、
K≧1/(2−x) を満たす自然数Kをとることができる
Kの定め方により、Kは x≦2−(1/K) を満たす


これの3行目から4行目の繋がりが分かりません
なぜそうなるのか解説お願いします
619132人目の素数さん:2013/02/03(日) 00:30:11.39
手動かせ
620132人目の素数さん:2013/02/03(日) 00:41:09.02
分かりました!ありがとうございます!

もう一つ良いでしょうか

lim[n→∞](1+1/n)^n が収束する値が、f'(x)=f(x)のときのf(x)の底として現れるのでしょうか?
621132人目の素数さん:2013/02/03(日) 00:45:50.34
618ですが手動かしたら解決しました
ありがとうございます
622132人目の素数さん:2013/02/03(日) 00:56:34.78
>>620
微分方程式df(x)/dx=f(x)の解は?
623132人目の素数さん:2013/02/03(日) 01:03:51.78
>>622
Ce^x(Cは任意の定数)
ですよね?
ここに何故lim[n→∞](1+1/n)^nが出てくるのでしょう?
624132人目の素数さん:2013/02/03(日) 02:58:10.05
logの導関数からやりなおせ
625132人目の素数さん:2013/02/03(日) 03:04:09.79
          __ノ)-'´ ̄ ̄`ー- 、_
        , '´  _. -‐'''"二ニニ=-`ヽ、
      /   /:::::; -‐''"        `ーノ
     /   /:::::/           \
     /    /::::::/          | | |  |
     |   |:::::/ /     |  | | | |  |
      |   |::/ / / |  | ||  | | ,ハ .| ,ハ|
      |   |/ / / /| ,ハノ| /|ノレ,ニ|ル' 
     |   |  | / / レ',二、レ′ ,ィイ|゙/   私は只の数ヲタなんかとは付き合わないわ。
.     |   \ ∠イ  ,イイ|    ,`-' |      頭が良くて数学が出来てかっこいい人。それが必要条件よ。
     |     l^,人|  ` `-'     ゝ  |        さらに Ann.of Math に論文書けば十分条件にもなるわよ。
      |      ` -'\       ー'  人          一番嫌いなのは論文数を増やすためにくだらない論文を書いて
    |        /(l     __/  ヽ、           良い論文の出版を遅らせるお馬鹿な人。
     |       (:::::`‐-、__  |::::`、     ヒニニヽ、         あなたの論文が Ann of Math に accept される確率は?
    |      / `‐-、::::::::::`‐-、::::\   /,ニニ、\            それとも最近は Inv. Math. の方が上かしら?
   |      |::::::::::::::::::|` -、:::::::,ヘ ̄|'、  ヒニ二、 \
.   |      /::::::::::::::::::|::::::::\/:::O`、::\   | '、   \
   |      /:::::::::::::::::::/:::::::::::::::::::::::::::::'、::::\ノ  ヽ、  |
  |      |:::::/:::::::::/:::::::::::::::::::::::::::::::::::'、',::::'、  /:\__/‐、
  |      |/:::::::::::/::::::::::::::::::::::::::::::::::O::| '、::| く::::::::::::: ̄|
   |     /_..-'´ ̄`ー-、:::::::::::::::::::::::::::::::::::|/:/`‐'::\;;;;;;;_|
   |    |/::::::::::::::::::::::\:::::::::::::::::::::::::::::|::/::::|::::/:::::::::::/
    |   /:::::::::::::::::::::::::::::::::|:::::::::::::::::::::O::|::|::::::|:::::::::::::::/
626132人目の素数さん:2013/02/03(日) 09:20:07.33
あああああああ!!
わかりましたありがとうございます!
627132人目の素数さん:2013/02/03(日) 10:15:19.47
微分の問題なんですがy=−x^3+9x   
これって解けないですよね?
628狢 ◆yEy4lYsULH68 :2013/02/03(日) 10:19:08.20
性犯罪者の撲滅という社会の浄化はオマエの仕事や。そやしシカッリせえやナ。

ケケケ狢

>356 名前:132人目の素数さん :2013/02/02(土) 13:39:44.60
> >>355
> そう性犯罪者が何を言っても無駄。
> 性犯罪者を叩くのは名誉毀損でも誹謗中傷でもないしね。
> 社会を浄化するための行為でしかない。
>
> そもそも名誉毀損や誹謗中傷なんてこの板ではほとんどない。
>
629132人目の素数さん:2013/02/03(日) 10:24:00.19
>>628
おはよう
630132人目の素数さん:2013/02/03(日) 10:24:39.99
>>627
解けない、って何をどうすることが出来ないのかが伝わってこない。
631狢 ◆yEy4lYsULH68 :2013/02/03(日) 10:28:15.61
性犯罪者の撲滅という社会の浄化はオマエの仕事や。そやしシカッリせえやナ。

ケケケ狢

>356 名前:132人目の素数さん :2013/02/02(土) 13:39:44.60
> >>355
> そう性犯罪者が何を言っても無駄。
> 性犯罪者を叩くのは名誉毀損でも誹謗中傷でもないしね。
> 社会を浄化するための行為でしかない。
>
> そもそも名誉毀損や誹謗中傷なんてこの板ではほとんどない。
>
632132人目の素数さん:2013/02/03(日) 10:30:58.51
はい解けません
だからあなたはなにも悪くないです
解けない問題をだすアホちゃんが悪いです
解けないことをみぬいたあなたはえらいでちゅね
633狢 ◆yEy4lYsULH68 :2013/02/03(日) 10:33:00.12
性犯罪者の撲滅という社会の浄化はオマエの仕事や。そやしシカッリせえやナ。

ケケケ狢

>356 名前:132人目の素数さん :2013/02/02(土) 13:39:44.60
> >>355
> そう性犯罪者が何を言っても無駄。
> 性犯罪者を叩くのは名誉毀損でも誹謗中傷でもないしね。
> 社会を浄化するための行為でしかない。
>
> そもそも名誉毀損や誹謗中傷なんてこの板ではほとんどない。
>
634132人目の素数さん:2013/02/03(日) 10:44:23.39
>>630
増減表を作りグラフを書けという問題です。言葉足らずですいません
635狢 ◆yEy4lYsULH68 :2013/02/03(日) 10:50:04.54
性犯罪者の撲滅という社会の浄化はオマエの仕事や。そやしシカッリせえやナ。

ケケケ狢

>356 名前:132人目の素数さん :2013/02/02(土) 13:39:44.60
> >>355
> そう性犯罪者が何を言っても無駄。
> 性犯罪者を叩くのは名誉毀損でも誹謗中傷でもないしね。
> 社会を浄化するための行為でしかない。
>
> そもそも名誉毀損や誹謗中傷なんてこの板ではほとんどない。
>
636132人目の素数さん:2013/02/03(日) 10:54:39.35
>>634
y'=-3x^2+9=0
で極値求まるじゃん
637狢 ◆yEy4lYsULH68 :2013/02/03(日) 10:56:13.35
性犯罪者の撲滅という社会の浄化はオマエの仕事や。そやしシカッリせえやナ。

ケケケ狢

>356 名前:132人目の素数さん :2013/02/02(土) 13:39:44.60
> >>355
> そう性犯罪者が何を言っても無駄。
> 性犯罪者を叩くのは名誉毀損でも誹謗中傷でもないしね。
> 社会を浄化するための行為でしかない。
>
> そもそも名誉毀損や誹謗中傷なんてこの板ではほとんどない。
>
638132人目の素数さん:2013/02/03(日) 11:06:42.94
>>636
その式からXがでないのです
y'=−3(x^2−3)=0
この場合x=3、−3でいいんですか?
639132人目の素数さん:2013/02/03(日) 11:08:52.79
>>638
中学生からやり直して来い
640132人目の素数さん:2013/02/03(日) 11:09:43.11
もしかすると中学生なのか?
641132人目の素数さん:2013/02/03(日) 11:14:44.53
>>638
x=±√3
642132人目の素数さん:2013/02/03(日) 11:21:27.35
x^2-5=0
を解け。

x^2-17=0を解け。


x^2-31=0を解け。
643132人目の素数さん:2013/02/03(日) 11:29:33.05
嫌なこった
644狢 ◆yEy4lYsULH68 :2013/02/03(日) 11:38:12.12
性犯罪者の撲滅という社会の浄化はオマエの仕事や。そやしシカッリせえやナ。

ケケケ狢

>356 名前:132人目の素数さん :2013/02/02(土) 13:39:44.60
> >>355
> そう性犯罪者が何を言っても無駄。
> 性犯罪者を叩くのは名誉毀損でも誹謗中傷でもないしね。
> 社会を浄化するための行為でしかない。
>
> そもそも名誉毀損や誹謗中傷なんてこの板ではほとんどない。
>
645132人目の素数さん:2013/02/03(日) 11:48:11.34
わかったああああああああああああああああああああああああ

みんなありがとう!!!
646狢 ◆yEy4lYsULH68 :2013/02/03(日) 11:54:26.70
性犯罪者の撲滅という社会の浄化はオマエの仕事や。そやしシカッリせえやナ。

ケケケ狢

>356 名前:132人目の素数さん :2013/02/02(土) 13:39:44.60
> >>355
> そう性犯罪者が何を言っても無駄。
> 性犯罪者を叩くのは名誉毀損でも誹謗中傷でもないしね。
> 社会を浄化するための行為でしかない。
>
> そもそも名誉毀損や誹謗中傷なんてこの板ではほとんどない。
>
647132人目の素数さん:2013/02/03(日) 12:59:02.20
http://beebee2see.appspot.com/i/azuY-rThBww.jpg

(a)の問題についてなんですが、考え方を教えてくださいませんか?
どのように解答の考え方に行きつくのか分からないのです。

思考回路も記述してくださると、こちら側としても大変幸いです。

どなたかよろしくお願い申し上げます。
648132人目の素数さん:2013/02/03(日) 13:30:10.80
>>647
5,6のときは点は動かず、1,2,3,4のとき、y座標が1大きい点に動く(つまり、点は戻らない)。
(2,0)から出発して、n回目のあと、(n,k)にいるということは
4以下がk回出て、5以上がn-k回出ている。
それゆえ、・・・
と考える。
649132人目の素数さん:2013/02/03(日) 13:35:46.07
http://beebee2see.appspot.com/i/azuYt8zhBww.jpg
ありがとうございます。
問題文の0≦k≦nが気になるんですが、k=0のときもあるので5,6もでることありますよね?
650132人目の素数さん:2013/02/03(日) 13:39:50.05
お前が出来ないのは数学じゃない国語だ
651132人目の素数さん:2013/02/03(日) 13:43:57.77
あ、わかりました
普通に書いてましたね。

そうなんですよ、國語ができなくて辛いです
652132人目の素数さん:2013/02/03(日) 13:48:09.63
>>649
ちょっと意味がよくわからない。
5、6が出ることがある理由はk=0があるからではないと思うが。
サイコロ振ったら5、6が出ることもあるだろ。
653132人目の素数さん:2013/02/03(日) 17:20:17.93
>>649
その答えの中で
(2/3)^k・(1/3)^(n-k)・C[n,k]
としてk=0の場合も含めて、5,6が出る場合もちゃんと反映されてるのが読めるか?
654132人目の素数さん:2013/02/03(日) 17:31:07.87
もう、既に理解されてるのがこのスレッドのレスから読めるか?
655132人目の素数さん:2013/02/03(日) 17:34:30.00
          __ノ)-'´ ̄ ̄`ー- 、_
        , '´  _. -‐'''"二ニニ=-`ヽ、
      /   /:::::; -‐''"        `ーノ
     /   /:::::/           \
     /    /::::::/          | | |  |
     |   |:::::/ /     |  | | | |  |
      |   |::/ / / |  | ||  | | ,ハ .| ,ハ|
      |   |/ / / /| ,ハノ| /|ノレ,ニ|ル' 
     |   |  | / / レ',二、レ′ ,ィイ|゙/   私は只の数ヲタなんかとは付き合わないわ。
.     |   \ ∠イ  ,イイ|    ,`-' |      頭が良くて数学が出来てかっこいい人。それが必要条件よ。
     |     l^,人|  ` `-'     ゝ  |        さらに Ann.of Math に論文書けば十分条件にもなるわよ。
      |      ` -'\       ー'  人          一番嫌いなのは論文数を増やすためにくだらない論文を書いて
    |        /(l     __/  ヽ、           良い論文の出版を遅らせるお馬鹿な人。
     |       (:::::`‐-、__  |::::`、     ヒニニヽ、         あなたの論文が Ann of Math に accept される確率は?
    |      / `‐-、::::::::::`‐-、::::\   /,ニニ、\            それとも最近は Inv. Math. の方が上かしら?
   |      |::::::::::::::::::|` -、:::::::,ヘ ̄|'、  ヒニ二、 \
.   |      /::::::::::::::::::|::::::::\/:::O`、::\   | '、   \
   |      /:::::::::::::::::::/:::::::::::::::::::::::::::::'、::::\ノ  ヽ、  |
  |      |:::::/:::::::::/:::::::::::::::::::::::::::::::::::'、',::::'、  /:\__/‐、
  |      |/:::::::::::/::::::::::::::::::::::::::::::::::O::| '、::| く::::::::::::: ̄|
   |     /_..-'´ ̄`ー-、:::::::::::::::::::::::::::::::::::|/:/`‐'::\;;;;;;;_|
   |    |/::::::::::::::::::::::\:::::::::::::::::::::::::::::|::/::::|::::/:::::::::::/
    |   /:::::::::::::::::::::::::::::::::|:::::::::::::::::::::O::|::|::::::|:::::::::::::::/
656132人目の素数さん:2013/02/03(日) 18:04:47.26
>>654
読めないよ、あほ。
657132人目の素数さん:2013/02/03(日) 18:35:11.48
それはどちらでしょうか?
考えて見てください。
658132人目の素数さん:2013/02/03(日) 18:45:31.94
sin(x)-e^x=0 となるxの値を求めたいのですが、高校数学の範囲で可能ですか?
659132人目の素数さん:2013/02/03(日) 19:09:49.17
tan(x)=1/(1-i)
660132人目の素数さん:2013/02/03(日) 20:20:53.05
年平均(円/ドル)(円/ユーロ)
2003 116.49 ↑ 130.92 
2004 108.28小泉134.40
2005 109.64 ↓ 137.67
2006 116.25  146.01
      安倍
2007 117.93  161.24
      福田
2008 104.23  152.41
      麻生
2009 93.52  130.19
      鳩山
2010 88.09  116.44
      管
2011 79.97  111.06
      野田
2012 78.71  102.65
  今92円だが126円だが、どこが円安?何か文句ある?
661132人目の素数さん:2013/02/03(日) 21:04:31.88
麻生さんが円安と言っているので
662132人目の素数さん:2013/02/03(日) 22:41:33.37
そのうちに、AV女優で
円(まどか)安(あん)
なんてのが出てくるぞー
663132人目の素数さん:2013/02/04(月) 00:37:11.55
x-y平面上における曲線C:F(x,y)=0を考える。曲線Cをx軸方向にもy軸方向にもa倍した曲線をC'とするとその方程式はF(x/a,y/a)=0である。このことを原点Oを相似の中心として曲線C上の任意の点(x_0,y_0)が曲線C'上のどの点に対応しているかに着目して説明せよ
どうすればいいのか分かりません。ヒントがあれば教えてください
664132人目の素数さん:2013/02/04(月) 00:40:00.39
665132人目の素数さん:2013/02/04(月) 00:48:01.04
>>663
CとC'は相似だから、(x_0,y_0)に対応する点を(x'_0, y'_0)として、
原点が相似の中心であることも使った相似比の式が立てれる
666132人目の素数さん:2013/02/04(月) 00:58:22.75
>>665
1:a=√(x_0^2+y_0^2):√x'_0^2+y'_0^2)みたいな感じですか?
667132人目の素数さん:2013/02/04(月) 01:03:37.11
>>666
それも成り立つがもっとシンプルに、適当に直角三角形考えて、
例えばx_0とx'_0とかが個別にどうなるかの相似比の式を立てる
中3レベルの問題
668132人目の素数さん:2013/02/04(月) 01:08:40.52
>>667
分かりやすく有難うございました
669132人目の素数さん:2013/02/04(月) 01:16:07.27
今参考書の問題を解説見ながら解いていて、
x^2+5≧0 は
x^2≧0, 5>0
となると書いてあるのですが、なぜそうなるのかがさっぱり分かりません。
わかる方がいらっしゃったら教えてください。
670132人目の素数さん:2013/02/04(月) 01:21:27.56
>>669
それは君の解釈だろ、多分そうは書いてないんじゃね
まず本に書いてある通りにこっちでも書かんと答えようがないぞ
671132人目の素数さん:2013/02/04(月) 01:28:21.94
そりゃx^2は0以上だろ
672132人目の素数さん:2013/02/04(月) 01:28:37.20
A(√3,0)B(−√3,0)があり、APB=60°となる点P(x,y)の軌跡の求め方を教えてください。
673132人目の素数さん:2013/02/04(月) 01:30:23.98
円周角の定理
中3レベル
674132人目の素数さん:2013/02/04(月) 01:31:08.82
675132人目の素数さん:2013/02/04(月) 01:37:58.56
>>674
ありがとうございます。余弦定理ですね
676132人目の素数さん:2013/02/04(月) 01:53:09.74
円x^2+y^2+4x-6y+12=0を直線3x-4y+8=0の回りに回転してできる立体の体積を求めよ。

この問題がわかりません;
知恵袋にも同じ問題が書いてあったのですが
途中で間違えていてよくわからないのでお願いします
677132人目の素数さん:2013/02/04(月) 01:53:17.56
>>670
数学1、絶対値付きの二次不等式の問題の
場合分けの途中の式で、解説にその通りに書いてあります。

問題は、
|x-4|-|x+3|≦x^2-2 で、

4≦x のときは
x-4-x-3≦x^2-2
-x^2-5≦0
x^2+5≧0
このあと、
x^2≧, 5>0
すべての実数xにおいて成立する と書いてあります。
ax^2+bx+cでもないし、因数分解もできないし、どうしてx^2≧, 5>0なのかがわからないんです・・・
678132人目の素数さん:2013/02/04(月) 01:56:59.42
          __ノ)-'´ ̄ ̄`ー- 、_
        , '´  _. -‐'''"二ニニ=-`ヽ、
      /   /:::::; -‐''"        `ーノ
     /   /:::::/           \
     /    /::::::/          | | |  |
     |   |:::::/ /     |  | | | |  |
      |   |::/ / / |  | ||  | | ,ハ .| ,ハ|
      |   |/ / / /| ,ハノ| /|ノレ,ニ|ル' 
     |   |  | / / レ',二、レ′ ,ィイ|゙/   私は只の数ヲタなんかとは付き合わないわ。
.     |   \ ∠イ  ,イイ|    ,`-' |      頭が良くて数学が出来てかっこいい人。それが必要条件よ。
     |     l^,人|  ` `-'     ゝ  |        さらに Ann.of Math に論文書けば十分条件にもなるわよ。
      |      ` -'\       ー'  人          一番嫌いなのは論文数を増やすためにくだらない論文を書いて
    |        /(l     __/  ヽ、           良い論文の出版を遅らせるお馬鹿な人。
     |       (:::::`‐-、__  |::::`、     ヒニニヽ、         あなたの論文が Ann of Math に accept される確率は?
    |      / `‐-、::::::::::`‐-、::::\   /,ニニ、\            それとも最近は Inv. Math. の方が上かしら?
   |      |::::::::::::::::::|` -、:::::::,ヘ ̄|'、  ヒニ二、 \
.   |      /::::::::::::::::::|::::::::\/:::O`、::\   | '、   \
   |      /:::::::::::::::::::/:::::::::::::::::::::::::::::'、::::\ノ  ヽ、  |
  |      |:::::/:::::::::/:::::::::::::::::::::::::::::::::::'、',::::'、  /:\__/‐、
  |      |/:::::::::::/::::::::::::::::::::::::::::::::::O::| '、::| く::::::::::::: ̄|
   |     /_..-'´ ̄`ー-、:::::::::::::::::::::::::::::::::::|/:/`‐'::\;;;;;;;_|
   |    |/::::::::::::::::::::::\:::::::::::::::::::::::::::::|::/::::|::::/:::::::::::/
    |   /:::::::::::::::::::::::::::::::::|:::::::::::::::::::::O::|::|::::::|:::::::::::::::/
679132人目の素数さん:2013/02/04(月) 01:59:29.08
ちなみに
x^2≧0と5>0自体は分かるのですが、
なぜx^2+5≧0のあとに
b<x<a とか x<b, a<x の形で書かれていないのかがわかりません・・・。
680132人目の素数さん:2013/02/04(月) 02:10:27.47
>>679
任意の実数 x に対して x^2 ≧ 0 であり, また明らかに 5 > 0であるから、これらから導かれる結論として
x^2 + 5 ≧ 0 を満たす x は任意の実数である、ということ
681132人目の素数さん:2013/02/04(月) 02:49:47.17
>>680
ありがとうございます。
なんとなく理解できました。
こういう証明のしかたもあるということを覚えておこうと思います。
682狢 ◆yEy4lYsULH68 :2013/02/04(月) 11:18:47.89
性犯罪者の撲滅という社会の浄化はオマエの仕事や。そやしシカッリせえやナ。

ケケケ狢

>356 名前:132人目の素数さん :2013/02/02(土) 13:39:44.60
> >>355
> そう性犯罪者が何を言っても無駄。
> 性犯罪者を叩くのは名誉毀損でも誹謗中傷でもないしね。
> 社会を浄化するための行為でしかない。
>
> そもそも名誉毀損や誹謗中傷なんてこの板ではほとんどない。
>
683132人目の素数さん:2013/02/04(月) 12:39:08.30
>>681
「証明のしかた」じゃない
3つの区間 (−∞, −3], [−3, 4], [4, ∞) に分けて調べて
[4, ∞) では問題なく成り立ってると確認しただけだよ
4≦x のときに |x-4|-|x+3|≦x^2-2 が成り立つことを自力で確認できるようにならなきゃダメ
684132人目の素数さん:2013/02/04(月) 12:44:32.48
うるせー
685132人目の素数さん:2013/02/04(月) 13:34:30.21
>>676
円の中心と直線の間の距離と、円の半径を求める
686132人目の素数さん:2013/02/04(月) 14:05:50.38
>>683
すみません、
4≦xのとき……@
(省略)
x^2≧0, 5>0
すべての実数xにおいて成立する……A
@Aより4≦x
と書いてありました。

|x-4|-|x+3|≦x^2-2
x^2+5≧0 は、
左辺は明らかに正の整数(x^2≧0)だし、
また5>0なので、
xにどんな実数を入れても左辺>0で、式は成立する。
でも、@の条件より、4≦x となる
という考え方でいいですか?

この形の確認の仕方は始めて見て、解説もなくて、3日前からモヤモヤしたままでした。式が成り立つかの確認をしているってことを覚えておこうとおもいます。
687132人目の素数さん:2013/02/04(月) 14:25:13.11
>>676
まず素直にグラフかいてどんな立体になるか確認しろ
そうすりゃおのずと分かる
688132人目の素数さん:2013/02/04(月) 14:58:11.17
>>676
パップス・ギュルダンの定理
689132人目の素数さん:2013/02/04(月) 15:18:15.50
>>403
おい!ちゃんと>>358のこともバカにしろよ!
たわけめ!
690132人目の素数さん:2013/02/04(月) 15:50:19.68
>>687
わかりました。グラフ書いてもうすこし考えてみます。
ありがとうございます。
691狢 ◆yEy4lYsULH68 :2013/02/04(月) 15:57:14.96
性犯罪者の撲滅という社会の浄化はオマエの仕事や。そやしシカッリせえやナ。

ケケケ狢

>356 名前:132人目の素数さん :2013/02/02(土) 13:39:44.60
> >>355
> そう性犯罪者が何を言っても無駄。
> 性犯罪者を叩くのは名誉毀損でも誹謗中傷でもないしね。
> 社会を浄化するための行為でしかない。
>
> そもそも名誉毀損や誹謗中傷なんてこの板ではほとんどない。
>
692132人目の素数さん:2013/02/04(月) 19:19:51.02
          __ノ)-'´ ̄ ̄`ー- 、_
        , '´  _. -‐'''"二ニニ=-`ヽ、
      /   /:::::; -‐''"        `ーノ
     /   /:::::/           \
     /    /::::::/          | | |  |
     |   |:::::/ /     |  | | | |  |
      |   |::/ / / |  | ||  | | ,ハ .| ,ハ|
      |   |/ / / /| ,ハノ| /|ノレ,ニ|ル' 
     |   |  | / / レ',二、レ′ ,ィイ|゙/   私は只の数ヲタなんかとは付き合わないわ。
.     |   \ ∠イ  ,イイ|    ,`-' |      頭が良くて数学が出来てかっこいい人。それが必要条件よ。
     |     l^,人|  ` `-'     ゝ  |        さらに Ann.of Math に論文書けば十分条件にもなるわよ。
      |      ` -'\       ー'  人          一番嫌いなのは論文数を増やすためにくだらない論文を書いて
    |        /(l     __/  ヽ、           良い論文の出版を遅らせるお馬鹿な人。
     |       (:::::`‐-、__  |::::`、     ヒニニヽ、         あなたの論文が Ann of Math に accept される確率は?
    |      / `‐-、::::::::::`‐-、::::\   /,ニニ、\            それとも最近は Inv. Math. の方が上かしら?
   |      |::::::::::::::::::|` -、:::::::,ヘ ̄|'、  ヒニ二、 \
.   |      /::::::::::::::::::|::::::::\/:::O`、::\   | '、   \
   |      /:::::::::::::::::::/:::::::::::::::::::::::::::::'、::::\ノ  ヽ、  |
  |      |:::::/:::::::::/:::::::::::::::::::::::::::::::::::'、',::::'、  /:\__/‐、
  |      |/:::::::::::/::::::::::::::::::::::::::::::::::O::| '、::| く::::::::::::: ̄|
   |     /_..-'´ ̄`ー-、:::::::::::::::::::::::::::::::::::|/:/`‐'::\;;;;;;;_|
   |    |/::::::::::::::::::::::\:::::::::::::::::::::::::::::|::/::::|::::/:::::::::::/
    |   /:::::::::::::::::::::::::::::::::|:::::::::::::::::::::O::|::|::::::|:::::::::::::::/
693あのこうちやんは始皇帝だった:2013/02/04(月) 19:24:14.02
>>692

テメ〜、いいかげんにしねえと、ブッ殺すぞ!

 30代の、無職の、知的障害の、女性恐怖症の、頭デッカチの虚弱児・ひ弱の、ゴミ・クズ・カス・無能・虫けらのクソガキ!

 死ね!!!!!!!!!!!!!!!!!!!!!!
694132人目の素数さん:2013/02/04(月) 19:27:13.43
          __ノ)-'´ ̄ ̄`ー- 、_
        , '´  _. -‐'''"二ニニ=-`ヽ、
      /   /:::::; -‐''"        `ーノ
     /   /:::::/           \
     /    /::::::/          | | |  |
     |   |:::::/ /     |  | | | |  |
      |   |::/ / / |  | ||  | | ,ハ .| ,ハ|
      |   |/ / / /| ,ハノ| /|ノレ,ニ|ル' 
     |   |  | / / レ',二、レ′ ,ィイ|゙/   私は只の数ヲタなんかとは付き合わないわ。
.     |   \ ∠イ  ,イイ|    ,`-' |      頭が良くて数学が出来てかっこいい人。それが必要条件よ。
     |     l^,人|  ` `-'     ゝ  |        さらに Ann.of Math に論文書けば十分条件にもなるわよ。
      |      ` -'\       ー'  人          一番嫌いなのは論文数を増やすためにくだらない論文を書いて
    |        /(l     __/  ヽ、           良い論文の出版を遅らせるお馬鹿な人。
     |       (:::::`‐-、__  |::::`、     ヒニニヽ、         あなたの論文が Ann of Math に accept される確率は?
    |      / `‐-、::::::::::`‐-、::::\   /,ニニ、\            それとも最近は Inv. Math. の方が上かしら?
   |      |::::::::::::::::::|` -、:::::::,ヘ ̄|'、  ヒニ二、 \
.   |      /::::::::::::::::::|::::::::\/:::O`、::\   | '、   \
   |      /:::::::::::::::::::/:::::::::::::::::::::::::::::'、::::\ノ  ヽ、  |
  |      |:::::/:::::::::/:::::::::::::::::::::::::::::::::::'、',::::'、  /:\__/‐、
  |      |/:::::::::::/::::::::::::::::::::::::::::::::::O::| '、::| く::::::::::::: ̄|
   |     /_..-'´ ̄`ー-、:::::::::::::::::::::::::::::::::::|/:/`‐'::\;;;;;;;_|
   |    |/::::::::::::::::::::::\:::::::::::::::::::::::::::::|::/::::|::::/:::::::::::/
    |   /:::::::::::::::::::::::::::::::::|:::::::::::::::::::::O::|::|::::::|:::::::::::::::/
695132人目の素数さん:2013/02/04(月) 22:15:29.05
ここで上から目線のレスは企業の研究グループにも加われず、
だからといって高校教師はイヤだとする半ニートの落ちこぼれだ。
高校の先生やればわかるよ。底辺校の生徒相手に数学を教えるとなると
このスレでのいらいらの比じゃない。
こんなところで怒りを爆発してるやつはとてもじゃないが、
高校の教師にはなれない。あんだけ社会的地位の低い高校教師でさえもな。
696132人目の素数さん:2013/02/04(月) 22:16:34.21
という上から目線のレス
697132人目の素数さん:2013/02/04(月) 23:09:48.51
今ちょっと化学をやってるんですけど
X^2 + 1.0×10^-8 - 1.0×10^-14 = 0はどの様に解けばよいのでしたっけ
x自体は 9.5×10^-8 らしいのですが…… 
どうかよろしくお願いします (解の公式使うんですよね??)
698132人目の素数さん:2013/02/04(月) 23:15:38.04
699132人目の素数さん:2013/02/04(月) 23:22:28.89
解の公式を使うと分かっているのなら公式に数値を代入するだけで元丸が
700132人目の素数さん:2013/02/04(月) 23:32:29.48
          __ノ)-'´ ̄ ̄`ー- 、_
        , '´  _. -‐'''"二ニニ=-`ヽ、
      /   /:::::; -‐''"        `ーノ
     /   /:::::/           \
     /    /::::::/          | | |  |
     |   |:::::/ /     |  | | | |  |
      |   |::/ / / |  | ||  | | ,ハ .| ,ハ|
      |   |/ / / /| ,ハノ| /|ノレ,ニ|ル' 
     |   |  | / / レ',二、レ′ ,ィイ|゙/   私は只の数ヲタなんかとは付き合わないわ。
.     |   \ ∠イ  ,イイ|    ,`-' |      頭が良くて数学が出来てかっこいい人。それが必要条件よ。
     |     l^,人|  ` `-'     ゝ  |        さらに Ann.of Math に論文書けば十分条件にもなるわよ。
      |      ` -'\       ー'  人          一番嫌いなのは論文数を増やすためにくだらない論文を書いて
    |        /(l     __/  ヽ、           良い論文の出版を遅らせるお馬鹿な人。
     |       (:::::`‐-、__  |::::`、     ヒニニヽ、         あなたの論文が Ann of Math に accept される確率は?
    |      / `‐-、::::::::::`‐-、::::\   /,ニニ、\            それとも最近は Inv. Math. の方が上かしら?
   |      |::::::::::::::::::|` -、:::::::,ヘ ̄|'、  ヒニ二、 \
.   |      /::::::::::::::::::|::::::::\/:::O`、::\   | '、   \
   |      /:::::::::::::::::::/:::::::::::::::::::::::::::::'、::::\ノ  ヽ、  |
  |      |:::::/:::::::::/:::::::::::::::::::::::::::::::::::'、',::::'、  /:\__/‐、
  |      |/:::::::::::/::::::::::::::::::::::::::::::::::O::| '、::| く::::::::::::: ̄|
   |     /_..-'´ ̄`ー-、:::::::::::::::::::::::::::::::::::|/:/`‐'::\;;;;;;;_|
   |    |/::::::::::::::::::::::\:::::::::::::::::::::::::::::|::/::::|::::/:::::::::::/
    |   /:::::::::::::::::::::::::::::::::|:::::::::::::::::::::O::|::|::::::|:::::::::::::::/
701132人目の素数さん:2013/02/05(火) 00:03:10.62
中学生レベルだろ
中学校卒業すんなよアホ
出来てないのに出てクンなks
もいっかい中学やりなおせよ
それともカンニング入試でガッコに入ったのか
低能無能が中学レベルの質問でわめいてんじゃねーよ
702132人目の素数さん:2013/02/05(火) 01:48:27.27
>>686
全然ダメ
「4≦xのとき……@」を前提にしてるのに「@の条件より、4≦x となる」なんて馬鹿げた論理をやってるんじゃない
確認すべきなのは「4≦x のとき |x-4|-|x+3|≦x^2-2 になる」だ
だいたい「4≦x」と「|x-4|-|x+3|≦x^2-2」と「x^2+5≧0」の関係は分かってるのか?
「4≦x」と「|x-4|-|x+3|≦x^2-2」から「x^2+5≧0」が出るんだぞ
703132人目の素数さん:2013/02/05(火) 02:08:58.50
|x-4|-|x+3|≦x^2-2

x^2+5≧0(x≧4のとき)、
x^2+2x-3≧0(-3≦x<4のとき)、
x^2-9≧0(x<-3のとき)

x≧4のときは、実数xについてx^2>0、5>0ゆえ、x^2+5>0であるからx^2+5≧0は常に成り立つ。よってx≧4は求めるxの範囲の一部である。
-3≦x<4のとき、x^2+2x-3=(x+3)(x-1)≧0の解はx≦-3またはx≧1ゆえ、x=-3または1≦x<4 が求めるxの範囲の一部である。
x<-3のとき、x^2-9=(x-3)(x+3)≧0の解はx≦-3またはx≧3ゆえ、x<-3が求めるxの範囲の一部である。

以上総合して、求めるxの範囲は、 x≦-3 または1≦x である。
704132人目の素数さん:2013/02/05(火) 04:05:02.97
座標空間において円柱面C={(x−1)^2+y^2−1}が
半球面S={x^2+y^2+z^2=4,z≧0}
によって切り取られる部分Fの面積を求めよ
という問題で、
円柱と球の交点Qの座標がQ(1+cosφ,sinφ,2sin(φ/2))
とあらわされるのは分かるのですが、

Fを平面上に展開してXZ座標を定めると、
F:0≦z≦2sin(X/2)(0≦X≦2π)
より、∫[0→2π]2sin(X/2)dX=8と求められているのですが、
このXの意味がよく分かりません。もしかして円上に座標をとっているのでしょうか?
なぜこのように取れるのか・・・
705132人目の素数さん:2013/02/05(火) 04:49:49.48
とりあえず1行目ラスト3文字あたりでチョンボしているようにみえる
706132人目の素数さん:2013/02/05(火) 04:51:20.61
>>701
何を言っとるんだ、君は!
707132人目の素数さん:2013/02/05(火) 05:09:39.53
>>704
まあチョンボの件はいいやどうせC={(x−1)^2+y^2−1=0}でしょ
で、その解説、文字にXを使うとか意味不明だね
普通はφそのまま使うところでしょ、それで何の問題もないはず

S=∫[0,2π]|2sin(φ/2)|dφ  φ/2=ξ、dφ=2dξ
=2*2∫[0,π]sinξdξ
=4[-cosξ]_[ξ=0,π]
=4{1-(-1)}
=8
708132人目の素数さん:2013/02/05(火) 08:24:28.31
          __ノ)-'´ ̄ ̄`ー- 、_
        , '´  _. -‐'''"二ニニ=-`ヽ、
      /   /:::::; -‐''"        `ーノ
     /   /:::::/           \
     /    /::::::/          | | |  |
     |   |:::::/ /     |  | | | |  |
      |   |::/ / / |  | ||  | | ,ハ .| ,ハ|
      |   |/ / / /| ,ハノ| /|ノレ,ニ|ル' 
     |   |  | / / レ',二、レ′ ,ィイ|゙/   私は只の数ヲタなんかとは付き合わないわ。
.     |   \ ∠イ  ,イイ|    ,`-' |      頭が良くて数学が出来てかっこいい人。それが必要条件よ。
     |     l^,人|  ` `-'     ゝ  |        さらに Ann.of Math に論文書けば十分条件にもなるわよ。
      |      ` -'\       ー'  人          一番嫌いなのは論文数を増やすためにくだらない論文を書いて
    |        /(l     __/  ヽ、           良い論文の出版を遅らせるお馬鹿な人。
     |       (:::::`‐-、__  |::::`、     ヒニニヽ、         あなたの論文が Ann of Math に accept される確率は?
    |      / `‐-、::::::::::`‐-、::::\   /,ニニ、\            それとも最近は Inv. Math. の方が上かしら?
   |      |::::::::::::::::::|` -、:::::::,ヘ ̄|'、  ヒニ二、 \
.   |      /::::::::::::::::::|::::::::\/:::O`、::\   | '、   \
   |      /:::::::::::::::::::/:::::::::::::::::::::::::::::'、::::\ノ  ヽ、  |
  |      |:::::/:::::::::/:::::::::::::::::::::::::::::::::::'、',::::'、  /:\__/‐、
  |      |/:::::::::::/::::::::::::::::::::::::::::::::::O::| '、::| く::::::::::::: ̄|
   |     /_..-'´ ̄`ー-、:::::::::::::::::::::::::::::::::::|/:/`‐'::\;;;;;;;_|
   |    |/::::::::::::::::::::::\:::::::::::::::::::::::::::::|::/::::|::::/:::::::::::/
    |   /:::::::::::::::::::::::::::::::::|:::::::::::::::::::::O::|::|::::::|:::::::::::::::/
709132人目の素数さん:2013/02/05(火) 09:38:21.94
平面上にある異なる4つの点A〜Dについて
 AB^2 + CD^2 = AD^2 + BC^2 が成り立つとき、線分ACとBDは垂直に交わるといえるですか。
710132人目の素数さん:2013/02/05(火) 09:54:06.46
>>709
内積計算で
711132人目の素数さん:2013/02/05(火) 10:09:53.81
>>709
Aからのベクトルに直してゴニョゴニョやったら自然に出る
712132人目の素数さん:2013/02/05(火) 10:16:41.17
いえない
713132人目の素数さん:2013/02/05(火) 10:27:28.19
          __ノ)-'´ ̄ ̄`ー- 、_
        , '´  _. -‐'''"二ニニ=-`ヽ、
      /   /:::::; -‐''"        `ーノ
     /   /:::::/           \
     /    /::::::/          | | |  |
     |   |:::::/ /     |  | | | |  |
      |   |::/ / / |  | ||  | | ,ハ .| ,ハ|
      |   |/ / / /| ,ハノ| /|ノレ,ニ|ル' 
     |   |  | / / レ',二、レ′ ,ィイ|゙/   私は只の数ヲタなんかとは付き合わないわ。
.     |   \ ∠イ  ,イイ|    ,`-' |      頭が良くて数学が出来てかっこいい人。それが必要条件よ。
     |     l^,人|  ` `-'     ゝ  |        さらに Ann.of Math に論文書けば十分条件にもなるわよ。
      |      ` -'\       ー'  人          一番嫌いなのは論文数を増やすためにくだらない論文を書いて
    |        /(l     __/  ヽ、           良い論文の出版を遅らせるお馬鹿な人。
     |       (:::::`‐-、__  |::::`、     ヒニニヽ、         あなたの論文が Ann of Math に accept される確率は?
    |      / `‐-、::::::::::`‐-、::::\   /,ニニ、\            それとも最近は Inv. Math. の方が上かしら?
   |      |::::::::::::::::::|` -、:::::::,ヘ ̄|'、  ヒニ二、 \
.   |      /::::::::::::::::::|::::::::\/:::O`、::\   | '、   \
   |      /:::::::::::::::::::/:::::::::::::::::::::::::::::'、::::\ノ  ヽ、  |
  |      |:::::/:::::::::/:::::::::::::::::::::::::::::::::::'、',::::'、  /:\__/‐、
  |      |/:::::::::::/::::::::::::::::::::::::::::::::::O::| '、::| く::::::::::::: ̄|
   |     /_..-'´ ̄`ー-、:::::::::::::::::::::::::::::::::::|/:/`‐'::\;;;;;;;_|
   |    |/::::::::::::::::::::::\:::::::::::::::::::::::::::::|::/::::|::::/:::::::::::/
714132人目の素数さん:2013/02/05(火) 12:45:01.31
0,1,2,3,4,5の数字が書いてある玉が一つずつある。
(1)全ての玉を並べる時、1,2,3が隣り合う順列の総数を求めよ。
(2)全ての玉を並べる時、1,2,3が隣合わない順列の総数を求めよ。

(1)は4!3!で144通りだというのはわかる。
(2)は全ての玉を並べる順列である6!通りから(1)の144通りを引いた576通りじゃないのはなぜ?
715132人目の素数さん:2013/02/05(火) 13:01:22.72
>>714
問題文、それで正しいの?
たぶん、1と2と3はお互いどれとも隣り合わないという意味なんだろうと思う。
だから、120345とかも除かれる。
答えどうなってるの?
716132人目の素数さん:2013/02/05(火) 17:33:15.54
・1,2,3が(どの2つも互いに)隣り合わない順列の総数を求めよ。
・1,2,3が(3つ同時には)隣合わない順列の総数を求めよ。


前者なら
@*@*@*@
↑で*に0,4,5、@に1,2,3(と何も当てない)を割り当てる場合の数
3! * 4!=144(通り)

後者なら>>714の通り(全体)-(1)=576(通り)

多分前者なんだろうけど
717狢 ◆yEy4lYsULH68 :2013/02/05(火) 17:37:56.01
性犯罪者の撲滅という社会の浄化はオマエの仕事や。そやしシカッリせえやナ。

ケケケ狢

>356 名前:132人目の素数さん :2013/02/02(土) 13:39:44.60
> >>355
> そう性犯罪者が何を言っても無駄。
> 性犯罪者を叩くのは名誉毀損でも誹謗中傷でもないしね。
> 社会を浄化するための行為でしかない。
>
> そもそも名誉毀損や誹謗中傷なんてこの板ではほとんどない。
>
718あのこうちやんは始皇帝だった:2013/02/05(火) 19:37:20.81
 テメ〜ら、いいかげんにしねえと、ブッ殺すぞ!

 30代と56歳の、無職の、知的障害の、女性恐怖症の、頭デッカチの虚弱児・ひ弱の、ゴミ・クズ・カス・無能・虫けらのクソガキども!

 死ね!!!!!!!!!!!!!!!!!!!!!!
719132人目の素数さん:2013/02/05(火) 19:47:44.89
いい加減にしてるから殺しに来てみろバ〜カww
720132人目の素数さん:2013/02/05(火) 20:13:28.17
航空機事故に関する続報
このニュースはテレビでは流れない重要なことなので

【航空】ボーイング787バッテリー問題、米NTSBが内部告発者の情報に関心[13/01/25]
http://anago.2ch.net/test/read.cgi/bizplus/1359075767/
電池本体=日本
充電器=英系アメリカ工場
過充電防止装置=フランス

内部告発者の1人、マイケル・レオン氏は英メギット傘下セキュラプレーン・テクノロジーズ社が充電装置の出荷を急いでいたと主張。
この充電装置について、仕様と一致せず、正常に作動しなくなる可能性を指摘していた。
721132人目の素数さん:2013/02/05(火) 22:47:09.80
>>707
C={(x−1)^2+y^2−1=0}でした。
交点Qの座標がQ(1+cosφ,sinφ,2sin(φ/2)) までは分かるのですが、
なぜZ座標を積分するだけで求まるのでしょうか?
722132人目の素数さん:2013/02/05(火) 23:00:06.15
>>721
>半球面S={x^2+y^2+z^2=4,z≧0}
と平面 z=0 によって切り取られる部分の面積を求める問題ならz-0=zだからそうなる。
723721:2013/02/06(水) 00:21:33.36
問題はこれであっているのですが、
円柱を半球面のz≧0の部分で切り取ると、д←こんなのが取れると思うんですが、
これって高さ(z成分)だけでなく、縦横(x、y成分)も持ってますよね?
なぜ高さの積分だけで求まるのか・・・???
724132人目の素数さん:2013/02/06(水) 00:31:21.52
>>723
まあ係数サボらずに書けば
S=∫[0,2π]1*|2sin(φ/2)|dφ
で、この1はC={(x−1)^2+y^2−1=0}が半径1だから

半径rならもちろんS=∫[0,2π]r*{z_1(φ)-z_2(φ)}dφとかそういう感じ
725132人目の素数さん:2013/02/06(水) 00:37:15.17
>>723
問題文が本当に
>座標空間において円柱面C={(x−1)^2+y^2−1 = 0}が
>半球面S={x^2+y^2+z^2=4,z≧0}
>によって切り取られる部分Fの面積を求めよ
だけだったら、Sはふたの無いボウルみたいな面だから、無限に長い円柱面Cを二つに切り分けるだけだとひねくれたくなる。
726132人目の素数さん:2013/02/06(水) 00:38:26.81
          __ノ)-'´ ̄ ̄`ー- 、_
        , '´  _. -‐'''"二ニニ=-`ヽ、
      /   /:::::; -‐''"        `ーノ
     /   /:::::/           \
     /    /::::::/          | | |  |
     |   |:::::/ /     |  | | | |  |
      |   |::/ / / |  | ||  | | ,ハ .| ,ハ|
      |   |/ / / /| ,ハノ| /|ノレ,ニ|ル' 
     |   |  | / / レ',二、レ′ ,ィイ|゙/   私は只の数ヲタなんかとは付き合わないわ。
.     |   \ ∠イ  ,イイ|    ,`-' |      頭が良くて数学が出来てかっこいい人。それが必要条件よ。
     |     l^,人|  ` `-'     ゝ  |        さらに Ann.of Math に論文書けば十分条件にもなるわよ。
      |      ` -'\       ー'  人          一番嫌いなのは論文数を増やすためにくだらない論文を書いて
    |        /(l     __/  ヽ、           良い論文の出版を遅らせるお馬鹿な人。
     |       (:::::`‐-、__  |::::`、     ヒニニヽ、         あなたの論文が Ann of Math に accept される確率は?
    |      / `‐-、::::::::::`‐-、::::\   /,ニニ、\            それとも最近は Inv. Math. の方が上かしら?
   |      |::::::::::::::::::|` -、:::::::,ヘ ̄|'、  ヒニ二、 \
.   |      /::::::::::::::::::|::::::::\/:::O`、::\   | '、   \
   |      /:::::::::::::::::::/:::::::::::::::::::::::::::::'、::::\ノ  ヽ、  |
  |      |:::::/:::::::::/:::::::::::::::::::::::::::::::::::'、',::::'、  /:\__/‐、
  |      |/:::::::::::/::::::::::::::::::::::::::::::::::O::| '、::| く::::::::::::: ̄|
   |     /_..-'´ ̄`ー-、:::::::::::::::::::::::::::::::::::|/:/`‐'::\;;;;;;;_|
   |    |/::::::::::::::::::::::\:::::::::::::::::::::::::::::|::/::::|::::/:::::::::::/
    |   /:::::::::::::::::::::::::::::::::|:::::::::::::::::::::O::|::|::::::|:::::::::::::::/
727132人目の素数さん:2013/02/06(水) 00:39:32.33
          __ノ)-'´ ̄ ̄`ー- 、_
        , '´  _. -‐'''"二ニニ=-`ヽ、
      /   /:::::; -‐''"        `ーノ
     /   /:::::/           \
     /    /::::::/          | | |  |
     |   |:::::/ /     |  | | | |  |
      |   |::/ / / |  | ||  | | ,ハ .| ,ハ|
      |   |/ / / /| ,ハノ| /|ノレ,ニ|ル' 
     |   |  | / / レ',二、レ′ ,ィイ|゙/   私は只の数ヲタなんかとは付き合わないわ。
.     |   \ ∠イ  ,イイ|    ,`-' |      頭が良くて数学が出来てかっこいい人。それが必要条件よ。
     |     l^,人|  ` `-'     ゝ  |        さらに Ann.of Math に論文書けば十分条件にもなるわよ。
      |      ` -'\       ー'  人          一番嫌いなのは論文数を増やすためにくだらない論文を書いて
    |        /(l     __/  ヽ、           良い論文の出版を遅らせるお馬鹿な人。
     |       (:::::`‐-、__  |::::`、     ヒニニヽ、         あなたの論文が Ann of Math に accept される確率は?
    |      / `‐-、::::::::::`‐-、::::\   /,ニニ、\            それとも最近は Inv. Math. の方が上かしら?
   |      |::::::::::::::::::|` -、:::::::,ヘ ̄|'、  ヒニ二、 \
.   |      /::::::::::::::::::|::::::::\/:::O`、::\   | '、   \
   |      /:::::::::::::::::::/:::::::::::::::::::::::::::::'、::::\ノ  ヽ、  |
  |      |:::::/:::::::::/:::::::::::::::::::::::::::::::::::'、',::::'、  /:\__/‐、
  |      |/:::::::::::/::::::::::::::::::::::::::::::::::O::| '、::| く::::::::::::: ̄|
   |     /_..-'´ ̄`ー-、:::::::::::::::::::::::::::::::::::|/:/`‐'::\;;;;;;;_|
   |    |/::::::::::::::::::::::\:::::::::::::::::::::::::::::|::/::::|::::/:::::::::::/
    |   /:::::::::::::::::::::::::::::::::|:::::::::::::::::::::O::|::|::::::|:::::::::::::::/
728721:2013/02/06(水) 01:32:33.00
>>724
その1は何なんでしょうか?
729132人目の素数さん:2013/02/06(水) 01:35:25.44
>>728
φが何だか分かってないね。
730721:2013/02/06(水) 02:39:39.51
>>729
φは円柱をxy平面に投影した時の(x,y)座標ですよね
731132人目の素数さん:2013/02/06(水) 03:07:45.66
なわけあるかい
732132人目の素数さん:2013/02/06(水) 03:38:13.86
(1+cosφ,sinφ)とφは違う
733721:2013/02/06(水) 04:15:02.44
>>731
ぶw

円柱をxy平面に投影した時の(x,y)を極座標表示したときの動径ですよね。
で、なぜ1を掛けたら答えになることにつながるのかが分からない。
734132人目の素数さん:2013/02/06(水) 04:32:53.51
(1)グラフ用紙を用意します
(2)左の端をx=0、右の端をx=2π、下の端をy=0とします
(3)y=|2sin(φ/2)|より上の部分を切り捨てます
(4)グラフ用紙の右端と左端を合わせて筒状にします。これがFです
(5)(3)直後の状態に戻して、この面積を積分するには?

難しければy=|2sin(φ/2)|の代わりにy=1でやってみましょう
735132人目の素数さん:2013/02/06(水) 06:42:24.20
          __ノ)-'´ ̄ ̄`ー- 、_
        , '´  _. -‐'''"二ニニ=-`ヽ、
      /   /:::::; -‐''"        `ーノ
     /   /:::::/           \
     /    /::::::/          | | |  |
     |   |:::::/ /     |  | | | |  |
      |   |::/ / / |  | ||  | | ,ハ .| ,ハ|
      |   |/ / / /| ,ハノ| /|ノレ,ニ|ル' 
     |   |  | / / レ',二、レ′ ,ィイ|゙/   私は只の数ヲタなんかとは付き合わないわ。
.     |   \ ∠イ  ,イイ|    ,`-' |      頭が良くて数学が出来てかっこいい人。それが必要条件よ。
     |     l^,人|  ` `-'     ゝ  |        さらに Ann.of Math に論文書けば十分条件にもなるわよ。
      |      ` -'\       ー'  人          一番嫌いなのは論文数を増やすためにくだらない論文を書いて
    |        /(l     __/  ヽ、           良い論文の出版を遅らせるお馬鹿な人。
     |       (:::::`‐-、__  |::::`、     ヒニニヽ、         あなたの論文が Ann of Math に accept される確率は?
    |      / `‐-、::::::::::`‐-、::::\   /,ニニ、\            それとも最近は Inv. Math. の方が上かしら?
   |      |::::::::::::::::::|` -、:::::::,ヘ ̄|'、  ヒニ二、 \
.   |      /::::::::::::::::::|::::::::\/:::O`、::\   | '、   \
   |      /:::::::::::::::::::/:::::::::::::::::::::::::::::'、::::\ノ  ヽ、  |
  |      |:::::/:::::::::/:::::::::::::::::::::::::::::::::::'、',::::'、  /:\__/‐、
  |      |/:::::::::::/::::::::::::::::::::::::::::::::::O::| '、::| く::::::::::::: ̄|
   |     /_..-'´ ̄`ー-、:::::::::::::::::::::::::::::::::::|/:/`‐'::\;;;;;;;_|
   |    |/::::::::::::::::::::::\:::::::::::::::::::::::::::::|::/::::|::::/:::::::::::/
    |   /:::::::::::::::::::::::::::::::::|:::::::::::::::::::::O::|::|::::::|:::::::::::::::/
736132人目の素数さん:2013/02/06(水) 13:06:14.52
質問です

3人でサイコロを投げる。1〜2の目が出ると1点、その他の目が出ると0点とする。1人が2回サイコロを投げるとき、1人だけが2点となる確率を求めよ。

解き方の方針を教えて下さい。
737132人目の素数さん:2013/02/06(水) 13:13:01.26
>>736
全ての場合を書きだしたってそんな労力じゃないだろうに
取り掛かってないことが透けて見えてしょうがない
738132人目の素数さん:2013/02/06(水) 13:14:31.44
          __ノ)-'´ ̄ ̄`ー- 、_
        , '´  _. -‐'''"二ニニ=-`ヽ、
      /   /:::::; -‐''"        `ーノ
     /   /:::::/           \
     /    /::::::/          | | |  |
     |   |:::::/ /     |  | | | |  |
      |   |::/ / / |  | ||  | | ,ハ .| ,ハ|
      |   |/ / / /| ,ハノ| /|ノレ,ニ|ル' 
     |   |  | / / レ',二、レ′ ,ィイ|゙/   私は只の数ヲタなんかとは付き合わないわ。
.     |   \ ∠イ  ,イイ|    ,`-' |      頭が良くて数学が出来てかっこいい人。それが必要条件よ。
     |     l^,人|  ` `-'     ゝ  |        さらに Ann.of Math に論文書けば十分条件にもなるわよ。
      |      ` -'\       ー'  人          一番嫌いなのは論文数を増やすためにくだらない論文を書いて
    |        /(l     __/  ヽ、           良い論文の出版を遅らせるお馬鹿な人。
     |       (:::::`‐-、__  |::::`、     ヒニニヽ、         あなたの論文が Ann of Math に accept される確率は?
    |      / `‐-、::::::::::`‐-、::::\   /,ニニ、\            それとも最近は Inv. Math. の方が上かしら?
   |      |::::::::::::::::::|` -、:::::::,ヘ ̄|'、  ヒニ二、 \
.   |      /::::::::::::::::::|::::::::\/:::O`、::\   | '、   \
   |      /:::::::::::::::::::/:::::::::::::::::::::::::::::'、::::\ノ  ヽ、  |
  |      |:::::/:::::::::/:::::::::::::::::::::::::::::::::::'、',::::'、  /:\__/‐、
  |      |/:::::::::::/::::::::::::::::::::::::::::::::::O::| '、::| く::::::::::::: ̄|
   |     /_..-'´ ̄`ー-、:::::::::::::::::::::::::::::::::::|/:/`‐'::\;;;;;;;_|
   |    |/::::::::::::::::::::::\:::::::::::::::::::::::::::::|::/::::|::::/:::::::::::/
    |   /:::::::::::::::::::::::::::::::::|:::::::::::::::::::::O::|::|::::::|:::::::::::::::/
739132人目の素数さん:2013/02/06(水) 13:16:01.26
>>737
いや、全部は大変だろう

まず1〜2の目が出る確率が1/3
それが2回出れば2点だから1/9
2点にならない確率は8/9

(8/9)*(8/9)*(1/9)+(8/9)*(1/9)*(8/9)+(1/9)*(8/9)*(8/9)
=64/243
740132人目の素数さん:2013/02/06(水) 13:16:11.95
1人だけ2点出る確率 = 誰か1人は2点出る確率 - 2人は2点出る確率 - 3人とも2点出る確率
741132人目の素数さん:2013/02/06(水) 13:19:03.54
>>740
>2人は2点出る確率
これって3人ともを含んでるんじゃないの
742740:2013/02/06(水) 13:20:58.01
ごめん適当にメモってたら途中で送信しちゃった
743狢 ◆yEy4lYsULH68 :2013/02/06(水) 13:29:05.58
性犯罪者の撲滅という社会の浄化はオマエの仕事や。そやしシカッリせえやナ。

ケケケ狢

>356 名前:132人目の素数さん :2013/02/02(土) 13:39:44.60
> >>355
> そう性犯罪者が何を言っても無駄。
> 性犯罪者を叩くのは名誉毀損でも誹謗中傷でもないしね。
> 社会を浄化するための行為でしかない。
>
> そもそも名誉毀損や誹謗中傷なんてこの板ではほとんどない。
>
744132人目の素数さん:2013/02/06(水) 13:32:31.48
>>739
解決しました。ありがとうございました!

>>737
>>740
>>741
ありがとうございました!
745狢 ◆yEy4lYsULH68 :2013/02/06(水) 13:36:58.87
性犯罪者の撲滅という社会の浄化はオマエの仕事や。そやしシカッリせえやナ。

ケケケ狢

>356 名前:132人目の素数さん :2013/02/02(土) 13:39:44.60
> >>355
> そう性犯罪者が何を言っても無駄。
> 性犯罪者を叩くのは名誉毀損でも誹謗中傷でもないしね。
> 社会を浄化するための行為でしかない。
>
> そもそも名誉毀損や誹謗中傷なんてこの板ではほとんどない。
>
746132人目の素数さん:2013/02/06(水) 13:38:20.20
このケケケはなんなんだ
747132人目の素数さん:2013/02/06(水) 13:42:39.80
みけねこの派生でね?
748狢 ◆yEy4lYsULH68 :2013/02/06(水) 13:55:30.26
性犯罪者の撲滅という社会の浄化はオマエの仕事や。そやしシカッリせえやナ。

ケケケ狢

>356 名前:132人目の素数さん :2013/02/02(土) 13:39:44.60
> >>355
> そう性犯罪者が何を言っても無駄。
> 性犯罪者を叩くのは名誉毀損でも誹謗中傷でもないしね。
> 社会を浄化するための行為でしかない。
>
> そもそも名誉毀損や誹謗中傷なんてこの板ではほとんどない。
>
749132人目の素数さん:2013/02/06(水) 14:02:58.18
>>746
日本人なら wwwと書くところをチョンは kkkと書くそうだから
そのたぐいだろ
750狢 ◆yEy4lYsULH68 :2013/02/06(水) 14:24:22.11
なるほどナ。

ケケケ狢
751132人目の素数さん:2013/02/06(水) 15:37:10.68
          __ノ)-'´ ̄ ̄`ー- 、_
        , '´  _. -‐'''"二ニニ=-`ヽ、
      /   /:::::; -‐''"        `ーノ
     /   /:::::/           \
     /    /::::::/          | | |  |
     |   |:::::/ /     |  | | | |  |
      |   |::/ / / |  | ||  | | ,ハ .| ,ハ|
      |   |/ / / /| ,ハノ| /|ノレ,ニ|ル' 
     |   |  | / / レ',二、レ′ ,ィイ|゙/   私は只の数ヲタなんかとは付き合わないわ。
.     |   \ ∠イ  ,イイ|    ,`-' |      頭が良くて数学が出来てかっこいい人。それが必要条件よ。
     |     l^,人|  ` `-'     ゝ  |        さらに Ann.of Math に論文書けば十分条件にもなるわよ。
      |      ` -'\       ー'  人          一番嫌いなのは論文数を増やすためにくだらない論文を書いて
    |        /(l     __/  ヽ、           良い論文の出版を遅らせるお馬鹿な人。
     |       (:::::`‐-、__  |::::`、     ヒニニヽ、         あなたの論文が Ann of Math に accept される確率は?
    |      / `‐-、::::::::::`‐-、::::\   /,ニニ、\            それとも最近は Inv. Math. の方が上かしら?
   |      |::::::::::::::::::|` -、:::::::,ヘ ̄|'、  ヒニ二、 \
.   |      /::::::::::::::::::|::::::::\/:::O`、::\   | '、   \
   |      /:::::::::::::::::::/:::::::::::::::::::::::::::::'、::::\ノ  ヽ、  |
  |      |:::::/:::::::::/:::::::::::::::::::::::::::::::::::'、',::::'、  /:\__/‐、
  |      |/:::::::::::/::::::::::::::::::::::::::::::::::O::| '、::| く::::::::::::: ̄|
   |     /_..-'´ ̄`ー-、:::::::::::::::::::::::::::::::::::|/:/`‐'::\;;;;;;;_|
   |    |/::::::::::::::::::::::\:::::::::::::::::::::::::::::|::/::::|::::/:::::::::::/
    |   /:::::::::::::::::::::::::::::::::|:::::::::::::::::::::O::|::|::::::|:::::::::::::::/
752132人目の素数さん:2013/02/06(水) 17:36:25.49
>>702
>>703
686です。レスありがとうございます。
論理展開がまったく理解できていませんね・・・
レスを参考にもう少し考えてみます・・・
753132人目の素数さん:2013/02/06(水) 17:47:36.04
このスレでいいのか分かりませんが質問します。

ABCDEFGHの8種類の項目一つ一つに7種類の記号があります。
重複ありでこの組み合わせを計算すると、Aが7通りBが7通り〜で
7の8乗で良いのですか?
よろしくおねがいします。
754狢 ◆yEy4lYsULH68 :2013/02/06(水) 17:50:42.58
性犯罪者の撲滅という社会の浄化はオマエの仕事や。そやしシカッリせえやナ。

ケケケ狢

>356 名前:132人目の素数さん :2013/02/02(土) 13:39:44.60
> >>355
> そう性犯罪者が何を言っても無駄。
> 性犯罪者を叩くのは名誉毀損でも誹謗中傷でもないしね。
> 社会を浄化するための行為でしかない。
>
> そもそも名誉毀損や誹謗中傷なんてこの板ではほとんどない。
>
755132人目の素数さん:2013/02/06(水) 18:05:46.24
>>753
それは重複順列
組合せなら重複組合せだろ
756狢 ◆yEy4lYsULH68 :2013/02/06(水) 18:08:56.26
性犯罪者の撲滅という社会の浄化はオマエの仕事や。そやしシカッリせえやナ。

ケケケ狢

>356 名前:132人目の素数さん :2013/02/02(土) 13:39:44.60
> >>355
> そう性犯罪者が何を言っても無駄。
> 性犯罪者を叩くのは名誉毀損でも誹謗中傷でもないしね。
> 社会を浄化するための行為でしかない。
>
> そもそも名誉毀損や誹謗中傷なんてこの板ではほとんどない。
>
757132人目の素数さん:2013/02/06(水) 18:59:49.16
>>755
そしたら3003で合ってますか?
758132人目の素数さん:2013/02/06(水) 19:03:35.39
n(n+2)(n^2+1)が3n+1で割りきれるための正整数nに関する必要十分条件を求めよ

という問題なんですが、3か9の倍数かと思い剰余類で分類してみたりしたのですができません
いくつか値を入れましたが3ぐらいしか成り立ちそうにありません(大きい数だとあるかも)

ヒントでもいいのでどうかご教授お願いします
759132人目の素数さん:2013/02/06(水) 19:06:27.47
>>753
いいよ
760132人目の素数さん:2013/02/06(水) 19:10:06.04
とりあえずnは奇数
761753:2013/02/06(水) 19:10:12.67
>>759
よかった。ありがとうございます。
762132人目の素数さん:2013/02/06(水) 19:15:50.28
>>760
それはわかります(;´д`)
763753:2013/02/06(水) 19:16:54.98
>>759
あれれ、重複順列でいいのですか?5764801?
重複組み合わせで3003?
このスレIDでないから、わかりづらいですすね。
764132人目の素数さん:2013/02/06(水) 19:33:01.84
>>758
n(n+2)(n^2+1)が3n+1で割りきれる<=> -50が3n+1で割りきれる
765132人目の素数さん:2013/02/06(水) 19:33:14.73
>>753
どんな場合の数を求めたいのか正確に。
766あのこうちやんは始皇帝だった:2013/02/06(水) 19:40:13.39
 テメ〜、いいかげんにしねえと、ブッ殺すぞ!

 30代の、無職の、知的障害の、女性恐怖症の、頭デッカチの虚弱児・ひ弱の、ゴミ・クズ・カス・無能・虫けらのクソガキ!

 死ね!!!!!!!!!!!!!!!!!!!!!!
767753:2013/02/06(水) 19:44:34.33
>>765
>>753を書き直します。
1枚の紙に営業、経理、総務〜8個の評価部門が記載されています。
それぞれA〜Hは1〜7までランクの評価が付いてます。
その組み合わせが何通りか知りたいです。
重複組み合わせで3003ですね?
重複順列だと意味が無いですよね。
768753:2013/02/06(水) 19:46:34.78
間違えた。訂正します。
>>765
>>753を書き直します。
1枚の紙に営業、経理、総務〜8個の評価部門が記載されています。
それぞれ営業〜8つの評価部門は、1〜7までランクの評価が付いてます。
その組み合わせが何通りか知りたいです。
重複組み合わせで3003ですね?
重複順列だと意味が無いですよね。
769132人目の素数さん:2013/02/06(水) 20:09:24.78
>>768
だ・か・ら、問題文をそのまま書けよ
そうせずにまともな答えが帰ってくると思うな

>>753にも>>768にも「組み合わせ」なんて書いてあるので、
全く意図不明なんだよ
「組み合わせ」という言葉はたぶん君だけにしか通じてない、君の解釈でそ?
770753:2013/02/06(水) 20:18:32.95
>>769
問題文なんて無いです。
なんでわからないの?
重複順列だと意味がないでしょ。
771753:2013/02/06(水) 20:21:35.69
>>769
重複組み合わせなら3003でしょ?
772132人目の素数さん:2013/02/06(水) 20:24:29.24
>>770
もう自分の中で答えが決まってるんなら、
それでいいんじゃね
終了
773753:2013/02/06(水) 20:25:46.32
>>769
じゃ、これなら分かります?
1枚のおみくじで学業、相場、転居等、8項目記載してある。
記載場所は固定してある。
各項目に凄く良いから凄く悪いまで7ランクある。
このおみくじは何パターンあるか。
774132人目の素数さん:2013/02/06(水) 20:29:13.69
>>773
重複順列でも重複組み合わせでもなく、7^8=5764801通り
775132人目の素数さん:2013/02/06(水) 20:31:44.94
何様だこのアホは
自分が重複順列とか言うから訳分からなくなったんだろドアホ
学力以前の問題やわ
776132人目の素数さん:2013/02/06(水) 20:36:07.48
          __ノ)-'´ ̄ ̄`ー- 、_
        , '´  _. -‐'''"二ニニ=-`ヽ、
      /   /:::::; -‐''"        `ーノ
     /   /:::::/           \
     /    /::::::/          | | |  |
     |   |:::::/ /     |  | | | |  |
      |   |::/ / / |  | ||  | | ,ハ .| ,ハ|
      |   |/ / / /| ,ハノ| /|ノレ,ニ|ル' 
     |   |  | / / レ',二、レ′ ,ィイ|゙/   
.     |   \ ∠イ  ,イイ|    ,`-' |      
     |     l^,人|  ` `-'     ゝ  |        このスレには馬と鹿と豚さんしかいないのね。
      |      ` -'\       ー'  人            
    |        /(l     __/  ヽ、          
     |       (:::::`‐-、__  |::::`、     ヒニニヽ、         
    |      / `‐-、::::::::::`‐-、::::\   /,ニニ、\            
   |      |::::::::::::::::::|` -、:::::::,ヘ ̄|'、  ヒニ二、 \
.   |      /::::::::::::::::::|::::::::\/:::O`、::\   | '、   \
   |      /:::::::::::::::::::/:::::::::::::::::::::::::::::'、::::\ノ  ヽ、  |
  |      |:::::/:::::::::/:::::::::::::::::::::::::::::::::::'、',::::'、  /:\__/‐、
  |      |/:::::::::::/::::::::::::::::::::::::::::::::::O::| '、::| く::::::::::::: ̄|
   |     /_..-'´ ̄`ー-、:::::::::::::::::::::::::::::::::::|/:/`‐'::\;;;;;;;_|
   |    |/::::::::::::::::::::::\:::::::::::::::::::::::::::::|::/::::|::::/:::::::::::/
    |   /:::::::::::::::::::::::::::::::::|:::::::::::::::::::::O::|::|::::::|:::::::::::::::/
777753:2013/02/06(水) 20:38:35.58
>>774
>>755の人が合っているんじゃないの?
>>755の回答した人が間違っているの?

5764801は重複順列ですよね。
場所が固定してあるから、重複組み合わせじゃないんですか?
778132人目の素数さん:2013/02/06(水) 20:52:02.20
          __ノ)-'´ ̄ ̄`ー- 、_
        , '´  _. -‐'''"二ニニ=-`ヽ、
      /   /:::::; -‐''"        `ーノ
     /   /:::::/           \
     /    /::::::/          | | |  |
     |   |:::::/ /     |  | | | |  |
      |   |::/ / / |  | ||  | | ,ハ .| ,ハ|
      |   |/ / / /| ,ハノ| /|ノレ,ニ|ル' 
     |   |  | / / レ',二、レ′ ,ィイ|゙/   私は只の数ヲタなんかとは付き合わないわ。
.     |   \ ∠イ  ,イイ|    ,`-' |      頭が良くて数学が出来てかっこいい人。それが必要条件よ。
     |     l^,人|  ` `-'     ゝ  |        さらに Ann.of Math に論文書けば十分条件にもなるわよ。
      |      ` -'\       ー'  人          一番嫌いなのは論文数を増やすためにくだらない論文を書いて
    |        /(l     __/  ヽ、           良い論文の出版を遅らせるお馬鹿な人。
     |       (:::::`‐-、__  |::::`、     ヒニニヽ、         あなたの論文が Ann of Math に accept される確率は?
    |      / `‐-、::::::::::`‐-、::::\   /,ニニ、\            それとも最近は Inv. Math. の方が上かしら?
   |      |::::::::::::::::::|` -、:::::::,ヘ ̄|'、  ヒニ二、 \
.   |      /::::::::::::::::::|::::::::\/:::O`、::\   | '、   \
   |      /:::::::::::::::::::/:::::::::::::::::::::::::::::'、::::\ノ  ヽ、  |
  |      |:::::/:::::::::/:::::::::::::::::::::::::::::::::::'、',::::'、  /:\__/‐、
  |      |/:::::::::::/::::::::::::::::::::::::::::::::::O::| '、::| く::::::::::::: ̄|
   |     /_..-'´ ̄`ー-、:::::::::::::::::::::::::::::::::::|/:/`‐'::\;;;;;;;_|
   |    |/::::::::::::::::::::::\:::::::::::::::::::::::::::::|::/::::|::::/:::::::::::/
    |   /:::::::::::::::::::::::::::::::::|:::::::::::::::::::::O::|::|::::::|:::::::::::::::/
779132人目の素数さん:2013/02/06(水) 21:00:10.54
>>764
どうやってその同値関係を導いたのでしょうか
780132人目の素数さん:2013/02/06(水) 21:29:56.36
>>779
-50の-をつけたままにしたのがヒントのヒントのつもりだったんだけどな

左側
<=> (3^4)*n(n+2)(n^2+1)=3n(3n+6)((3n)^2+9) が3n+1で割りきれる
<=> 3n(3n+6)((3n)^2+9)が3n+1で割りきれる
多項式 F(x):=x(x+6)(x^2+9) を考えると F(x)=(x+1)*(とある整数係数多項式)-50
x=3n を代入して右側
781132人目の素数さん:2013/02/06(水) 21:33:10.93
あー、ちょっとミスったが適当に訂正してくれ
782132人目の素数さん:2013/02/06(水) 21:38:58.90
>>780
解りましたありがとうございます!!
こんな発想出てきませんでした!
783721:2013/02/06(水) 22:59:53.69
大体理解したのですが、
半径がrだとrを掛ける理由(1なら1)
が分からない。
円柱軸にそって角度積分ではなぜだめなんでしょう
784狢 ◆yEy4lYsULH68 :2013/02/06(水) 23:05:49.13
性犯罪者の撲滅という社会の浄化はオマエの仕事や。そやしシカッリせえやナ。

ケケケ狢

>356 名前:132人目の素数さん :2013/02/02(土) 13:39:44.60
> >>355
> そう性犯罪者が何を言っても無駄。
> 性犯罪者を叩くのは名誉毀損でも誹謗中傷でもないしね。
> 社会を浄化するための行為でしかない。
>
> そもそも名誉毀損や誹謗中傷なんてこの板ではほとんどない。
>
785132人目の素数さん:2013/02/06(水) 23:07:03.40
>>783
>交点Qの座標がQ(1+cosφ,sinφ,2sin(φ/2)) までは分かる
どうしてこうなるのか、またφはどんな変数か、を説明してみて
786132人目の素数さん:2013/02/06(水) 23:08:18.90
>>783
中心角 dθ ,半径 r の円弧の長さは r dθ
これに高さをかけて得られる微小長方形を積分して面積を求めてる
787132人目の素数さん:2013/02/06(水) 23:23:25.13
2^n の最高位の数字が1になるn,2になるn,3になるn,・・・,9になるn の個数はいずれも無限個あるでしょうか。
788132人目の素数さん:2013/02/06(水) 23:49:01.55
【尖閣】 防衛省「レーダー照射、以前からあった!民主党がビビって発表しなかっただけw」
http://hayabusa3.2ch.net/test/read.cgi/news/1360148473/

2013年2月6日 水曜アンカー B
http://www.youtube.com/watch?v=ZbqTsl6aoh8
789132人目の素数さん:2013/02/07(木) 00:12:24.06
          __ノ)-'´ ̄ ̄`ー- 、_
        , '´  _. -‐'''"二ニニ=-`ヽ、
      /   /:::::; -‐''"        `ーノ
     /   /:::::/           \
     /    /::::::/          | | |  |
     |   |:::::/ /     |  | | | |  |
      |   |::/ / / |  | ||  | | ,ハ .| ,ハ|
      |   |/ / / /| ,ハノ| /|ノレ,ニ|ル' 
     |   |  | / / レ',二、レ′ ,ィイ|゙/   
.     |   \ ∠イ  ,イイ|    ,`-' |      
     |     l^,人|  ` `-'     ゝ  |        このスレは馬と鹿と豚さんばかりね。
      |      ` -'\       ー'  人            
    |        /(l     __/  ヽ、          
     |       (:::::`‐-、__  |::::`、     ヒニニヽ、         
    |      / `‐-、::::::::::`‐-、::::\   /,ニニ、\            
   |      |::::::::::::::::::|` -、:::::::,ヘ ̄|'、  ヒニ二、 \
.   |      /::::::::::::::::::|::::::::\/:::O`、::\   | '、   \
   |      /:::::::::::::::::::/:::::::::::::::::::::::::::::'、::::\ノ  ヽ、  |
  |      |:::::/:::::::::/:::::::::::::::::::::::::::::::::::'、',::::'、  /:\__/‐、
  |      |/:::::::::::/::::::::::::::::::::::::::::::::::O::| '、::| く::::::::::::: ̄|
   |     /_..-'´ ̄`ー-、:::::::::::::::::::::::::::::::::::|/:/`‐'::\;;;;;;;_|
   |    |/::::::::::::::::::::::\:::::::::::::::::::::::::::::|::/::::|::::/:::::::::::/
    |   /:::::::::::::::::::::::::::::::::|:::::::::::::::::::::O::|::|::::::|:::::::::::::::/
790132人目の素数さん:2013/02/07(木) 02:18:24.47
体積積分の件ですが分かりましたありがとうございます

周期が〜であることを示せ といわれたら、
最小周期であることは示さなくても良いのですよね?

∫[t→2t]f(x)dxをtで微分すると、f(2t)×2-f(t)であってますでしょうか?
791132人目の素数さん:2013/02/07(木) 02:33:10.49
>>787
それ前気になって100万ぐらい走らせたんだけど
1〜9までほぼ均等に出現するよ
792132人目の素数さん:2013/02/07(木) 02:37:07.01
失礼100万より多かったわ
もっと満足するまででした
793132人目の素数さん:2013/02/07(木) 02:38:24.04
100万程度じゃあ意味ねぇんだよ!!
794132人目の素数さん:2013/02/07(木) 02:48:58.70
2^1,000,000だとしたらどうか
795132人目の素数さん:2013/02/07(木) 03:03:53.39
なわけないじゃん
796132人目の素数さん:2013/02/07(木) 03:05:20.33
小学生みたいだね
797132人目の素数さん:2013/02/07(木) 03:12:14.47
>>787
対数とって小数部分みりゃいいんだよ。無限個ある
で、自然数から適当にnをとってくると最上位桁が1になる確率は(log2)/(log10)
2は{(log3)-(log2)}/(log10)、3は{(log4)-(log3)}/(log10)…
798132人目の素数さん:2013/02/07(木) 08:09:38.24
一般化して、
任意の正整数kに対して、
2^n の頭位がkとなる(例えばk = 20なら 2^11 = '20'48 って具合 )ようなnも無限個あるといえる?
799132人目の素数さん:2013/02/07(木) 08:12:27.54
当然
適当にnをとったとき{(log 23)-(log 22)}/(log 100)の確率でそうなる
800132人目の素数さん:2013/02/07(木) 08:14:21.76
ごめん数値見間違えた
{(log 21)-(log 20)}/(log 100)だわな…
801132人目の素数さん:2013/02/07(木) 08:17:40.52
>>777
場所が固定してあるから順列なのだが。
例えば、aaabbbbとabababaを同じものとみなすのが組み合わせ。
802132人目の素数さん:2013/02/07(木) 08:21:25.47
「Clapte」
http://www.vector.co.jp/magazine/softnews/130130/n1301301.html
必要なときにすぐ使える、操作性の高い、常駐型多機能 数式電卓ソフト
フリーソフト
803132人目の素数さん:2013/02/07(木) 10:28:00.01
質問です
iim_[x→2]{√(3+x)-√(7-x)}/{√(1+x)(3-x)-√(1-x)(1-2x)}
これがどうしても解けません。
答えは-2√15/35と分かっているのですが、x-2=tとおいて展開して……などいろいろやってもどうしても分母が0になってしまいます
よろしくお願いします
804132人目の素数さん:2013/02/07(木) 10:37:15.63
>>803
分母も分子も有理化する
805803:2013/02/07(木) 11:02:52.49
>>804
解決しました! ありがとうございました
806132人目の素数さん:2013/02/07(木) 13:23:16.20
          __ノ)-'´ ̄ ̄`ー- 、_
        , '´  _. -‐'''"二ニニ=-`ヽ、
      /   /:::::; -‐''"        `ーノ
     /   /:::::/           \
     /    /::::::/          | | |  |
     |   |:::::/ /     |  | | | |  |
      |   |::/ / / |  | ||  | | ,ハ .| ,ハ|
      |   |/ / / /| ,ハノ| /|ノレ,ニ|ル' 
     |   |  | / / レ',二、レ′ ,ィイ|゙/   私は只の数ヲタなんかとは付き合わないわ。
.     |   \ ∠イ  ,イイ|    ,`-' |      頭が良くて数学が出来てかっこいい人。それが必要条件よ。
     |     l^,人|  ` `-'     ゝ  |        さらに Ann.of Math に論文書けば十分条件にもなるわよ。
      |      ` -'\       ー'  人          一番嫌いなのは論文数を増やすためにくだらない論文を書いて
    |        /(l     __/  ヽ、           良い論文の出版を遅らせるお馬鹿な人。
     |       (:::::`‐-、__  |::::`、     ヒニニヽ、         あなたの論文が Ann of Math に accept される確率は?
    |      / `‐-、::::::::::`‐-、::::\   /,ニニ、\            それとも最近は Inv. Math. の方が上かしら?
   |      |::::::::::::::::::|` -、:::::::,ヘ ̄|'、  ヒニ二、 \
.   |      /::::::::::::::::::|::::::::\/:::O`、::\   | '、   \
   |      /:::::::::::::::::::/:::::::::::::::::::::::::::::'、::::\ノ  ヽ、  |
  |      |:::::/:::::::::/:::::::::::::::::::::::::::::::::::'、',::::'、  /:\__/‐、
  |      |/:::::::::::/::::::::::::::::::::::::::::::::::O::| '、::| く::::::::::::: ̄|
   |     /_..-'´ ̄`ー-、:::::::::::::::::::::::::::::::::::|/:/`‐'::\;;;;;;;_|
   |    |/::::::::::::::::::::::\:::::::::::::::::::::::::::::|::/::::|::::/:::::::::::/
    |   /:::::::::::::::::::::::::::::::::|:::::::::::::::::::::O::|::|::::::|:::::::::::::::/
807132人目の素数さん:2013/02/07(木) 14:07:02.85
周期が〜であることを示せ といわれたら、
最小周期であることは示さなくても良いのですよね?

∫[t→2t]f(x)dxをtで微分すると、f(2t)×2-f(t)であってますでしょうか?
808狢 ◆yEy4lYsULH68 :2013/02/07(木) 14:33:52.25
性犯罪者の撲滅という社会の浄化はオマエの仕事や。そやしシカッリせえやナ。

ケケケ狢

>356 名前:132人目の素数さん :2013/02/02(土) 13:39:44.60
> >>355
> そう性犯罪者が何を言っても無駄。
> 性犯罪者を叩くのは名誉毀損でも誹謗中傷でもないしね。
> 社会を浄化するための行為でしかない。
>
> そもそも名誉毀損や誹謗中傷なんてこの板ではほとんどない。
>
809132人目の素数さん:2013/02/07(木) 15:06:57.89
>>807 合ってるよー
810132人目の素数さん:2013/02/07(木) 15:09:05.53
1からn+1までの自然数を並べた順列があるとき、
 ・その初項から第n項までを 小→大 の順に並べ替える
 ・次に、第2項から第(n+1)項までを 小→大 の順に並べ替える
と言う操作を行います。

例えば n=6 の例で 2,5,1,6,4,3 → [2,5,1,6,4],3 → [1,2,4,5,6],3 → 1,[2,4,5,6,3] → 1,[2,3,4,5,6] → 1,2,3,4,5,6
([ ] の中を小→大 の順に並べ替えている) 
というふうに、です。

このとき、どんな順列でも必ず 1,2,3,・・・,n,n+1 になるといえますか。当然のような気もするのですが。
811132人目の素数さん:2013/02/07(木) 15:21:15.31
反例 末項が1のとき
812810:2013/02/07(木) 15:39:34.93
>>811
あああっっそうだったあああ。ありがとうございます。


ということは
 ・その初項から第n項までを 小→大 の順に並べ替える
 ・次に、第2項から第(n+1)項までを 小→大 の順に並べ替える
 ・さらに、初項から第n項までを 小→大 の順に並べ替える
とすればOKでしょうか?
813132人目の素数さん:2013/02/07(木) 15:49:33.35
はい
814132人目の素数さん:2013/02/07(木) 15:55:32.00
          __ノ)-'´ ̄ ̄`ー- 、_
        , '´  _. -‐'''"二ニニ=-`ヽ、
      /   /:::::; -‐''"        `ーノ
     /   /:::::/           \
     /    /::::::/          | | |  |
     |   |:::::/ /     |  | | | |  |
      |   |::/ / / |  | ||  | | ,ハ .| ,ハ|
      |   |/ / / /| ,ハノ| /|ノレ,ニ|ル' 
     |   |  | / / レ',二、レ′ ,ィイ|゙/   
.     |   \ ∠イ  ,イイ|    ,`-' |      
     |     l^,人|  ` `-'     ゝ  |        このスレは馬と鹿と豚さんばかりね。
      |      ` -'\       ー'  人            
    |        /(l     __/  ヽ、          
     |       (:::::`‐-、__  |::::`、     ヒニニヽ、         
    |      / `‐-、::::::::::`‐-、::::\   /,ニニ、\            
   |      |::::::::::::::::::|` -、:::::::,ヘ ̄|'、  ヒニ二、 \
.   |      /::::::::::::::::::|::::::::\/:::O`、::\   | '、   \
   |      /:::::::::::::::::::/:::::::::::::::::::::::::::::'、::::\ノ  ヽ、  |
  |      |:::::/:::::::::/:::::::::::::::::::::::::::::::::::'、',::::'、  /:\__/‐、
  |      |/:::::::::::/::::::::::::::::::::::::::::::::::O::| '、::| く::::::::::::: ̄|
   |     /_..-'´ ̄`ー-、:::::::::::::::::::::::::::::::::::|/:/`‐'::\;;;;;;;_|
   |    |/::::::::::::::::::::::\:::::::::::::::::::::::::::::|::/::::|::::/:::::::::::/
    |   /:::::::::::::::::::::::::::::::::|:::::::::::::::::::::O::|::|::::::|:::::::::::::::/
815132人目の素数さん:2013/02/07(木) 16:13:57.14
lim[x→0] (e^x-e^-x)/x
=lim[x→0] (e^(2x)-1)/e^x*x
=lim[x→0] (2/e^x)*(e^2x-1)/2x
=2
これってあってますか?
816132人目の素数さん:2013/02/07(木) 16:24:53.19
ぶっさいくだな
微分の定義使うなら
f(x)=e^x-e^-x
って見ればいいじゃん無駄な変換してんなよ
817753:2013/02/07(木) 17:04:09.61
>>801
どうもありがと。
818132人目の素数さん:2013/02/07(木) 18:12:11.47
>>816
気づきませんでした
ありがとう
819狢 ◆yEy4lYsULH68 :2013/02/07(木) 18:27:02.21
性犯罪者の撲滅という社会の浄化はオマエの仕事や。そやしシカッリせえやナ。

ケケケ狢

>356 名前:132人目の素数さん :2013/02/02(土) 13:39:44.60
> >>355
> そう性犯罪者が何を言っても無駄。
> 性犯罪者を叩くのは名誉毀損でも誹謗中傷でもないしね。
> 社会を浄化するための行為でしかない。
>
> そもそも名誉毀損や誹謗中傷なんてこの板ではほとんどない。
>
820132人目の素数さん:2013/02/07(木) 18:28:25.88
          __ノ)-'´ ̄ ̄`ー- 、_
        , '´  _. -‐'''"二ニニ=-`ヽ、
      /   /:::::; -‐''"        `ーノ
     /   /:::::/           \
     /    /::::::/          | | |  |
     |   |:::::/ /     |  | | | |  |
      |   |::/ / / |  | ||  | | ,ハ .| ,ハ|
      |   |/ / / /| ,ハノ| /|ノレ,ニ|ル' 
     |   |  | / / レ',二、レ′ ,ィイ|゙/   
.     |   \ ∠イ  ,イイ|    ,`-' |      
     |     l^,人|  ` `-'     ゝ  |        このスレは馬と鹿と豚さんばかりね。
      |      ` -'\       ー'  人            
    |        /(l     __/  ヽ、          
     |       (:::::`‐-、__  |::::`、     ヒニニヽ、         
    |      / `‐-、::::::::::`‐-、::::\   /,ニニ、\            
   |      |::::::::::::::::::|` -、:::::::,ヘ ̄|'、  ヒニ二、 \
.   |      /::::::::::::::::::|::::::::\/:::O`、::\   | '、   \
   |      /:::::::::::::::::::/:::::::::::::::::::::::::::::'、::::\ノ  ヽ、  |
  |      |:::::/:::::::::/:::::::::::::::::::::::::::::::::::'、',::::'、  /:\__/‐、
  |      |/:::::::::::/::::::::::::::::::::::::::::::::::O::| '、::| く::::::::::::: ̄|
   |     /_..-'´ ̄`ー-、:::::::::::::::::::::::::::::::::::|/:/`‐'::\;;;;;;;_|
   |    |/::::::::::::::::::::::\:::::::::::::::::::::::::::::|::/::::|::::/:::::::::::/
    |   /:::::::::::::::::::::::::::::::::|:::::::::::::::::::::O::|::|::::::|:::::::::::::::/
821あのこうちやんは始皇帝だった:2013/02/07(木) 19:23:39.18
>>820

テメ〜、いいかげんにしねえと、ブッ殺すぞ!

 30代の、無職の、知的障害の、女性恐怖症の、頭デッカチの虚弱児・ひ弱の、ゴミ・クズ・カス・無能・虫けらのクソガキ!

 死ね!!!!!!!!!!!!!!!!!!!!!!
822132人目の素数さん:2013/02/07(木) 19:24:14.53
          __ノ)-'´ ̄ ̄`ー- 、_
        , '´  _. -‐'''"二ニニ=-`ヽ、
      /   /:::::; -‐''"        `ーノ
     /   /:::::/           \
     /    /::::::/          | | |  |
     |   |:::::/ /     |  | | | |  |
      |   |::/ / / |  | ||  | | ,ハ .| ,ハ|
      |   |/ / / /| ,ハノ| /|ノレ,ニ|ル' 
     |   |  | / / レ',二、レ′ ,ィイ|゙/   
.     |   \ ∠イ  ,イイ|    ,`-' |      
     |     l^,人|  ` `-'     ゝ  |        このスレは馬と鹿と豚さんばかりね。
      |      ` -'\       ー'  人            
    |        /(l     __/  ヽ、          
     |       (:::::`‐-、__  |::::`、     ヒニニヽ、         
    |      / `‐-、::::::::::`‐-、::::\   /,ニニ、\            
   |      |::::::::::::::::::|` -、:::::::,ヘ ̄|'、  ヒニ二、 \
.   |      /::::::::::::::::::|::::::::\/:::O`、::\   | '、   \
   |      /:::::::::::::::::::/:::::::::::::::::::::::::::::'、::::\ノ  ヽ、  |
  |      |:::::/:::::::::/:::::::::::::::::::::::::::::::::::'、',::::'、  /:\__/‐、
  |      |/:::::::::::/::::::::::::::::::::::::::::::::::O::| '、::| く::::::::::::: ̄|
   |     /_..-'´ ̄`ー-、:::::::::::::::::::::::::::::::::::|/:/`‐'::\;;;;;;;_|
   |    |/::::::::::::::::::::::\:::::::::::::::::::::::::::::|::/::::|::::/:::::::::::/
    |   /:::::::::::::::::::::::::::::::::|:::::::::::::::::::::O::|::|::::::|:::::::::::::::/
823132人目の素数さん:2013/02/07(木) 19:37:26.52
>>812
反例: n=1 で 2,1 となってる場合
824132人目の素数さん:2013/02/07(木) 19:43:47.36
>>818
微分の定義として答えを出すなら
e^xの微分がe^xになるのを証明出来るようにしましょうね
825132人目の素数さん:2013/02/07(木) 20:02:54.65
          __ノ)-'´ ̄ ̄`ー- 、_
        , '´  _. -‐'''"二ニニ=-`ヽ、
      /   /:::::; -‐''"        `ーノ
     /   /:::::/           \
     /    /::::::/          | | |  |
     |   |:::::/ /     |  | | | |  |
      |   |::/ / / |  | ||  | | ,ハ .| ,ハ|
      |   |/ / / /| ,ハノ| /|ノレ,ニ|ル' 
     |   |  | / / レ',二、レ′ ,ィイ|゙/   
.     |   \ ∠イ  ,イイ|    ,`-' |      
     |     l^,人|  ` `-'     ゝ  |        このスレには馬と鹿と豚さんしかいないのね。
      |      ` -'\       ー'  人            
    |        /(l     __/  ヽ、          
     |       (:::::`‐-、__  |::::`、     ヒニニヽ、         
    |      / `‐-、::::::::::`‐-、::::\   /,ニニ、\            
   |      |::::::::::::::::::|` -、:::::::,ヘ ̄|'、  ヒニ二、 \
.   |      /::::::::::::::::::|::::::::\/:::O`、::\   | '、   \
   |      /:::::::::::::::::::/:::::::::::::::::::::::::::::'、::::\ノ  ヽ、  |
  |      |:::::/:::::::::/:::::::::::::::::::::::::::::::::::'、',::::'、  /:\__/‐、
  |      |/:::::::::::/::::::::::::::::::::::::::::::::::O::| '、::| く::::::::::::: ̄|
   |     /_..-'´ ̄`ー-、:::::::::::::::::::::::::::::::::::|/:/`‐'::\;;;;;;;_|
   |    |/::::::::::::::::::::::\:::::::::::::::::::::::::::::|::/::::|::::/:::::::::::/
    |   /:::::::::::::::::::::::::::::::::|:::::::::::::::::::::O::|::|::::::|:::::::::::::::/
826132人目の素数さん:2013/02/07(木) 20:03:47.64
ブサイク度
>>816 >>>>>>>>>>>> >>815
827狢 ◆yEy4lYsULH68 :2013/02/07(木) 20:04:15.78
性犯罪者の撲滅という社会の浄化はオマエの仕事や。そやしシカッリせえやナ。

ケケケ狢

>356 名前:132人目の素数さん :2013/02/02(土) 13:39:44.60
> >>355
> そう性犯罪者が何を言っても無駄。
> 性犯罪者を叩くのは名誉毀損でも誹謗中傷でもないしね。
> 社会を浄化するための行為でしかない。
>
> そもそも名誉毀損や誹謗中傷なんてこの板ではほとんどない。
>
828132人目の素数さん:2013/02/07(木) 20:06:41.84
          __ノ)-'´ ̄ ̄`ー- 、_
        , '´  _. -‐'''"二ニニ=-`ヽ、
      /   /:::::; -‐''"        `ーノ
     /   /:::::/           \
     /    /::::::/          | | |  |
     |   |:::::/ /     |  | | | |  |
      |   |::/ / / |  | ||  | | ,ハ .| ,ハ|
      |   |/ / / /| ,ハノ| /|ノレ,ニ|ル' 
     |   |  | / / レ',二、レ′ ,ィイ|゙/   
.     |   \ ∠イ  ,イイ|    ,`-' |      
     |     l^,人|  ` `-'     ゝ  |        このスレには馬と鹿と豚さんしかいないのね。
      |      ` -'\       ー'  人            
    |        /(l     __/  ヽ、          
     |       (:::::`‐-、__  |::::`、     ヒニニヽ、         
    |      / `‐-、::::::::::`‐-、::::\   /,ニニ、\            
   |      |::::::::::::::::::|` -、:::::::,ヘ ̄|'、  ヒニ二、 \
.   |      /::::::::::::::::::|::::::::\/:::O`、::\   | '、   \
   |      /:::::::::::::::::::/:::::::::::::::::::::::::::::'、::::\ノ  ヽ、  |
  |      |:::::/:::::::::/:::::::::::::::::::::::::::::::::::'、',::::'、  /:\__/‐、
  |      |/:::::::::::/::::::::::::::::::::::::::::::::::O::| '、::| く::::::::::::: ̄|
   |     /_..-'´ ̄`ー-、:::::::::::::::::::::::::::::::::::|/:/`‐'::\;;;;;;;_|
   |    |/::::::::::::::::::::::\:::::::::::::::::::::::::::::|::/::::|::::/:::::::::::/
    |   /:::::::::::::::::::::::::::::::::|:::::::::::::::::::::O::|::|::::::|:::::::::::::::/
829132人目の素数さん:2013/02/07(木) 20:17:38.21
教科書で調べたのですが見つけられなかったので質問させていただきます

8. f(x)=x^3+ax^2+bx+1がある。方程式f'(x)=0の解が、x=0、2のときa、bの値は【シ】である。

11. y=-x^3+kx^2-1上のx座標が2である点における接線が原点を通るとき、定数kの値は【タ】である。

答えは8.がa=3、b=0で、11が15/4です。わかる方解説お願い致します
830132人目の素数さん:2013/02/07(木) 20:36:15.80
微分しろよ

3x^2+2ax+b=0
3x(x-2)=3x^2-6x=0

y'=-3x^2+2kx
y-4k+9=(4k-12)(x-2)
0-4k+9=(4k-12)(0-2)
4k=15
831132人目の素数さん:2013/02/07(木) 20:49:23.36
>>816
横からで申し訳ないけど、これどういう事?
832132人目の素数さん:2013/02/07(木) 20:51:26.46
>>830
11のほうは理解できたのですが、
8の1行目から2行目への過程がわかりません
833132人目の素数さん:2013/02/07(木) 20:58:04.67
>>831
f'(x)=e^x+e^(-x) なので 求める極限値=f'(0)=2
ってことだろ
ブサイク極まりないな
834132人目の素数さん:2013/02/07(木) 21:00:07.75
>>831
f(0)=0だから微分の定義式を利用したんだろう
835132人目の素数さん:2013/02/07(木) 21:06:01.83
>>833
お前ならどうやるんだよ?言ってみろよwww
836132人目の素数さん:2013/02/07(木) 21:10:52.04
ネタでなくて、マジに自分のブサイクさに気付いていないのか?
837132人目の素数さん:2013/02/07(木) 21:14:15.17
まだよくわからん…
どういう事だ……?
838132人目の素数さん:2013/02/07(木) 21:36:12.94
>>832
そこは式変形じゃない
係数比較しろってこと
839132人目の素数さん:2013/02/07(木) 21:36:38.58
lim[x→0](f(x)-0)/(x-0)=f'(0)
840132人目の素数さん:2013/02/07(木) 21:37:56.55
あ、f(0)ね
841132人目の素数さん:2013/02/07(木) 22:13:23.02
>>838
そういうことでしたか
ありがとうございました
842132人目の素数さん:2013/02/07(木) 22:28:34.05
lim f(x)/g(x)=a ならば lim g(x)/f(x)=1/a

これっておk?
843132人目の素数さん:2013/02/07(木) 22:31:27.89
aが0でなければおk
844132人目の素数さん:2013/02/07(木) 23:32:58.28
          __ノ)-'´ ̄ ̄`ー- 、_
        , '´  _. -‐'''"二ニニ=-`ヽ、
      /   /:::::; -‐''"        `ーノ
     /   /:::::/           \
     /    /::::::/          | | |  |
     |   |:::::/ /     |  | | | |  |
      |   |::/ / / |  | ||  | | ,ハ .| ,ハ|
      |   |/ / / /| ,ハノ| /|ノレ,ニ|ル' 
     |   |  | / / レ',二、レ′ ,ィイ|゙/   
.     |   \ ∠イ  ,イイ|    ,`-' |      
     |     l^,人|  ` `-'     ゝ  |        このスレには馬と鹿と豚さんしかいないのね。
      |      ` -'\       ー'  人            
    |        /(l     __/  ヽ、          
     |       (:::::`‐-、__  |::::`、     ヒニニヽ、         
    |      / `‐-、::::::::::`‐-、::::\   /,ニニ、\            
   |      |::::::::::::::::::|` -、:::::::,ヘ ̄|'、  ヒニ二、 \
.   |      /::::::::::::::::::|::::::::\/:::O`、::\   | '、   \
   |      /:::::::::::::::::::/:::::::::::::::::::::::::::::'、::::\ノ  ヽ、  |
  |      |:::::/:::::::::/:::::::::::::::::::::::::::::::::::'、',::::'、  /:\__/‐、
  |      |/:::::::::::/::::::::::::::::::::::::::::::::::O::| '、::| く::::::::::::: ̄|
   |     /_..-'´ ̄`ー-、:::::::::::::::::::::::::::::::::::|/:/`‐'::\;;;;;;;_|
   |    |/::::::::::::::::::::::\:::::::::::::::::::::::::::::|::/::::|::::/:::::::::::/
    |   /:::::::::::::::::::::::::::::::::|:::::::::::::::::::::O::|::|::::::|:::::::::::::::/
845132人目の素数さん:2013/02/08(金) 01:08:33.94
>>835
>>816
f(x)=e^x-e^(-x), f(0)=0
>>833
f'(x)=e^x+e^(-x)
>>815>>839
lim[x→0](e^x-e^(-x))/x=lim[x→0](f(x)-f(0))/(x-0)=f'(0)
846132人目の素数さん:2013/02/08(金) 01:11:39.21
東大後期2002の3(4)の解説お願いします

大数の入試の軌跡には二進法の解答が書かれていたので、十進法の正攻法(?)でお願いします
847132人目の素数さん:2013/02/08(金) 02:39:40.87
>>846
問題文ぐらい書けや
848132人目の素数さん:2013/02/08(金) 02:59:48.17
>>846
東大ならたとえばK合塾とかY村先生とかが解答をwebで公開しているので
まずそれを見ればよかろう
849132人目の素数さん:2013/02/08(金) 07:02:40.33
東大後期2002の3(4)とやら、
検索してたどりついてもいいけど
普通はurlくらい貼るだろ
850132人目の素数さん:2013/02/08(金) 07:21:28.86
>>1も読めないやつは放置しろよ
851132人目の素数さん:2013/02/08(金) 09:01:40.24
河合の東大塾にはのってなかったので、、、

すいません

f(x)=2x(0≦x≦1/2)、2-2x(1/2<x≦1)で定義する

a_(1)は0≦a_(1)≦1でかつ自然数iとkを用いてi/(3・2^k-2)と表せられるもの以外である
(厳密にはi/(3・2^k-2)を除く作業を(3)までで行っています)

a_(n+1)=f(a_(n))とする

あるnに対してa_(n)が3/4≦a_(n)となることを示せ

が問題です
852132人目の素数さん:2013/02/08(金) 09:08:45.69
回答してくれようとする人間に問題を探させる時点で常識知らず。
853132人目の素数さん:2013/02/08(金) 09:14:23.97
(3)までとか何だよバカ
854132人目の素数さん:2013/02/08(金) 09:16:22.64
いやみなさんなら解いたことあるかな、と思いまして、、、

i/(3・2^k-2)は全部0か2/3に収束するので除くんです
855132人目の素数さん:2013/02/08(金) 09:16:59.00
数学以前にちょおおおっと頭の働きが弱いみたいだね
勉強以外でもすこーーーしは頭を働かせた方がいいよあなた
856132人目の素数さん:2013/02/08(金) 09:18:31.45
問題を貼れといわれたので貼ったんですけどなにか間違えてますか
857132人目の素数さん:2013/02/08(金) 09:21:17.20
>>856
>>855みたいなやつは絡み癖あるから無視で
858132人目の素数さん:2013/02/08(金) 09:21:46.35
だから
> 除く作業を(3)までで
この(3)てどこだよ

だからオマエには
数学以前にもっと頭を働かせろ
って言ってやってんの

マジで何も考えられない頭だから勉強以前のお話だっちゅーの
859132人目の素数さん:2013/02/08(金) 09:23:27.61
なんだただのアスペか
860132人目の素数さん:2013/02/08(金) 09:24:09.84
>>1
> ・質問者は回答者がわかるように問題を書くようにしましょう。でないと放置されることがあります。
>   (変に省略するより全文書いた方がいい、また説明なく習慣的でない記号を使わないように)

>   (変に省略するより全文書いた方がいい
>   (変に省略するより全文書いた方がいい
>   (変に省略するより全文書いた方がいい

昔は、「結局全部書くことになります」とかってなってたよなw
861132人目の素数さん:2013/02/08(金) 09:35:44.50
それは全文かかないと意味が通じなくなることがあるから全部書いた方がいいよとの注意書き、あるいは推奨ですよね?

自分が書き込みしたものを見返す限り、題意が伝わらないことはないと思います

あなたの主張は目的と手段が入れ替わっていますよ
862132人目の素数さん:2013/02/08(金) 09:39:22.62
真摯に読み返してみ。
現に伝わってないんだからw
863132人目の素数さん:2013/02/08(金) 09:44:05.87
あるnって任意のnのことか?
n=1の時成り立たないねぇ
864132人目の素数さん:2013/02/08(金) 09:47:13.23
だから、放置しろって言ったじゃん。
しかし、聞く耳持たないくせに回答求めるって矛盾してるよなw
865132人目の素数さん:2013/02/08(金) 09:49:37.72
>>863
揚げ足取ってる積もりだろうがお前はただの馬鹿だ
866132人目の素数さん:2013/02/08(金) 09:53:05.22
>>864
そりゃ、こういう状況になってもまだ答えがもらえると思ってる時点で(ry
867132人目の素数さん:2013/02/08(金) 09:57:50.74
>>865
nが存在するって事いいたいんだろ?
でも多分原文は漸化式の後にこんな書き方してねぇと思うぜ
任意とか存在について無頓着な奴が書いた感じがする文だからなぁ
868132人目の素数さん:2013/02/08(金) 11:05:46.40
俺は本物見てからスレ見たので意味が分かるが、初見じゃ分からないだろうな

URL:tp//server-test.net/math/php.php?name=tokyo&v1=1&v2=2002&v3=2&v4=3&y=2002&n=9

はっきり言って>>846は失礼だと思いんす
869132人目の素数さん:2013/02/08(金) 11:09:03.29
URLはhttpの後に:が抜けてたりするから適宜直してくれ
870132人目の素数さん:2013/02/08(金) 11:12:47.22
そんなこと以前に参照する気すら起きねえ
871132人目の素数さん:2013/02/08(金) 11:16:50.14
パイこね変換というか力学系・カオスの問題だというのはすぐ分かる
教えてやる気は全く起きない
872132人目の素数さん:2013/02/08(金) 11:28:00.00
ここで高校生の質問に答えようって人らは基本かなり優しい人達だから、
それを怒らせるってのは相当だってのを自覚した方がいいよ
こういう人が東大受験すると思うと悲しいわ
873132人目の素数さん:2013/02/08(金) 11:48:59.33
高校数学しかわからない暇人だろ
874132人目の素数さん:2013/02/08(金) 12:47:53.94
暇人であることと優しいことは両立するよ
875132人目の素数さん:2013/02/08(金) 13:34:28.57
やさしさとはそういうものだろうか?
876132人目の素数さん:2013/02/08(金) 13:43:48.49
優しさとは何かしら?
877132人目の素数さん:2013/02/08(金) 13:52:40.97
優しさとは・・・フヒヒ
878132人目の素数さん:2013/02/08(金) 13:53:00.77
高校数学しかわからない人が高校生の質問に答えられるはずがない
879132人目の素数さん:2013/02/08(金) 13:58:11.10
暇があることは確かだな
その暇を人助けに使うとこが違うだけだ
880132人目の素数さん:2013/02/08(金) 16:39:47.38
>>846
k合は消されてたんだな ケチ臭えな
昔落としたやつ
www1.axfc.net/uploader/so/2785159?key=math
881132人目の素数さん:2013/02/08(金) 16:45:58.66
ここはダウンロード板ではありません
882132人目の素数さん:2013/02/08(金) 19:11:38.81
因数分解についてです。
12x^2 -14xy -6y^2
という問題で、答えは
2(2x-3y)(3x+y)
とかかれているのですが、たすき掛けをして
(6x+2y)(2x-3y)
でもいいですか?
883あのこうちやんは始皇帝だった:2013/02/08(金) 19:17:06.76
 テメ〜ら、いいかげんにしねえと、ブッ殺すぞ!

 20代と30代の、無職の、知的障害の、女性恐怖症の、頭デッカチの虚弱児・ひ弱の、ゴミ・クズ・カス・無能・虫けらのクソガキども!

 死ね!!!!!!!!!!!!!!!!!!!!!!
884132人目の素数さん:2013/02/08(金) 19:18:27.05
>>882
まったく問題ありません。
885132人目の素数さん:2013/02/08(金) 19:25:25.26
          __ノ)-'´ ̄ ̄`ー- 、_
        , '´  _. -‐'''"二ニニ=-`ヽ、
      /   /:::::; -‐''"        `ーノ
     /   /:::::/           \
     /    /::::::/          | | |  |
     |   |:::::/ /     |  | | | |  |
      |   |::/ / / |  | ||  | | ,ハ .| ,ハ|
      |   |/ / / /| ,ハノ| /|ノレ,ニ|ル' 
     |   |  | / / レ',二、レ′ ,ィイ|゙/   
.     |   \ ∠イ  ,イイ|    ,`-' |      
     |     l^,人|  ` `-'     ゝ  |        このスレには馬と鹿と豚さんしかいないのね。
      |      ` -'\       ー'  人            
    |        /(l     __/  ヽ、          
     |       (:::::`‐-、__  |::::`、     ヒニニヽ、         
    |      / `‐-、::::::::::`‐-、::::\   /,ニニ、\            
   |      |::::::::::::::::::|` -、:::::::,ヘ ̄|'、  ヒニ二、 \
.   |      /::::::::::::::::::|::::::::\/:::O`、::\   | '、   \
   |      /:::::::::::::::::::/:::::::::::::::::::::::::::::'、::::\ノ  ヽ、  |
  |      |:::::/:::::::::/:::::::::::::::::::::::::::::::::::'、',::::'、  /:\__/‐、
  |      |/:::::::::::/::::::::::::::::::::::::::::::::::O::| '、::| く::::::::::::: ̄|
   |     /_..-'´ ̄`ー-、:::::::::::::::::::::::::::::::::::|/:/`‐'::\;;;;;;;_|
   |    |/::::::::::::::::::::::\:::::::::::::::::::::::::::::|::/::::|::::/:::::::::::/
    |   /:::::::::::::::::::::::::::::::::|:::::::::::::::::::::O::|::|::::::|:::::::::::::::/
886132人目の素数さん:2013/02/08(金) 19:35:52.04
運営乙
887132人目の素数さん:2013/02/08(金) 19:55:48.43
>>884
ありがとうございます
888132人目の素数さん:2013/02/08(金) 20:40:33.60
数列の問題なんですが、

二次方程式x^2-x-1=0相異なる解をα,βとし、a[n]=α^n+β^n(n=1,2,3・・・)
とする。
(1)a[1],a[2],a[3],a[4]を求めよ。
(2)n≧3のとき、a[n]をa[n-1]とa[n-2]を用いて表せ。
という問題で、(1)から(2)はa[n]=a[n-1]+a[n-2]となるのがすぐに分かったので、数学的帰納法を用いて証明したのですが

n=kのとき((α+1)/α)*α^k-1+((β+1)/β)*β^k-1
n=k+1のとき((α+1)/α)*α^k+((β+1)/β)*β^k
となったんですがこれで証明ってできてますか?
889132人目の素数さん:2013/02/08(金) 20:45:26.53
>>888
>数学的帰納法を用いて証明したのですが
証明を全部書いて
890132人目の素数さん:2013/02/08(金) 21:07:27.24
したいのですが、だろ
そんなことも読み取れないんか
891132人目の素数さん:2013/02/08(金) 21:11:56.17
余計なこと書いて人が離れていく
892132人目の素数さん:2013/02/08(金) 21:27:10.62
たぶん>>890>>888ではないな

それはそれとて
>n=kのとき((α+1)/α)*α^k-1+((β+1)/β)*β^k-1
>n=k+1のとき((α+1)/α)*α^k+((β+1)/β)*β^k
だとやってることの意図が不明

愚直にやればいい
a[n]=α^n+β^n a[n]の定義より
=α^n+β^n+a[n-1]-{α^(n-1)+β^(n-1)}+a[n-2]-{α^(n-2)+β^(n-2)} a[n-1],a[n-2]の定義より
=a[n-1]+a[n-2]+α^n+β^n-{α^(n-1)+β^(n-1)+α^(n-2)+β^(n-2)} 見やすく移動しただけ
=a[n-1]+a[n-2]+α^(n-2){α^2-α-1}+β^(n-2){β^2-β-1}
=a[n-1]+a[n-2]+α^(n-2)*0+β^(n-2)*0 なぜならαとβはx^2-x-1の解
=a[n-1]+a[n-2]
893132人目の素数さん:2013/02/08(金) 21:43:07.33
漸化式が分かるなら帰納法なんか使わず
一般項代入して右辺と左辺等しくなってます
でも全く問題ないけどね
894132人目の素数さん:2013/02/08(金) 21:47:39.86
>>888
これは数学的帰納法ではなく、単にa_{n}=α^n+β^n(n=1,2,・・・)なら
(1)から類推できる、等式 a_{n}=a_{n-1}+a_{n-2}  が成立することを示すだけ。
895132人目の素数さん:2013/02/08(金) 21:52:21.70
893と同じだけど、x^2-x-1=0の二解なんだから数学的帰納法持ち出さなくても
α^2=α+1よりα^n=α^(n-1)+α^(n-2)
βも同じようにして辺々足して
a[n]=a[n-1]+a[n-2]だよなぁと
896132人目の素数さん:2013/02/08(金) 21:57:45.21
>>882
2はくくり出すべきだと思うけど?
897132人目の素数さん:2013/02/08(金) 21:58:45.26
http://www.amazon.co.jp/o/ASIN/B0076ECI5I/777777778888888-22

Microsoft Office2010並行輸入品(=正規品≠海賊版)
インストールはレビュー参照
898132人目の素数さん:2013/02/08(金) 21:59:17.68
どっちでもいいだろ
まあ、たすきがけが簡単になるというメリットはあるが
899132人目の素数さん:2013/02/08(金) 22:01:55.51
 5次以上の代数方程式で累乗根で表せない実数解て、まさか超越数ではないのですよね。
900132人目の素数さん:2013/02/08(金) 22:03:23.33
超越数の定義知ってる人はそんな疑問持たないよ
901132人目の素数さん:2013/02/08(金) 22:05:35.78
>>899
代数方程式の根を超越数って・・・
902132人目の素数さん:2013/02/08(金) 22:05:49.09
確率の問題です。白玉16個赤玉4個があり、区別できる10個の箱に2個ずつ無造作に入れる.
赤玉2個入った箱が1つのみできる確率を求める問題で、以下のように考えたのですが間違っているようでどこが誤っているのかわかりません教えてください。
(解答)20個の玉の並べ方は 20!/(16!4!)=4845通り。
求める確率は赤玉が2つ入った箱が2つできるときと赤玉2つ入った箱ができないときの余事象である。
赤2つの箱が2つのとき |○○|○○|○○|○○|○○|○○|○○|○○|
|のところに赤玉2つを入れる。|を2つ選んで 9C2= 36 通り

赤玉のみの箱なしのとき 白玉16個をならべその間および一番初め、最後の玉の前後ろを4箇所選び赤玉を入れる
これは17C4=2380 通り

以上より求める確率は
1-(36/4845)-(2380/4845)=2429/4845
903132人目の素数さん:2013/02/08(金) 22:06:15.48
コーヒーフイタw
904132人目の素数さん:2013/02/08(金) 22:10:01.51
>>895 x^2-x-1=0より
   α^2=α+1
両辺にα^(n-2)を掛けてα^n=α^(n-1)+α^(n-2)
同様にβ^n=β^(n-1)+β^(n-2)
辺々足してα^n+β^n=α^(n-1)+α^(n-2)+β^(n-1)+β^(n-2)
よってa[n]=a[n-1]+a[n-2]
  こんな感じですかね?
905132人目の素数さん:2013/02/08(金) 22:10:50.61
>>901
xの1次方程式 x-π=0 の解は超越数
906132人目の素数さん:2013/02/08(金) 22:18:04.12
ハイハイ、よく出来ました。
907132人目の素数さん:2013/02/08(金) 22:47:47.38
>>902
赤玉2つ入った箱が2つ並んでるとき
908132人目の素数さん:2013/02/08(金) 22:48:21.43
>>905
座布団一枚!
909132人目の素数さん:2013/02/08(金) 23:20:12.93
>>902
赤玉のみの箱無しのほうもおかしいな。
例えば、白赤・赤白・白赤・赤白・白白・白白・白白・白白・白白・白白などを数えていない。

答って、1440/4845=96/323?
910132人目の素数さん:2013/02/08(金) 23:25:24.67
          __ノ)-'´ ̄ ̄`ー- 、_
        , '´  _. -‐'''"二ニニ=-`ヽ、
      /   /:::::; -‐''"        `ーノ
     /   /:::::/           \
     /    /::::::/          | | |  |
     |   |:::::/ /     |  | | | |  |
      |   |::/ / / |  | ||  | | ,ハ .| ,ハ|
      |   |/ / / /| ,ハノ| /|ノレ,ニ|ル' 
     |   |  | / / レ',二、レ′ ,ィイ|゙/   
.     |   \ ∠イ  ,イイ|    ,`-' |      
     |     l^,人|  ` `-'     ゝ  |        このスレには馬と鹿と豚さんしかいないのね。
      |      ` -'\       ー'  人            
    |        /(l     __/  ヽ、          
     |       (:::::`‐-、__  |::::`、     ヒニニヽ、         
    |      / `‐-、::::::::::`‐-、::::\   /,ニニ、\            
   |      |::::::::::::::::::|` -、:::::::,ヘ ̄|'、  ヒニ二、 \
.   |      /::::::::::::::::::|::::::::\/:::O`、::\   | '、   \
   |      /:::::::::::::::::::/:::::::::::::::::::::::::::::'、::::\ノ  ヽ、  |
  |      |:::::/:::::::::/:::::::::::::::::::::::::::::::::::'、',::::'、  /:\__/‐、
  |      |/:::::::::::/::::::::::::::::::::::::::::::::::O::| '、::| く::::::::::::: ̄|
   |     /_..-'´ ̄`ー-、:::::::::::::::::::::::::::::::::::|/:/`‐'::\;;;;;;;_|
   |    |/::::::::::::::::::::::\:::::::::::::::::::::::::::::|::/::::|::::/:::::::::::/
    |   /:::::::::::::::::::::::::::::::::|:::::::::::::::::::::O::|::|::::::|:::::::::::::::/
911 忍法帖【Lv=37,xxxPT】(1+0:8) :2013/02/08(金) 23:31:17.07
理科大の問題です
お願いします
http://www.dotup.org/uploda/www.dotup.org3924539.jpg
912132人目の素数さん:2013/02/08(金) 23:33:25.03
>>1
> ・質問者は何が分からないのか、どこまで考えたのかを明記しましょう。それがない場合、放置されることがあります。
913 忍法帖【Lv=37,xxxPT】(2+0:8) :2013/02/08(金) 23:36:19.10
>>912
すいません
ルートが無ければグラフを書いて解けそうなんですがルートがあるとよくわからなくなってしまいます
914132人目の素数さん:2013/02/08(金) 23:38:33.61
>>913
倍角公式
915 忍法帖【Lv=37,xxxPT】(3+0:8) :2013/02/08(金) 23:52:24.31
解けました!
ありがとうございます
ルートの中が上手い具合に2乗になるんですね
916132人目の素数さん:2013/02/09(土) 01:00:13.90
根号はずしちゃダメじゃね?
917132人目の素数さん:2013/02/09(土) 01:26:35.35
符号を間違えなければ問題ないよ。
918132人目の素数さん:2013/02/09(土) 01:51:39.00
>>915
値はいくつになりました?
919132人目の素数さん:2013/02/09(土) 01:57:09.19
恒等的に0でない,周期2πの連続関数f(θ)が条件∫_[0,2π]f(θ)dθ=∫_[0,2π]f(θ)sinθdθ=∫_[0,2π]f(θ)=0
をみたしている.0≦θ≦2πの範囲に,方程式f(θ)=0の異なる実数解が少なくとも4つ存在することを証明せよ.
920132人目の素数さん:2013/02/09(土) 02:02:47.93
訂正します.
誤:∫_[0,2π]f(θ)dθ=∫_[0,2π]f(θ)sinθdθ=∫_[0,2π]f(θ)=0
正:∫_[0,2π]f(θ)dθ=∫_[0,2π]f(θ)sinθdθ=∫_[0,2π]f(θ)cosθ=0
921132人目の素数さん:2013/02/09(土) 02:05:13.44
>>919
ご自分の打鍵したものをよく見て、打ち直してくださいませ
922132人目の素数さん:2013/02/09(土) 02:14:19.39
訂正します.これで最後.

恒等的に0でない,周期2πの連続関数f(θ)が条件
∫_[0,2π]f(θ)dθ=∫_[0,2π]f(θ)sinθdθ=∫_[0,2π]f(θ)cosθdθ=0をみたしている.
0≦θ≦2πの範囲に,方程式f(θ)=0の異なる実数解が少なくとも4つ存在することを証明せよ.
923狢 ◆yEy4lYsULH68 :2013/02/09(土) 10:42:40.03
ワシかてそう願ってるがな。

ケケケ狢

>236 名前:132人目の素数さん :2013/02/05(火) 23:59:06.65
> そうは行きませんよ猫さん。
> 数学板は何度でも甦ります。
>
924132人目の素数さん:2013/02/09(土) 14:30:25.43
          __ノ)-'´ ̄ ̄`ー- 、_
        , '´  _. -‐'''"二ニニ=-`ヽ、
      /   /:::::; -‐''"        `ーノ
     /   /:::::/           \
     /    /::::::/          | | |  |
     |   |:::::/ /     |  | | | |  |
      |   |::/ / / |  | ||  | | ,ハ .| ,ハ|
      |   |/ / / /| ,ハノ| /|ノレ,ニ|ル' 
     |   |  | / / レ',二、レ′ ,ィイ|゙/   
.     |   \ ∠イ  ,イイ|    ,`-' |      
     |     l^,人|  ` `-'     ゝ  |        このスレは馬と鹿と豚さんばかりね。
      |      ` -'\       ー'  人            
    |        /(l     __/  ヽ、          
     |       (:::::`‐-、__  |::::`、     ヒニニヽ、         
    |      / `‐-、::::::::::`‐-、::::\   /,ニニ、\            
   |      |::::::::::::::::::|` -、:::::::,ヘ ̄|'、  ヒニ二、 \
.   |      /::::::::::::::::::|::::::::\/:::O`、::\   | '、   \
   |      /:::::::::::::::::::/:::::::::::::::::::::::::::::'、::::\ノ  ヽ、  |
  |      |:::::/:::::::::/:::::::::::::::::::::::::::::::::::'、',::::'、  /:\__/‐、
  |      |/:::::::::::/::::::::::::::::::::::::::::::::::O::| '、::| く::::::::::::: ̄|
   |     /_..-'´ ̄`ー-、:::::::::::::::::::::::::::::::::::|/:/`‐'::\;;;;;;;_|
   |    |/::::::::::::::::::::::\:::::::::::::::::::::::::::::|::/::::|::::/:::::::::::/
    |   /:::::::::::::::::::::::::::::::::|:::::::::::::::::::::O::|::|::::::|:::::::::::::::/
925915:2013/02/09(土) 17:21:26.95
>>918
√2 + 2 になりました
あっていますか?
926狢 ◆yEy4lYsULH68 :2013/02/09(土) 17:49:23.91
ワシかてそう願ってるがな。

ケケケ狢

>236 名前:132人目の素数さん :2013/02/05(火) 23:59:06.65
> そうは行きませんよ猫さん。
> 数学板は何度でも甦ります。
>
927狢 ◆yEy4lYsULH68 :2013/02/09(土) 18:01:18.30
ワシかてそう願ってるがな。

ケケケ狢

>236 名前:132人目の素数さん :2013/02/05(火) 23:59:06.65
> そうは行きませんよ猫さん。
> 数学板は何度でも甦ります。
>
928132人目の素数さん:2013/02/09(土) 18:40:03.41
>>925
(3/2)√2か?
929915:2013/02/09(土) 18:56:29.18
>>928
√(2) + 2です
まちがっているんでしょうか・・・?
930132人目の素数さん:2013/02/09(土) 19:49:29.99
y^2+xy+x^2+x-1=0の判別式ってどうやって出すの?

D=x^2-4*1*(x^2+x-1)ってなるはずなんだけど
経過がわかんない

おねがいします
931132人目の素数さん:2013/02/09(土) 20:04:07.17
          __ノ)-'´ ̄ ̄`ー- 、_
        , '´  _. -‐'''"二ニニ=-`ヽ、
      /   /:::::; -‐''"        `ーノ
     /   /:::::/           \
     /    /::::::/          | | |  |
     |   |:::::/ /     |  | | | |  |
      |   |::/ / / |  | ||  | | ,ハ .| ,ハ|
      |   |/ / / /| ,ハノ| /|ノレ,ニ|ル' 
     |   |  | / / レ',二、レ′ ,ィイ|゙/   
.     |   \ ∠イ  ,イイ|    ,`-' |      
     |     l^,人|  ` `-'     ゝ  |        このスレは馬と鹿と豚さんばかりね。
      |      ` -'\       ー'  人            
    |        /(l     __/  ヽ、          
     |       (:::::`‐-、__  |::::`、     ヒニニヽ、         
    |      / `‐-、::::::::::`‐-、::::\   /,ニニ、\            
   |      |::::::::::::::::::|` -、:::::::,ヘ ̄|'、  ヒニ二、 \
.   |      /::::::::::::::::::|::::::::\/:::O`、::\   | '、   \
   |      /:::::::::::::::::::/:::::::::::::::::::::::::::::'、::::\ノ  ヽ、  |
  |      |:::::/:::::::::/:::::::::::::::::::::::::::::::::::'、',::::'、  /:\__/‐、
  |      |/:::::::::::/::::::::::::::::::::::::::::::::::O::| '、::| く::::::::::::: ̄|
   |     /_..-'´ ̄`ー-、:::::::::::::::::::::::::::::::::::|/:/`‐'::\;;;;;;;_|
   |    |/::::::::::::::::::::::\:::::::::::::::::::::::::::::|::/::::|::::/:::::::::::/
    |   /:::::::::::::::::::::::::::::::::|:::::::::::::::::::::O::|::|::::::|:::::::::::::::/
932132人目の素数さん:2013/02/09(土) 20:09:43.06
>>930
そのまんまじゃねーか
933132人目の素数さん:2013/02/09(土) 20:13:55.06
>>930
y^2+ay+a^2+a-1=0 だったら?
934132人目の素数さん:2013/02/09(土) 20:17:22.53
>>929
∫_[0,3π/4] √(1-cos4x) dx=3∫_[0,π/4] √(2 sin^2 2x) dx=3√2∫_[0,π/4] sin2x dx=3√2/2

>>930
ay^2+by+c=y^2+xy+x^2+x-1 → a=1, b=x, c=x^2+x-1 を聞いてるの?
935狢 ◆yEy4lYsULH68 :2013/02/09(土) 20:21:02.24
ワシかてそう願ってるがな。

ケケケ狢

>236 名前:132人目の素数さん :2013/02/05(火) 23:59:06.65
> そうは行きませんよ猫さん。
> 数学板は何度でも甦ります。
>
936132人目の素数さん:2013/02/09(土) 20:25:33.26
>>934
あ、なるほど
理解できた

ありがとう
937狢 ◆yEy4lYsULH68 :2013/02/09(土) 20:26:22.91
ワシかてそう願ってるがな。

ケケケ狢

>236 名前:132人目の素数さん :2013/02/05(火) 23:59:06.65
> そうは行きませんよ猫さん。
> 数学板は何度でも甦ります。
>
938132人目の素数さん:2013/02/09(土) 20:30:38.24
          __ノ)-'´ ̄ ̄`ー- 、_
        , '´  _. -‐'''"二ニニ=-`ヽ、
      /   /:::::; -‐''"        `ーノ
     /   /:::::/           \
     /    /::::::/          | | |  |
     |   |:::::/ /     |  | | | |  |
      |   |::/ / / |  | ||  | | ,ハ .| ,ハ|
      |   |/ / / /| ,ハノ| /|ノレ,ニ|ル' 
     |   |  | / / レ',二、レ′ ,ィイ|゙/   
.     |   \ ∠イ  ,イイ|    ,`-' |      
     |     l^,人|  ` `-'     ゝ  |        このスレは馬と鹿と豚さんばかりね。
      |      ` -'\       ー'  人            
    |        /(l     __/  ヽ、          
     |       (:::::`‐-、__  |::::`、     ヒニニヽ、         
    |      / `‐-、::::::::::`‐-、::::\   /,ニニ、\            
   |      |::::::::::::::::::|` -、:::::::,ヘ ̄|'、  ヒニ二、 \
.   |      /::::::::::::::::::|::::::::\/:::O`、::\   | '、   \
   |      /:::::::::::::::::::/:::::::::::::::::::::::::::::'、::::\ノ  ヽ、  |
  |      |:::::/:::::::::/:::::::::::::::::::::::::::::::::::'、',::::'、  /:\__/‐、
  |      |/:::::::::::/::::::::::::::::::::::::::::::::::O::| '、::| く::::::::::::: ̄|
   |     /_..-'´ ̄`ー-、:::::::::::::::::::::::::::::::::::|/:/`‐'::\;;;;;;;_|
   |    |/::::::::::::::::::::::\:::::::::::::::::::::::::::::|::/::::|::::/:::::::::::/
    |   /:::::::::::::::::::::::::::::::::|:::::::::::::::::::::O::|::|::::::|:::::::::::::::/
939狢 ◆yEy4lYsULH68 :2013/02/09(土) 20:41:38.80
コレは一体どういう意味なんですかね?
★★★『阪大基礎工あがりの人』★★★
何だか蔑みの様にも、また見下しの様にも見えませんかね。日本の学歴
階層構造というのか、或いは理学部が他所を見下してるのか、極めて不
思議な価値観を醸し出してますわナ。

ケケケ狢

>785 :132人目の素数さん:2013/02/02(土) 16:27:31.55
> >>782
> 極端な平等主義?
>
> あほか。
> だから阪大基礎工あがりの人でも数学者になれたんだろ。
>
> 東大、京大って言ったって、
> 高校数学の学力試験を勝ち抜いたくらいで大きい顔をされてもね。
> (しかも、数学では差がつかずに、他の古文、漢文、日本史、世界史などの
> 教科で得点に差がついただけ)
>
> 結集する意味なし。
> 別にカリキュラムに沿ってお勉強してるんじゃあるまいし。
> 天才はどこでも育つ。個人の問題だから。
>
> 余裕のあるところで、自分で好き勝手なことをやってればいい。
> 特にこれからの時代、既存の難問を解いてるだけの数学者よりも
> 問題を見つけ出す数学者が必要とされる。
> 秀才型数学者は黙ってろ、って。
>
940132人目の素数さん:2013/02/09(土) 21:09:21.35
x^2log{2}3+2xlog{2}(a)+log{2}(3)≧x^2+2x+1

これを満たすaの範囲を求めよ


よくわかんね。
941915:2013/02/09(土) 21:26:07.65
計算ミスしてました
ありがとうございます
942狢 ◆yEy4lYsULH68 :2013/02/09(土) 21:52:19.01
コレは一体どういう意味なんですかね?
★★★『阪大基礎工あがりの人でも数学者になれたんだろ』★★★
何だか蔑みの様にも、また見下しの様にも見えませんかね。日本の学歴
階層構造というのか、或いは理学部が他所を見下してるのか、極めて不
思議な価値観を醸し出してますわナ。コレをもし:
★★★『日本人如き(のサル)でも数学者になれたんだろ』★★★
な〜んてどっかの国の誰かが言ったら怒るんですかね、ソレとも褒め言
葉なんで嬉しがるべきなんですかね?

ケケケ狢

>785 :132人目の素数さん:2013/02/02(土) 16:27:31.55
> >>782
> 極端な平等主義?
>
> あほか。
> だから阪大基礎工あがりの人でも数学者になれたんだろ。
>
> 東大、京大って言ったって、
> 高校数学の学力試験を勝ち抜いたくらいで大きい顔をされてもね。
> (しかも、数学では差がつかずに、他の古文、漢文、日本史、世界史などの
> 教科で得点に差がついただけ)
>
> 結集する意味なし。
> 別にカリキュラムに沿ってお勉強してるんじゃあるまいし。
> 天才はどこでも育つ。個人の問題だから。
>
> 余裕のあるところで、自分で好き勝手なことをやってればいい。
> 特にこれからの時代、既存の難問を解いてるだけの数学者よりも
> 問題を見つけ出す数学者が必要とされる。
> 秀才型数学者は黙ってろ、って。
>
943132人目の素数さん:2013/02/09(土) 23:32:59.91
(1)a,b,c,d の 4 人を 2人ずつ 2 つの部屋に分ける。数え上げれば
 ab, ac, ad, bc, bd, cd
 部屋の区別がないのだから 1 つの部屋に入る方法だけを考える。よって 4C2 = 12/2 = 6.
(2)区別のできる 6 個の球を 3 個ずつ 2 つの箱に分ける。
 箱の区別がないのだから 1 つの箱に入る方法だけを考える。よって 6C3 = 120/6 = 20.
 ところが数え上げてみると 10 通りでよい。(1)と(2)の違いを教えてください。
     abc  def  |   bcd  efa
     abd  efc  |   bce  fad
     abe  fcd  |   bcf  ade
     abf  cde  |   bde  fac
     acd  efb  |   bdf  ace
     ace  fbd  |   bef  acd
     acf  bde  |   cde  fab
     ade  fbc  |   cdf  abe
     adf  bce  |   cef  abd
     aef  bcd  |   def  abc
944132人目の素数さん:2013/02/09(土) 23:37:32.96
>>943
(2)と同じように(1)を数え上げてみ
945132人目の素数さん:2013/02/09(土) 23:49:35.77
>>943
(1)は部屋の区別をしている
もし部屋を区別しないならaの同室者を選ぶだけだから3通り


(2)
球をabcdefとしたらaと同じ箱に入る球の選び方C[5,2]=10通り
946943:2013/02/10(日) 00:03:46.40
 あっ!
 そうかぁ!! (1)は区別してたんですね。ありがとうございました。
947132人目の素数さん:2013/02/10(日) 00:21:40.12
          __ノ)-'´ ̄ ̄`ー- 、_
        , '´  _. -‐'''"二ニニ=-`ヽ、
      /   /:::::; -‐''"        `ーノ
     /   /:::::/           \
     /    /::::::/          | | |  |
     |   |:::::/ /     |  | | | |  |
      |   |::/ / / |  | ||  | | ,ハ .| ,ハ|
      |   |/ / / /| ,ハノ| /|ノレ,ニ|ル' 
     |   |  | / / レ',二、レ′ ,ィイ|゙/   
.     |   \ ∠イ  ,イイ|    ,`-' |      
     |     l^,人|  ` `-'     ゝ  |        このスレは馬と鹿と豚さんばかりね。
      |      ` -'\       ー'  人            
    |        /(l     __/  ヽ、          
     |       (:::::`‐-、__  |::::`、     ヒニニヽ、         
    |      / `‐-、::::::::::`‐-、::::\   /,ニニ、\            
   |      |::::::::::::::::::|` -、:::::::,ヘ ̄|'、  ヒニ二、 \
.   |      /::::::::::::::::::|::::::::\/:::O`、::\   | '、   \
   |      /:::::::::::::::::::/:::::::::::::::::::::::::::::'、::::\ノ  ヽ、  |
  |      |:::::/:::::::::/:::::::::::::::::::::::::::::::::::'、',::::'、  /:\__/‐、
  |      |/:::::::::::/::::::::::::::::::::::::::::::::::O::| '、::| く::::::::::::: ̄|
   |     /_..-'´ ̄`ー-、:::::::::::::::::::::::::::::::::::|/:/`‐'::\;;;;;;;_|
   |    |/::::::::::::::::::::::\:::::::::::::::::::::::::::::|::/::::|::::/:::::::::::/
    |   /:::::::::::::::::::::::::::::::::|:::::::::::::::::::::O::|::|::::::|:::::::::::::::/
948132人目の素数さん:2013/02/10(日) 09:17:17.25
z=2-s+s*sint・・・@について
dz/dtを求めるときに
dz/dt=s*costとなっているのですが、
ここでのsは定数扱いなんですか?
でも、y=x^2・・・Aをyで微分するときは
1=2x*dx/dyとなり、xを定数扱いすることはできません。
まぁ、xはyの関数でありyはxの関数であるから合成関数の微分法になってるんですが、@式についてはすぐにどれとどれが関数になってるのか分かりません。
sはなぜ定数扱いになるのでしょうか?
@式では2も定数、sも定数となるんですか?

「@とAを微分するときに気をつけることが違うのはなぜでしょう?」
949132人目の素数さん:2013/02/10(日) 09:24:22.16
>>948
自分で書いてるけど、sがtの関数じゃない(tが変化してもsが変化しない)から。
950132人目の素数さん:2013/02/10(日) 09:24:58.04
>>948
dz/dtなんだからtで微分では?
特に断りがなければzはtの関数で、それ以外は定数と考えていいでしょ。
951132人目の素数さん:2013/02/10(日) 10:26:27.64
誰か>>940頼む!
952132人目の素数さん:2013/02/10(日) 10:29:55.36
判別式で瞬殺
953132人目の素数さん:2013/02/10(日) 10:40:30.13
>>940
式を正確に
954132人目の素数さん:2013/02/10(日) 11:00:51.42
定期テストとかでの成績がなかなか上がらないのですが何がいけないのかがわかりません
学校でもらった教材をしっかりとやっているのに
誰か教えてください
955132人目の素数さん:2013/02/10(日) 11:28:59.38
しっかりとはやってないんだろう。
956132人目の素数さん:2013/02/10(日) 12:01:43.22
だな
957132人目の素数さん:2013/02/10(日) 12:14:58.16
そ、そんな…
958132人目の素数さん:2013/02/10(日) 13:01:40.33
>>954
まずはテメェのセンコーに聞け
そいつが教えてクンネーならあきらめろ
そいつは教える気ねぇから
959狢 ◆yEy4lYsULH68 :2013/02/10(日) 13:10:05.98
コレは一体どういう意味なんですかね?
★★★『阪大基礎工あがりの人でも数学者になれたんだろ』★★★
何だか蔑みの様にも、また見下しの様にも見えませんかね。日本の学歴
階層構造というのか、或いは理学部が他所を見下してるのか、極めて不
思議な価値観を醸し出してますわナ。コレをもし:
★★★『日本人如き(のサル)でも数学者になれたんだろ』★★★
な〜んてどっかの国の誰かが言ったら怒るんですかね、ソレとも褒め言
葉なんで嬉しがるべきなんですかね?

ケケケ狢

>785 :132人目の素数さん:2013/02/02(土) 16:27:31.55
> >>782
> 極端な平等主義?
>
> あほか。
> だから阪大基礎工あがりの人でも数学者になれたんだろ。
>
> 東大、京大って言ったって、
> 高校数学の学力試験を勝ち抜いたくらいで大きい顔をされてもね。
> (しかも、数学では差がつかずに、他の古文、漢文、日本史、世界史などの
> 教科で得点に差がついただけ)
>
> 結集する意味なし。
> 別にカリキュラムに沿ってお勉強してるんじゃあるまいし。
> 天才はどこでも育つ。個人の問題だから。
>
> 余裕のあるところで、自分で好き勝手なことをやってればいい。
> 特にこれからの時代、既存の難問を解いてるだけの数学者よりも
> 問題を見つけ出す数学者が必要とされる。
> 秀才型数学者は黙ってろ、って。
>
960132人目の素数さん:2013/02/10(日) 13:14:33.30
>>949
zが定数なら、合成関数の微分法ですか?

本当は断面積Sがsとtで表されていて(0<s<1)それを「z=2-2sからz=2まで」・・・@動かしてSを積分する話なんですが(切断面はz=2-s+s*sintです)、sとtで表されている断面積をz軸方向に積分するのはイメージがつきます。

でも、ここからtで置換積分するんですが「断面積にzがないので、tで置換積分する」という解釈でいいですか?
ちなみに-π/2≦t≦π/2に対応してます@は。

Q.ここではただの計算方法を変えるだけで
もはや、イメージはできませんよね?
Q.これをsで積分とかできますか?

あと話変わるんですけど、y=x^2ってyの値を一つ決めると例によってxの値が2つでるときありますよね?
これって関数ですか?
xの値を1つ決めるとyの値は1つ定まるのでxはyの関数なのは分かりますが。
961132人目の素数さん:2013/02/10(日) 13:38:34.72
小出しにするくらいなら問題文を正確に書け
問題文がないと答えられん
後者は関数の定義による
まあ一般的な定義では多価関数は関数と扱わない
962132人目の素数さん:2013/02/10(日) 14:52:39.95
>>960
設定がさっぱりわからない。抽象論なら>>949に戻る。
963132人目の素数さん:2013/02/10(日) 15:04:52.73
すみませんでした。
964132人目の素数さん:2013/02/10(日) 15:19:12.15
すみません。

http://beebee2see.appspot.com/i/azuYiYrlBww.jpg
(1)P,Q,Rを含む平面の方程式を求めよ。
(2)RP=RQを示せ。

点Qは、点Rを中心としRPを半径とする円周上に存在する。このとき、弦PQに対する弧PQと、半径RPおよび半径RQで囲まれる扇形をCとする。ただし、Cの中心角∠PRQはπ以下とする。
(3)Cの面積をsとtを用いて表せ。
(4)tが-π/2≦t≦π/2の範囲を動くとき、Rのzざひの動く範囲をsを用いて表せ。
(5)tが-π/2≦t≦π/2の範囲を動くとき、扇形Cが通過する部分の体積V1をsを用いて表せ。

(1)はz=(2-s)+s*sint
(3)は面積をSとすると、
S=π/8(2-s+s*cost)^2
(4)は2-2s≦z≦2
となり、ここまでは分かってます。
(5)で本題になるんですが
解答は

『(4)を導く過程より、tが-π/2≦t≦π/2の範囲を動くとき、平面z=2-s+s*sint上にある扇形Cは、平面z=2-2sから平面z=2までの間をz座標が増える方向へ通過していく。
つまり、この通過する部分の、平面z=2-s+s*sintによる切り口が扇形Cということである。
よって
[α,β]←下端α、上端βの意

V1=∫[2-2s,2]S dz
=∫[-π/2,π/2]S (dz/dt)*dt』
965132人目の素数さん:2013/02/10(日) 15:31:58.05
となり、tで置換積分しています。
やってることは分かりますが、
Sがzを含んでいないのでs,tで置換積分しか他なく、都合よく-π/2≦t≦π/2を動くとき2-2s≦z≦2となり対応してる、OK。
なのでtで置換積分しよう。


という解釈でいいですか?極端に言ってますが。
sで置換積分することはできないですか?
断面積をz軸方向に積分することはイメージできますが、tで置換積分するところは計算を同値的なもの,楽なものにするだけで、積分のイメージはなくなりますよね?(足して寄せ集めてみたいな)
966132人目の素数さん:2013/02/10(日) 15:57:58.23
次の問題が分かりません。お手上げ状態です。

自然数を2乗した数(平方数)の下4桁が同じ数なら、その同じ数は0である。
967132人目の素数さん:2013/02/10(日) 16:24:38.45
968132人目の素数さん:2013/02/10(日) 16:34:01.33
sinθ=1/nのとき 

h*tanθ=x

ですおねがします。
969132人目の素数さん:2013/02/10(日) 16:41:03.75
>>968
これ思い出した

「閣下、(a+b^n)/n=x 故に神は存在する。如何か?」
970狢 ◆yEy4lYsULH68 :2013/02/10(日) 17:37:21.34
コレは一体どういう意味なんですかね?
★★★『阪大基礎工あがりの人でも数学者になれたんだろ』★★★
何だか蔑みの様にも、また見下しの様にも見えませんかね。日本の学歴
階層構造というのか、或いは理学部が他所を見下してるのか、極めて不
思議な価値観を醸し出してますわナ。コレをもし:
★★★『日本人如き(のサル)でも数学者になれたんだろ』★★★
な〜んてどっかの国の誰かが言ったら怒るんですかね、ソレとも褒め言
葉なんで嬉しがるべきなんですかね?

ケケケ狢

>785 :132人目の素数さん:2013/02/02(土) 16:27:31.55
> >>782
> 極端な平等主義?
>
> あほか。
> だから阪大基礎工あがりの人でも数学者になれたんだろ。
>
> 東大、京大って言ったって、
> 高校数学の学力試験を勝ち抜いたくらいで大きい顔をされてもね。
> (しかも、数学では差がつかずに、他の古文、漢文、日本史、世界史などの
> 教科で得点に差がついただけ)
>
> 結集する意味なし。
> 別にカリキュラムに沿ってお勉強してるんじゃあるまいし。
> 天才はどこでも育つ。個人の問題だから。
>
> 余裕のあるところで、自分で好き勝手なことをやってればいい。
> 特にこれからの時代、既存の難問を解いてるだけの数学者よりも
> 問題を見つけ出す数学者が必要とされる。
> 秀才型数学者は黙ってろ、って。
>
971132人目の素数さん:2013/02/10(日) 18:03:54.96
          __ノ)-'´ ̄ ̄`ー- 、_
        , '´  _. -‐'''"二ニニ=-`ヽ、
      /   /:::::; -‐''"        `ーノ
     /   /:::::/           \
     /    /::::::/          | | |  |
     |   |:::::/ /     |  | | | |  |
      |   |::/ / / |  | ||  | | ,ハ .| ,ハ|
      |   |/ / / /| ,ハノ| /|ノレ,ニ|ル' 
     |   |  | / / レ',二、レ′ ,ィイ|゙/   
.     |   \ ∠イ  ,イイ|    ,`-' |      
     |     l^,人|  ` `-'     ゝ  |        このスレは馬と鹿と豚さんばかりね。
      |      ` -'\       ー'  人            
    |        /(l     __/  ヽ、          
     |       (:::::`‐-、__  |::::`、     ヒニニヽ、         
    |      / `‐-、::::::::::`‐-、::::\   /,ニニ、\            
   |      |::::::::::::::::::|` -、:::::::,ヘ ̄|'、  ヒニ二、 \
.   |      /::::::::::::::::::|::::::::\/:::O`、::\   | '、   \
   |      /:::::::::::::::::::/:::::::::::::::::::::::::::::'、::::\ノ  ヽ、  |
  |      |:::::/:::::::::/:::::::::::::::::::::::::::::::::::'、',::::'、  /:\__/‐、
  |      |/:::::::::::/::::::::::::::::::::::::::::::::::O::| '、::| く::::::::::::: ̄|
   |     /_..-'´ ̄`ー-、:::::::::::::::::::::::::::::::::::|/:/`‐'::\;;;;;;;_|
   |    |/::::::::::::::::::::::\:::::::::::::::::::::::::::::|::/::::|::::/:::::::::::/
    |   /:::::::::::::::::::::::::::::::::|:::::::::::::::::::::O::|::|::::::|:::::::::::::::/
972132人目の素数さん:2013/02/10(日) 18:57:24.65
>>952
若干亀レスだが

やってみたら本当に瞬殺だったわ

微分やらなんやら考えすぎたよ

サンクス
973132人目の素数さん:2013/02/10(日) 19:03:37.27


高校数学の質問スレPART347
http://uni.2ch.net/test/read.cgi/math/1360490569/


-
974132人目の素数さん:2013/02/10(日) 19:46:31.01
>>964,>>965
分かりませんか?
975132人目の素数さん:2013/02/10(日) 19:47:45.76
.ap@safe
976132人目の素数さん:2013/02/10(日) 19:50:57.72
次の関数のグラフをかけ
y=x+cosX   (0≦x≦2π)

という問題で解答を見るとy=xのグラフも書かれてありました
y=xのグラフも書いてないとだめなんですか?
977132人目の素数さん:2013/02/10(日) 20:19:51.26
いいえ
978132人目の素数さん:2013/02/10(日) 21:25:23.66
31÷√48÷√28ってどうやって計算したら0.845...になるの?
電卓なしでできますか?
979132人目の素数さん:2013/02/10(日) 21:32:24.74
(31 / √(48)) / √(28) = 0.845

この順序でやれば計算できる
平方根の計算を知っていればでんたくナシで計算できる
980132人目の素数さん:2013/02/10(日) 21:40:36.50
答えは0.845じゃなくてそのあとに無限に小数が続くんだけど約0.845ってことです
31√21/168からどうやってそれに持っていくのかわからない
981132人目の素数さん:2013/02/10(日) 21:52:45.76
筆算で √21=4.5825 まで出せば 0.845 までは求まる
982132人目の素数さん:2013/02/10(日) 21:58:26.99
          __ノ)-'´ ̄ ̄`ー- 、_
        , '´  _. -‐'''"二ニニ=-`ヽ、
      /   /:::::; -‐''"        `ーノ
     /   /:::::/           \
     /    /::::::/          | | |  |
     |   |:::::/ /     |  | | | |  |
      |   |::/ / / |  | ||  | | ,ハ .| ,ハ|
      |   |/ / / /| ,ハノ| /|ノレ,ニ|ル' 
     |   |  | / / レ',二、レ′ ,ィイ|゙/   
.     |   \ ∠イ  ,イイ|    ,`-' |      
     |     l^,人|  ` `-'     ゝ  |        このスレには馬と鹿と豚さんしかいないのね。
      |      ` -'\       ー'  人            
    |        /(l     __/  ヽ、          
     |       (:::::`‐-、__  |::::`、     ヒニニヽ、         
    |      / `‐-、::::::::::`‐-、::::\   /,ニニ、\            
   |      |::::::::::::::::::|` -、:::::::,ヘ ̄|'、  ヒニ二、 \
.   |      /::::::::::::::::::|::::::::\/:::O`、::\   | '、   \
   |      /:::::::::::::::::::/:::::::::::::::::::::::::::::'、::::\ノ  ヽ、  |
  |      |:::::/:::::::::/:::::::::::::::::::::::::::::::::::'、',::::'、  /:\__/‐、
  |      |/:::::::::::/::::::::::::::::::::::::::::::::::O::| '、::| く::::::::::::: ̄|
   |     /_..-'´ ̄`ー-、:::::::::::::::::::::::::::::::::::|/:/`‐'::\;;;;;;;_|
   |    |/::::::::::::::::::::::\:::::::::::::::::::::::::::::|::/::::|::::/:::::::::::/
    |   /:::::::::::::::::::::::::::::::::|:::::::::::::::::::::O::|::|::::::|:::::::::::::::/
983132人目の素数さん:2013/02/10(日) 22:05:03.18
結局√21の近似値を求めなきゃだめなのか
もっとスマートに解けない?
984132人目の素数さん:2013/02/10(日) 22:16:27.75
(31√3/64)*√(128/126)
√(128/126)の誤差を呑めるならその前の計算だけでいい
985132人目の素数さん:2013/02/10(日) 23:03:39.80
何贅沢なことほざいてんだか
986132人目の素数さん:2013/02/11(月) 02:46:25.28
次の同次積分方程式の固有値および解を求めよ

φ(x) = λ∫xtφ(t)dt

ただし積分範囲はt=0, 1
λは定数、x,tは変数(x, t∈R)

お願いします。
987132人目の素数さん:2013/02/11(月) 07:38:31.31
そういうスレじゃない
988132人目の素数さん:2013/02/11(月) 08:47:03.45
すみません。

http://beebee2see.appspot.com/i/azuYiYrlBww.jpg
(1)P,Q,Rを含む平面の方程式を求めよ。
(2)RP=RQを示せ。

点Qは、点Rを中心としRPを半径とする円周上に存在する。このとき、弦PQに対する弧PQと、半径RPおよび半径RQで囲まれる扇形をCとする。ただし、Cの中心角∠PRQはπ以下とする。
(3)Cの面積をsとtを用いて表せ。
(4)tが-π/2≦t≦π/2の範囲を動くとき、Rのzざひの動く範囲をsを用いて表せ。
(5)tが-π/2≦t≦π/2の範囲を動くとき、扇形Cが通過する部分の体積V1をsを用いて表せ。

(1)はz=(2-s)+s*sint
(3)は面積をSとすると、
S=π/8(2-s+s*cost)^2
(4)は2-2s≦z≦2
となり、ここまでは分かってます。
(5)で本題になるんですが
解答は

『(4)を導く過程より、tが-π/2≦t≦π/2の範囲を動くとき、平面z=2-s+s*sint上にある扇形Cは、平面z=2-2sから平面z=2までの間をz座標が増える方向へ通過していく。
つまり、この通過する部分の、平面z=2-s+s*sintによる切り口が扇形Cということである。
よって
[α,β]←下端α、上端βの意

V1=∫[2-2s,2]S dz
=∫[-π/2,π/2]S (dz/dt)*dt』
989132人目の素数さん:2013/02/11(月) 09:00:41.50
となり、tで置換積分しています。
やってることは分かりますが、
Sがzを含んでいないのでs,tで置換積分しか他なく、都合よく-π/2≦t≦π/2を動くとき2-2s≦z≦2となり対応してる、OK。
なのでtで置換積分しよう。


という解釈でいいですか?極端に言ってますが。
sで置換積分することはできないですか?
断面積をz軸方向に積分することはイメージできますが、tで置換積分するところは計算を同値的なもの,楽なものにするだけで、積分のイメージはなくなりますよね?(足して寄せ集めてみたいな)
990132人目の素数さん:2013/02/11(月) 09:12:11.19
ゆうくん知恵袋に帰ったら?w
991132人目の素数さん:2013/02/11(月) 10:03:45.55
変数と定数が分かって無いってのは致命的だよね
丁寧な誘導がついてわざわざ定数と考えてくれって書いてあるのにそれが分からないのかな
空気読めない感じだし出題者の意図も読めないのかな
992132人目の素数さん:2013/02/11(月) 10:56:17.48
>>991
ん?よく分かりません。
質問に詳しく答えてくだされば幸いです。
993132人目の素数さん:2013/02/11(月) 11:02:50.35
>>992
アスペだから分からないらしいけど
お前はバカだって言われてるんだよ
994132人目の素数さん:2013/02/11(月) 11:03:27.77
>>993
あの、質問に答えてください。
馬鹿とかは今どうでもいいです。
995132人目の素数さん:2013/02/11(月) 11:32:19.96
>>986
定数部分を文字で置け。
996132人目の素数さん:2013/02/11(月) 11:45:37.61
けっきょく>>949
997132人目の素数さん:2013/02/11(月) 11:54:48.88
独立な上に定数として考えろって指示があるようなもん
定数で積分とか池沼だろ

独立なんだから積分区間どうなるよ?って以前に
定数として考えてないなら置換した積分出来ないし
色々考えてるフリしてるけど結局なんも分かってなくて雰囲気で数学やってるのが丸わかりって質問
998132人目の素数さん:2013/02/11(月) 12:22:22.45
>>986
λ=3, φ(x)=Cx
999132人目の素数さん:2013/02/11(月) 13:19:30.77
高校数学の質問スレPART347
http://uni.2ch.net/test/read.cgi/math/1360490569/
1000132人目の素数さん:2013/02/11(月) 13:21:03.73
わいがはじめての1000ゲットや
10011001
このスレッドは1000を超えました。
もう書けないので、新しいスレッドを立ててくださいです。。。